Nursing Study Guide

227
2 Nursing Formulas and Conversions Drugs and Dosage Formulas and Conversions Volume 60 minims = 1 dram = 5cc = 1tsp 4 drams = 0.5 ounces = 1tbsp 8 drams = 1 ounce 16 ounces = 1pt. 32 ounces = 1qt. Weight 60 grains = 1dram 1/60 grain=1mg 8 drams = 1 ounce 15 grains=1g 12 ounces = 1 lb. (apothecaries') 2.2 lbs.=1kg Household Apothecary 1tsp = 1 dram 1tsp = 60 gtts (drops) 3tsp = 0.5 ounce 1tbsp = 0.5 ounce Household Apothecary Metric 1tsp=5cc 1fl.dram=4cc 5cc=1tsp 3tsp=1tbsp 4drams=0.5oz 15cc=1tbsp 1tbsp=0.5oz or 15cc 8drams=2tbsp(1oz) 30cc=2tbsp(1oz) 2tbsp=1oz or 30cc 16minims=1cc 1cc=16minims 1pt.=16oz or 480cc 500cc=0.5L or 1pt. 1qt=32oz or 960cc 1000cc=1L or 1qt. Temp. Conversion C= F-32/1.8 F= 1.8*C-32

Transcript of Nursing Study Guide

2

Nursing Formulas and Conversions

Drugs and Dosage Formulas and Conversions

Volume

60 minims = 1 dram = 5cc = 1tsp 4 drams = 0.5 ounces = 1tbsp

8 drams = 1 ounce 16 ounces = 1pt. 32 ounces = 1qt.

Weight 60 grains = 1dram 1/60 grain=1mg 8 drams = 1 ounce 15 grains=1g

12 ounces = 1 lb. (apothecaries') 2.2 lbs.=1kg

Household Apothecary 1tsp = 1 dram

1tsp = 60 gtts (drops) 3tsp = 0.5 ounce 1tbsp = 0.5 ounce

Household Apothecary Metric 1tsp=5cc 1fl.dram=4cc 5cc=1tsp 3tsp=1tbsp 4drams=0.5oz 15cc=1tbsp

1tbsp=0.5oz or 15cc 8drams=2tbsp(1oz) 30cc=2tbsp(1oz) 2tbsp=1oz or 30cc 16minims=1cc 1cc=16minims

1pt.=16oz or 480cc 500cc=0.5L or 1pt. 1qt=32oz or 960cc 1000cc=1L or 1qt.

Temp. Conversion

C= F-32/1.8 F= 1.8*C-32

3

NOTES CARDIOVASCULAR Arterial Ulcer – Pale, deep base, surrounded by tissue that is cool with trophic changes such as dry, soluble skin and loss of hair. Cause by ischemia from inadequate arterial blood supply of oxygen and nutrients . Venous stasis Ulcer – Dark, red base, surrounded by skin that is brown in color with edema. Caused by the accumulation of waste products of metabolism that are not cleared due to venous congestion. Stage I Ulcer – Reddened area with intact skin surface. Management of DVT – Bed rest, limb elevation , relief of discomfort with warm, moist, heat and analgesics (Tylenol, not narcotics) prn. Ambulation is contraindicated. Sclerotherapy – Injection of a sclerosing agent into a varicosity. The agent damages the vessel and causes aseptic thrombosis that result in vein closure. With no blood flow thru the vessel, distention will not occur. The surgical procedure for varicose veins is vein ligation and stripping: tying off the varicose veins and large tributaries and then removal of the vein with the use of hooks and wires via multiple small incisions in the leg Cardiac Tamponade – Tachycardia, muffled of distant heart sounds, JVD, a falling BP accompanied by pulsus paradoxus (a drop in BP on inspiration of more than 10mm Hg) Self care of an ischemic leg ulcer – Same as Peripheral Arterial Disease / Diabetes Mellitus Sodium containing – Toothpaste, mouthwash, OTC meds such as analgesics, antacid, laxatives, and sedatives. Softened water and some mineral water. Fresh fruits and vegetables are low in sodium, INR – Normal 2.0 – 3.0 Should be 2.0 to 3.0 for anticoagulant therapy (Coumadin) Should be 2.5 to 3.5 for Mechanical prosthetic heart valve patients and survivors of acute MI IABP therapy for Cardiogenic Shock. Contraindicated for aortic insufficiency or aortic or thoracic aneurysms. Not used for CHF or pulmonary edema. Atrial fibrillation with ventricular rate of 150? Check for hypotension!! Patient is at risk for decreased cardiac output due to loss of atrial kick. Assessment includes palpitations, chest pain or discomfort, hypotension, pulse deficit, fatigue, weakness, dizziness, syncope, SOB, and distended neck veins. RSHF – Peripheral and sacral edema, JVD, and organomegaly. LSHF – Lung sounds, rales at the bases. Quinidine Sulfate for treatment of atrial fibrillation or atrial flutter – Only given after client has been digitalized. Take exactly as prescribed, do not chew the extended release capsules or open the capsules and mix them with food. Wear a Medic-Alert bracelet and to continue taking digoxin as prescribed PR interval – Impulse from the atrial to the ventricles: Normal is 0.12 to 0.20 seconds Ventricular tachycardia – Check for client’s unresponsiveness. This way you can determine if the client ifs affected by the decreased cardiac output caused by the VT state

4

Pneumatic Antishock Garment (PASG) for the treatment of hypovolemic shock provides circulatory assistance. Used temporarily, because it can compromise blood flow to the lower half of the body. Critical nursing assessment includes: Vascular status of lower extremities, Left Atrial Pressure – Normal if 1 – 10mm Hg Fibrinogen – Normal is 180 to 340mg/dl for males 190 to 420mg/dl for females Critical Value is less than 100mg/dl With DIC, fibrinogen levels drops as it is used up in the clotting process Raynaud’s Disease – PVD characterized by abnormal vasoconstriction of the extremities. Smoking cessation is one of the important lifestyle changes that the nurse can influence with teaching. The nurse should emphasize the effects of tobacco on the blood vessels and the principles involved to stop smoking. Provide community program information. Lidocaine to PVCs, if allergic to lidocaine, Procanaimide (Pronestyl), Othe drugs used are Bretylol (Bretylium tosylate) and magnesium sulfate PVCs are dangerous when they are frequent (more than 6 in a minute), occur in pairs or couplets, are multifocal (multiform) or fall on the T wave. In each of these cases, the cardiac rhythm could turn into Ventricular Tachycardia or Ventricular Fibrillation. Cardioversion – Airway, O2 administration, assessment of vitals signs and level of consciousness, and dysrrhythmia detection. Pacemakers – Atrial – Pacer spike before P waves Ventricular – Pacer spike followed by QRS complex Demand – Fires only when needed (when there’s no electrical activity in the client’s heart). DVT – Edema distal to the obstruction, warmth and redness on affected leg, tenderness, possible dilated veins, increased calf circumference. Pedal pulses are unchanged, because this is not an arterial problem, but a venous. Buerger’s Disease – Smoking is highly detrimental in clients with this disease Alpha 1 Receptors – Found in the peripheral arteries and veins, they cause powerful vasoconstriction when stimulated. Alpha 2 Receptors – Found in several tissues, contract smooth muscle inhibit lipolysis, and promote platelet aggregation. Beta 1 Receptors – Found in the heart and cause an increased heart rate, AV conduction and contractility. Beta 2 Receptors – Found in arterial and bronchial walls and cause vasodilation and bronchodilation. Hypothermia – Causes the heart rate and the Blood pressure to decrease. Vagus Nerve stimulation – Decreases heart rate and contractility. CK MB – Cardiac muscle CK MM – Skeletal muscle CK BB – Brain tissue Digoxin Therapeutic Level – 0.5 to 2.0 ng/ml. If > than 2.0 Notify Physician after doing an assessment.

5

PTT – Should be 1.5 to 2.5 times the control. PT – Should be 1.5 to 2 times the control. Isoenzymes that are affected during a Myocardial infarction include: LDH1 and LDH2 These enzymes being to elevate after 24 hours of a myocardial infarction and peak in 48 to 72 hours. They return to normal in 7 – 14 days. Troponin T normal level 0.1 - 0.2ng/dl. Normal is less than 0.6ng/dl, a level of 1.5ng/dl is consistent with a MI Client unresponsive, no pulse. Open airway, Initiate ventilation, initiate chest compressions. Thallium 201 Scan – This procedure dilates the coronary arteries as exercise would. Before the procedure, any form of caffeine should be withheld, as well as theophylline or aminophylline. Heart failure is precipitated by physical or emotional stress, dysrrhythmias, infections, anemia, thyroid disorders, pregnancy, Paget’s disease, nutritional deficiencies (thiamine, alcoholism), pulmonary disease, and hypervolemia. Digoxin is the medication of choice fro treatment of Heart Failure. Pulmonary edema – extreme breathlessness, dyspnea, air hunger, and production of frothy pink-tinged sputum. Rales to the apices. Digoxin and Furosemide – Check Potassium level!! V tach treatment – In client who is hemodynamically stable is Lidocaine (Xylocaine), Pronestyl and Bretylol. Cardioversion may be needed to correct the rhythm. Defibrillation is used only when there is LOC. Epinephrine is contraindicated b/c it would stimulate the ventricle. Cough CPR – In a stable V Tach patient. Inhale deeply and cough forcefully every 1-3 seconds. This may terminate the dysrrhythmias or sustain the cerebral or coronary circulations until other measures are implemented. PVCs – Worry about BP and peripheral perfusion. Defibrillator levels – 200 joules, 300 joules and 360 joules. Successful response to defibrillation – Arousable status, adequate BP and a sinus rhythm. AICD –Internal Automatic Cardioverter Defibrillator Used for patients who have survived sudden cardiac death unrelated to MI Those refractive to medication therapy Those with syncopal episodes orelated to Ventricualr Tachycardia. AICD postinsertion – Assess if it is activated, the heart rate cutoff above which it will fire, and the number of shocks it is programmed to deliver. Aminocaproic acid is the antidote for thrombolytic therapy. Vitamin K is the antidote for Coumadin Protamine Sulfate is the antidote for Heparin Warfarin Sodium – Inhibits synthesis in the liver of clotting factors X, IX, VII and II. Takes 3 -4 days to start acting. Thromboembolytic therapy is contraindicated in severe uncontrolled hypertension due to the risk of cerebral hemorrhage.

6

Pericardiocentesis – After pericardiocentesis, a rise in BP and a drop in CVP are expected. Client expresses immediate relief, heart sounds are clearly audible. Erb’s Point – Third Intercostal space, just left to the sternum Mitral area is in the fifth intercostals space, midclavicular area. Aortic area is located in the second intercostal space, just right to the sternum. Pulmonic area is located in the second intercostals space, just left to the sternum.

EAR Myringotomy – A surgical procedure that allows fluid to drain from the middle ear. Meniere’s Disease – Can cause severe vertigo in clients. Nursing Dx. Should focus on safety and injury prevention. It is a disturbance of the fluid in the endolymphatic system. Cause is unknown. Acute attack of Meniere’s disease – Primary complain will be tinnitus. After Mastoidectomy, nurse should inspect client for drainage or bleeding. Assess for cranial nerve injury (CN VII). Also pain, dizziness, nausea, HOB elevated 30 degrees, client should lie on unaffected side. After ear surgery clients should be instructed to avoid drinking with a straw for 2-3 weeks; avoid straining and takes stool softeners as prescribed, avoid getting head wet, or washing hair for at least a week and avoid moving the head rapidly, bouncing and bending fro at least 3 weeks. Also avoid air travel. Meniere’s Disease – Diet should be low in sodium, to decreased the endolymphatic fluid, should also restrict fluids, Prebyscusis – Type of hearing loss that occurs with aging. It is gradual and caused by severe nerve degeneration in the inner ear or auditory nerve. Sensorineural loss. Perforated eardrum – A round or oval darkened area will be seen Acute purulent Otitis media – Red bulging eardrum. Repeated ear infections – Dense, white patches on the eardrum. Yeast or fungal infection – Colony of black dots in the eardrum. Caloric Test – Cranial nerve VIII. Cold water will cause rotary nystagmus away from the ear being irrigated. Warm water will cause rotary nystagmus towards the side of the ear being irrigated. No nystagmus with warm water indicates disease of the labyrinth. Caloric Test is contraindicated when clients have or are suspected to have acute disease of the labyrinth, perforated tympanic membrane. To rule out, an otoscopic examination should be performed, if there were cerumen, it should be removed before the test. Motion sickness medications should be taken at least 1 hour prior to the triggering event. Miotic medications for glaucoma – Lower intraocular pressure, increase blood flow to the retina and decrease retinal damage and loss of vision.

ENDOCRINE Addison’s Crisis – Sudden profound weakness, severe abdominal, back, and leg pain, hyperreflexia, followed by hypothermia, peripheral vascular collapse, coma and renal shutdown.

7

Pheocromocytoma – Hypertension is a major symptom, as well as glycosuria, weight loss and diaphoresis. Hyperglycemia and glucosuria also occur in attacks. NPH – Insulin onset at 3-4 hours, peak at 6-12 hours, duration of 18-28 hours. Hyperglcemia – Polydipsia, polyuria and polyphagia Glycosylated hemoglobin (Hgb A1C) in a compliant client will be 7.5% or less. Adrenal insufficiency symptoms – weakness, hypotension, fever, and mental status changes. Thyroid Storm – Fever >100 F and tachycardia, flushing, sweating, marked agitation and restlessness. Delirium and coma can occur. Usually seen in clients with Grave’s Disease with the symptoms precipitated by a major stressor (trauma, sepsis, surgery, delivery) Pituitary Gland hormones – Posterior – ADH, Oxytocin Anterior – GH, FSH and LH Adrenal Medulla – Epinephrine and Norepinephrine Adrenal Cortex – Cortisol, aldosterone and the androgens Hypothalamus secretes ADH and stores it in the pituitary Grave’s Disease – Hyperthyroidism. Rest! DDAVP or Stimate – Synthetic form of ADH. It causes increased reabsorption of water with a resultant decrease in urine output. Also a decrease in serum osmolality, because more fluid is retained, and an increase in urine osmolality, because less fluid is excreted. Increased BP is a side effect, rather than a therapeutic effect of DDAVP or Stimate. Hyperparathyroidism – hypercalcemia is the hallmark. Eelvated serum calcium levels produce osmotic diuresis and thus, polyuria, which in turn leads to dehydration (weight loss). Common signs and symptoms: Nausea, vomiting, anorexia and constipation. Catecholamine testing – Normal up to 14mg/dl. Anything higher suggests pheochromocytoma. Hypertensive crisis treatment – Phentolamine Mesylate (Regitine). Short-acting alpha adrenergic blocker, given IV bolus or drip. Phenoxybenzamine HCl (Dibenzyline) is an oral, long acting alpha-adrenergic blocker, used preoperatively in the management of hypertension and prevention of hypertensive crisis. After alpha-adrenergics, low doses of Propanolol (Inderal) may be used to treat tachycardia or dysrrhythmias. Pheochromocytoma Diet – High in vitamins, minerals, and calories. Food that contain caffeine should be avoided, since they can precipitate hypertensive crisis. (Graham cracker and warm milk). Addison’s Disease Diet – High sodium, high complex carbohydrate, and high protein. To prevent excess fluid and sodium loss, the client is instructed to maintain and adequate salt intake daily, and to increase salt intake during hot weather, before strenuous exercise, and in response to fever, vomiting and diarrhea. Addison’s Disease signs and symptoms – hypotension from fluid loss, syncope, muscle weakness, anorexia, nausea, and vomiting, abdominal cramps, weight loss, depression and irritability.

8

EYE Errors of Refraction – Astigmatism Myopia Presbyopia Hyperopia Keratoplasty – Surgery for cataracts Cataracts – Bright light complaints Glaucoma – Difficulty seeing things out of the side of the eyes Macular Degeneration – Blurred vision Normal Intraocular pressure – 12 to 22mm Hg Mydriatic meds – (Such as Atropine) Dilates the pupil and causes increased intraocular pressure in the eye. Contraindicated in client with glaucoma. Miotic agents – (Such as Pilocarpine HCL) Increases blood flow to the retina and lowers intraocular pressure. Pilocarpine systemic reaction/toxicity – Vertigo, bradycardia, tremors, hypotension sycope, cardiac dysrrhythmias and seizures. Antedote is Atropine Sulfate Betablockers – Monitor for weight changes, hypo/hypertension. Latanoprost – used to lower intraocular pressure in clients with open-angle glaucoma. SE: Brown hyperpigmentation of the iris. Confrontation test – Measures peripheral vision. It compares the client and nurse’s peripheral vision, assuming that the nurse’s vision is normal. Aphakia – Absence of the lens of the eye and is corrected by prescriptive glasses, contact lenses, or intraocular lenses. Keratoplasty – Question med that dilate the pupil. Sutures are left in for about 6 months. An eye shield should be worn for the 2 months following surgery. Enucleacion – Chalazion – A cyst that result from blockage of a sebaceous material in a meibomian gland. Apply warm compresses to the affected eye 3-4 times a day. Condition is not contagious. Hordeolum – Timolol – Betablockers decreases IOP by decreasing the production of aqueous humor. Pilocarpine – Miotic agents decrease IOP by increasing contractions of the ciliary muscle and constrict the pupil thereby, increasing the outflow of aqueous humor. Arcus Senillis – Age-related change, formation of a yellow-grey ring around the periphery of the cornea surrounding the iris. Corneal Reflex is the blink reflex

9

Ptosis – Sagging of the upper eyelid. Lacrimal apparatus located in upper eyelid. To visualize, ask patient to look down as you retract the upper eyelid and observe the outer canthus of the eyelid. Cardinal Fields of gaze – III, IV and VI Corneal reflex tests functioning of the Cranial Nerve V – Trigeminal Peripheral and central vision are tested by the visual acuity test. Cataracts – Lens are opaque, causing blurry vision. Angle closure glaucoma – Base Optic nerve – Transmit images to brain for interpretation. Ocular muscle control – Governs eye movements. Binocular vision – Ability of both eyes to fuse two images into one Depth Perception – Ability to see images in 3 dimensions instead of 2. Ciliary body produces Aqueous humor. Then goes to posterior – anterior chamber Optic Disc – Blind spot of the eye. Cones – Detail and color vision. Rods – Responsible for perception in dim light. Night blindness due to dysfunction. Hyphema – Blood present in the anterior chamber of the eye. Treatment: Bedrest, semi-Fowler’s position. DIGESTIVE 4-6 weeks after colostomy formation – Diet should be low residue, then progress it to high carbohydrate, high protein. Levine tube – Single lumen Salem sump tube – Large lumen and an air vent Sengstaken-Blakemore tube – To control bleeding in the esophagus Dobbhoff Tube – Used for feedings Stomach decompression settings – Low and intermittent suction. Bentyl – Anticholinergic, antispasmodic, used to treat Irritable Bowel syndrome that in unresponsive to diet therapy. Should be taken 30 minutes prior to a meal. Prolapsed stoma – Bowel protrudes and causes an elongated and swollen appearance of the stoma. Retracted stoma – Sinking of the stoma.

10

Ischemia of the stoma – Dusky and bluish color. A stoma with a narrow opening is described as being stenosed. Peptic ulcer disease – Should avoid in diet foods such as highly spiced foods, alcohol, caffeine, chocolate, and fresh fruits. Dumping syndrome – Weakness, dizziness, diaphoresis, flushing, hypotension, abdominal pain and distention, hyperactive bowel sounds and diarrhea. To prevent this, eat in a Low Fowler’s position and lie down for at least 30 minutes after eating. Small frequent meals are best (avoid liquids with meals). Also avoiding high carbohydrates with meals will decrease the risk of developing this. Low residue (Low fiber) diet – For clients with ulcerative colitis, diverticulitis, and irritable bowel syndrome. Corn is an example of a high residue food that should be avoided. Laennec’s Cirrhosis – Results from long-terms alcohol abuse Cardiac Cirrhosis – Caused by long-term right sided congestive heart failure. Postnecrotic Cirrhosis – Caused by hepatotoxins, chemicals, infections or a metabolic disorder. Biliary Cirrhosis – Results from a decrease in bile flow or long-term obstruction of the bile ducts. Portal hypertension signs and symptoms – Jugular vein distention, rales, and decreased perfusion to all organs Ascites – Place client in Semi-Fowler’s position. Asterixis – Flapping tremor of the hand, early sign of encephalopathy. Tremors and drowsiness would also be noted. Neomycin – Prescribed orally to treat portal systemic encephalopathy. Destroys normal bacteria found in the bowel, thereby, decreasing protein breakdown and ammonia production. HHNK – Hyperglycemia results from increased production and reduced utilization of glucose. Tacrolimus (Prograf) – Immunosuppressant medication used in the prophylaxis of organ rejection. SE: Headache, tremors, insomnia, paresthesias, diarrhea, nausea, constipation, vomiting, abdominal pain and hypertension. Pancreatitis – Pain unrelieved by vomiting. Abdominal pain radiating to the back is also common. Normal serum amylase – 53 to 123 Somogyi U/dL Normal Platelet count – 150,000 to 400,000. Normal serum protein level – 6.0g/dL to 8.0 g/dL Colostomy irrigation procedure– Use 500 to 1000mL of warm tap water, solution is suspended 18 inches from the stoma, cone is inserted 2-4 inches into the stoma and should never be forced. If cramping occurs, client should decrease the flow rate of the irrigant as needed by closing the irrigation clamp. Pancreatic Juice is rich in bicarbonate, which helps neutralize the gastric acid in food entering the small intestine from the stomach. Appendix is attached to the ceccum.

11

Principle reflex for defecation is located in the parasympathetic center, which in turn is located in the 2nd to 4th sacral level of the spinal cord. Complete proteins derive from animal resources, such as meat, cheese, milk and eggs. Incomplete proteins can be found in fruits, nuts, vegetables, cereals, breads and legumes. Albumin is responsible fro maintaining the osmolality of the blood. When there is a low albumin level, there is decreased osmotic pressure, which in turn can lead to peripheral edema. The total protein level may also decrease when albumin levels are low. Proteins are broken into ammonia, then to urea leading to high serum ammonia levels when the liver has difficulty with protein metabolism. OB - Reassuring: accelerations of FHR, no variable decels, and the presence of short term variability - Variable decels = cord compression - Stadol – opiod analgesic - Mag sulfate: therapeutic index is 4-8 mg - Absence of DTRs indicates mag sulfate toxicity (also causes altered conciousness b/c is sedative, cardiac arrest/respiratory paralysis) - ANTIDOTE for mag sulfate is calcium gluconate - Abruption placenta: painful vag. Bleeding, abd pain, back pain - Mag sulfate produces flushing/sweating b/c of decreased peripheral BP, it decreases CNS activity, acting as an anticonvulsant, and it decreases the frequency and duration of uterine ctx - If slowing of FHR and loss of variability, nurse should turn pt on L side and give O2 at 8 – 10 L/min - Severe PIH: complete bed rest, quiet environment, L sidelying. - signs of complications: ctx lasting 90 secs. Or longer, 2 min. apart or less, bradycardia, tachycardia, persistently decreased variability, irregular HR. - if cord prolapse, wrap sterile saline towel around cord, put in sidelying or elevate hips (Trendelenberg) or hands/knees position, - main se of regional block: hypotension - To PREVENT decels, sidelying and upright positions are best (walking, standing, sitting) - the ductus venosus connects the umbilical vein to the inferior vena cava - 2nd stage of labor begins when cervix is dilated completely - late decels are due to uteroplacental insufficiency r/t decreased bld and o2 flow to baby. Tx: give 02 - nrml WBC ct during birth is 11,000-15,000, up to 18,000 - may be up to 30,000 AFTER delivery - if pt has to be supine, place wedge under R hip to prevent vena cava syndrome

12

- Episodic accelerations are a sign of fetal wellbeing and O2 reserve - station is determined by the relationship of the presenting part to the ischial spine - an anemic patient may be at risk for pp infection - amniotomy = AROM (pt will have increased efficacy of contractions) - EARLY DECELS = fetal head compression (normal) - VARIABLE DECELS = umbilical cord compression and decreased bld flow - LATE DECELS = ominous – uteroplacental insufficiency - in pushing stage, assess FHR q 15 min. - signs of fetal compromise: persistent nonreassuring FHR, fetal acidosis, passage of meconium - Tx for hypotonic uterine is pitocin and AROM - primary intervention in hypertonic labor is pain management - in multiple delivery, each baby’s heart needs to be monitored - if vaginal bleeding in 3rd trimester, no pelvic exams until placenta previa is ruled out - abruptio placenta: deliver baby ASAP and control hemorrhage - forceps delivery puts mom at risk for pp uterine rupture - APGAR: score of 8-10 indicates doing well, 5-7= resuscitative interventions, less than 5 means vigorous resuscitation. - mag sulfate is an anticonvulsant, not an antihypertensive, so main goal is to prevent seizures, not lower BP - If hypertonicity r/t pitocin too fast occurs, intervene in this order: stop pitocin, place in side lying position, administer O2 at 8-10 L, vag exam to check for prolapsed cord, check BP and HR. - maternal temp may go up to 100.2 in the postpartum period r/t dehydrating effects of labor and is still considered normal. Tx: increase oral fluids - orthostatic hypotn/dizziness is nrml up to 8 hours pp. *safety measures - normal newborns resps are 30-60/min - if getting postpartum rubella vaccine, don’t get pregnant for 2-3 months. - perineal pads should be weighed before and after use to gauge the amount of lochia - primary causes of subinvolution are infections and retained placental fragments - methergine may be given pp to help prevent hemorrhage by causing the uterus to contract - betamethasone may be given in preterm labor to promote fetal lung maturity by increasing surfactant - pp patients may normally experience a shaking chill (cause unknown). Not of clinical significance unless accompanied by a fever

13

- nrml NEWBORN HR is 100-170, it’s 120-160 during the pregnancy - cephalohematoma does not cross the suture line, caput succendum does - omphalocele: herniation of the gut into the umbilical cord (spina bifida in the front) - stomach capacity for a 1 month old is 90-150 ml - congenital diaphragmatic hernia causes bowel sounds heard over chest. - hiatal hernia causes coughing, wheezing, and short periods of apnea - normal newborn calcium level should be 7-12 mg - baby born to Hep B mom will need Hep B immune globulin and vaccine within 12 HOURS of birth - HIV mom: no breastfeeding - cephalohematoma puts the baby at risk for jaundice - a neg. direct Coomb’s test (performed on baby) means that no maternal antibodies were present on fetal RBC’s (good sign) - Classic Rh incompatibility situation: Rh neg. mom with a Rh pos. baby - if UTI, don’t drink carbonated drinks, they’ll alkalinize the urine. Instead drink cranberry, prune, and apricot juices to acidify the urine. - a mom with Hep. B virus should not breastfeed, should wear gloves when bottlefeeding, and should practice good handwashing. - in the immediate pp period, mom vitals should be checked q 15 min during the 1st hour and q 30 min. for the next 2 hours. - empty bladder before fundal assessment. Lie on back with knees flexed. - notify the physician if clots in the lochia are bigger than 1 cm - average lochia flow requires 6 peripads/day. More than 8 is considered danger. - normal bowel function returns 3 days pp - how to know if a pp woman with an epidural developed a vulvar hematoma: changes in vital signs r/t hypovolemia, main one is increase in pulse. (ice pack to tx hematoma). Serious hematomas require surgery to stop the bleeding if pt is becoming hypovolemic. These surgery pts will need to be on antibiotics postop to prevent infection in the wound - BP decreases normally after delivery as blood vol. slowly returns back to normal - for DVT pt, elevate legs to enhance venous return, give analgesics, promote rest, wear support stockings, warm compress. Don’t ambulate - if pulm. Embolism suspected, give O2 at 8-10 L. - mastitis pt: DO need to continue to breastfeed; empyting affected breast is important to prevent abcess. Will need abx tx, can use analgesics, ice packs, supportive bra.

14

- saturation of more than 1 pad/hour even in immediate pp is danger. - for DVT prevention, ambulate frequently. But to TREAT DVT, don’t ambulate. - if a pp DVT pt is on Heparin, the PTT- protamine sulfate will be monitored (PT for coumadin- vit K) - tachypnea and retractions are the 2 most common sx of RDS - can breastfeed as normal with hyperbilirubinemia - surfactant replacement tx has to be initiated thru an endotracheal tube into the lungs - bilirubin level - skin integrity and F/E status are important for the newborn having phototx - FAS babies may have facial abnormalities, IUGR, heart probs, abnrml palmar creases, and resp. distress. Primary intervention: establish nutrition/feeding and wt. gain patterns - asepsis most important focus for HIV mom - when an rh neg. mom gets rhogam, it prevents her blood from developing antibodies against rh pos. blood in the case that some of her possible rh pos. baby’s blood may have leaked into her circulation. This will protect her next baby from being affected by Rh incompatibility. - se of methergine is increased BP because causes vasoconstriction secondary to muscle contraction, so don’t use if fam. Hx of peripheral vascular dz - if using antiembolism stockings, should apply them before rising in the morning to prevent the venous congestions that happens on arising - BG is most important blood component to monitor in post term SGA baby - on prenatal visit, if BG is greater than normal, will follow up with glucose challenge. - for varicose vein prevention, don’t wear any kind of knee high constriction. Wear support hose or pantyhose - to do kick counts: sit or lie on side with hand on largest part of belly. *notify MD if less than 10 kicks in 12 hours. - heart begins beating @ week 5 - nonstress test: ultrasound records FHR activity while toco records fetal activity (movement). Fetus is not stressed by inducing a contraction. Records hr and movement - A reactive nonstress test = normal. Baseline HR should be 120-160, should be long term variability, 2 or more accelerations of atleast 15 bpm occur and last for 15 sec. All within 20 min. - if nonreactive, may do ctx stress test. A neg. stress test = normal. NEG. FOR late decels r/t ctx. - severe PIH has increased bleeding risks - L side lying promotes bld return to heart

15

- for a chronic DB mom: moderate exercise, stay on standard DB diet, BG monitoring. - *insulin needs INCREASE in the second half of pregnancy - risk of resp. dep. With mag sulfate - if HTN, have risk of abruption placenta - if chronic cardiac dz, risk of PTL - b/t 14-20 weeks, pulse will increase by 10-15 bpm. - BP will decrease in the first ½ of pregnancy - in monitoring a pt. with PIH, fetal movement/wellbeing is most important - Hegar’s sign: softening of cervix when 1st pregnant. (pos. sign) - Goodell’s sign: lengthening/thinning - umb. Cord has 2 arteries, 1 vein. BLOOD PUMPED BY THE EMBRYO’S HEART LEAVES THE EMBRYO THRU 2 UMBILICAL ARTERIES. ONCE OXYGENATED, THE BLOOD IS RETURNED BY 1 UMBILICAL VEIN. SO ARTERIES CARRY DE-O2’D BLOD AND WASTE AWAY FROM THE FETUS, AND VEIN CARRIES O2’D BLOOD, NUTRIENTS, AND O2 TO THE FETUS. - sex can be determined as early as 12 wks - you can hear FHT with a regular stethoscope at 18-20 wks, but with a Doppler at 10-12 wks - quickening is first felt @ 14-16 wks - during 1st trimester, FHR should be 160-170, and when it’s nearer to term, should slow down to 120-160. - but at any point in pregnancy, if it is below 120 or above 160/170, trouble. TPN:

• TPN, the greatest risk of TPN is hyperglycemia • Effects of TPN = weight gain. • When initiating TPN, you gradually increase the dose to avoid hyperglycemia. • Do not add any medication to TPN, 5% dextrose in water or dextrose should be kept at the

patient bed corner should TPN finish. D10 W at the same rate to avoid hypoglycemia. = Severe Hypoglycemia gives Glucagon.

• An initial of action a nurse in a DM patient is do assessment by assessing the finger stick. • The initial action of the nurse when given oxygen to an unconscious patient = have a patent

airway. Therefore before you give oxygen, you must clear the airway of mucous or have a patent air.

Breast-feeding:

• Biology level explains the reason why breast-feeding mothers feel abdominal cramps when breastfeeding.

• The reason is the physiology release of Prolactin and Oxytocin group by the pituitary gland as soon as the baby is put to the breast.

• The sucking reflexes help to release prolactin, helps contract the Lactiferous Ducts. • This leads to increase uterine contraction; mother refers to it as abdominal pain. Ensuring

action, tell them it is normal. Importance of early ambulation:

16

• To prevent auto-static hypotension or postural hypotension, advises patient who stayed too long in bed not to get up suddenly.

• Advice them to get up slowly or gradually in bed, sit up for a while at the edge of the bed. • Then dangle their feet for a while before they start ambulation. • Biology informs us that early ambulation helps to prevent DVT in postoperative patients. • Biology levels inform us that fracture of long bone and hip bone fractures are prone to fat and

air embolism. Fluid Drainage:

• When you’re doing postural drainage, is by posture and gravitation pulls. • Insertion of the enema tube = 2in-3in long. • The height of enema tube = 12-18 in. This is to allow less pressure into the GI, the flow is by

gravity. • During enema procedure, after giving 300 cc of fluid, pt complains of cramps, stop or pinch

for a while and ask pt to take deep breath. • G. Tube feeding by bolus = by gravity. • Jackson Pratt drain = gravity. Compress center to exert pressure.

Mathematical Applications:

• This involves calculations of drugs and IV lines. E.G. • 1 Kg = 2.2 lb. • 1 lb = 454 grams. • 1 grain = 60 milligrams. • 1 gram = 1000 milligrams. • 1 oz = 30 cc. • 1 mL = 1 cc. • 1 cup = 240 cc. • ½ cup = 120 cc. • 1 teaspoon = 5 cc. • 3 teaspoon = 1 tablespoon. • 1 tablespoon = 15 cc.

Culture Nursing:

• Respect people’s culture. • Culture influences the way people adapt to illness and health. • Culture tells us how different people react to skin illness. Examples: Hebrews bear pain more. • The Indians do not look directly at people’s faces. • African culture encourages women to bear labor pains. • Some cultures forbid certain foods hence are prone to anemia. • Asians eat more cereal and rice. • Africans eat roots, fibers, over cook and fried food.

Religion: • Please know your religion, especially = Adventists, Muslims, Jews, Jehovah’s Witnesses,

Mormons, Roman Catholics & Hindu, etc. General Information:

• This is a very important aspect of nursing practice; it influences patient perception to illness. • The nurse’s belief also influences their attitude to the care of the patient. • Respect other people’s religion. • Some religions accept blood transfusions while some refuse. • The Jehovah’s Witnesses do not accept blood transfusion. • The Roman Catholics call priests for baptisms and last sacrament if one is about to die. • No artificial contraceptives/no abortion = total abstinence. • Roman Catholics believe in natural birth control only, no other form of family planning. • Muslims/Jews do not eat pork.

17

• Muslim: no embalmment, autopsy, buries immediately after death. • Some religions forbid organ donation and transplant.

ADVENTIST(Seventh-day Adventist, church of God, Christian church):

Birth: • Opposed to infant baptism • Baptism in adulthood • Some believe in divine healing and practice anointing with oil and use of prayer. • Saturday = biblical day of worship. Food: • Meat is prohibited in some groups. • Some are vegetarians-encourage 2nd class protein, i.e. broccoli, beans, spinach. • No alcohol, coffee, tea. Death: • Believe the dead are asleep until the return of Jesus Christ at which time final rewards and

punishments will be given. Organ Donation/Transplantation: • Individual/families have the right to receive or donate those organs that will restore any of the

senses that will prolong life. Medical: • May desire communion or baptism when ill. • Believe in man’s choice and God’s sovereignty. • Some oppose hypnosis as therapy. • Give plant protein-beans legume. • Worship on Sunday.

Baptist: Birth:

• Opposed to infant baptism. • Believers are baptized by immersion as adults Death: • Clergy seeks to minister by counsel and prayer with patient and family Organ donation/transplantation: • Both organ donation and transplantation are generally approved of when they do not seriously

endanger donor and when they offer medical hope for recipient. • A transplant must offer possibility of physical improvement and extension of human life. Food: • Some groups discourage coffee, tea, alcohol Medical: • “Laying of hands” some may encounter some resistance to some therapies, such as abortion • Believe God functions through physician • Some believe in predestination and may respond passively to care. • Fundamentalists and conservative groups accept Bible as inspired word of God.

Buddhist churches of America

Birth: • No infant baptism • Infant presentation Death: • “Last rite” chanting is often practiced at bedside soon after death. The deceased’s Buddhist

priest should be contacted, or family should contact him. Organ Donation/transplantation: • Believe that organ donation is a matter of individual conscience • There is no written resolution on the issue of organ donation/transplantation. Food:

18

• No requirements or restrictions • Discourage use of alcohol and drugs.

THE 25 MOST IMPORTANT MUST-KNOW DRUGS FOR NCLEX ANALGESICS:

(1) Aspirin:Do not give together with other anticoagulants. Stop taking Aspirin some days before surgery. Do not give to children with viral infection(Reye syndrome)

(2) NSAID’s e.g. Ibuprofen—Take with food; contraindicated for people with GI ulcers (3) Morphine: A respiratory depressant. It should be withheld if the respirations are below 10

ANTI-CONVULSANTS: (4) Dilantin: Causes gum hyperplasia. Advice client to visit dentist frequently

ANTI-INFLAMMATORY

(5) Predisone: Causes Cushing like symptoms. Common side effects are immunosupression(monitor client for infection), hyperglycemia

ANTI-COAGULANTS

(6) Heparin: Monitor pt’s lab work-PTT. Antidote is protamine sulfate (7) Coumadin: Monitor pt’s lab work—PT. Antidote is Vitamin K

ANTI-PARKINSONIAN (8 )Cogentin: Used to treat EPS (9)Sinemet: Drug is effective when tremors are not observed RESPIRATORY (10)Theophylline/Aminophylline: Side effects--Tachycardia CARDIOVASCULAR (11)Digoxin (Lanoxin): Signs of toxicity: Pt will complaint of visual change in colors. They would also complain of loss of appetite. ANTIHYPERTENSIVE (PRE-ECLAMPSIA) (12)Magnesium Sulfate: Monitor for deep tendon reflex and respiratory depression DIURETICS (13)Hydrochlothiazide: Monitor potassium levels (14)Lasix: Monitor potassium levels (15)Aldactone: Potassium sparing PSYCHOTROPICS (16) Lithium Carbonate: Know therapeutic range (0.8 to 1.2mEq). Also know symptoms of toxicity. Adequate fluid and salt intake is important. (17) MAOI inhibitors: Have dangerous food-drug interactions. Food with Tyramine should be avoided. For example: aged cheese, wine etc. (18) Disulfiram (Antabuse): Used for alcohol aversion therapy. Clients started on Disulfiram must avoid any form of alcohol or they would develop a severe reaction. Teach pt to avoid some over-the-counter cough preparations, mouthwash etc. MATERNITY (18) Oxytocin: Assess uterus frequently for tetanic contraction. ANTIDOTES (19) Narcan: Reverses the effects of narcotics (20)Calcium Gluconate: Antidote for magnesium sulfate (21)Vitamin K: Antidote for Coumadin

19

Questions have been asked on NCLEX recently about the following drugs: (22Tegretol: side effects. (23)Atropine: What checks do you do before giving this drug (BP) (24)Epogen: Used in treating anemia because it increases RBC production. (25) Acyclovir: anti-viral medication used in treating shingles. EXTRA NOTES: (A) When a client is on antibiotics, teach the client to continue taking the medication even though they feel

better (B) Monitor client taking antibiotics such as Vancomycin for ototoxicity. Pt will complain of tinnitus,

room spinning (vertigo) and nausea. (C) Clients taking vasodilators e.g. Verapramil would complain of headache Delegation rules in the U.S. A registered nurse may not delegate: * Initial nursing assessment or advanced nursing assessment. * Nursing diagnosis determination * Development of nursing care plans * Evaluation of the patient regarding the nursing care plan *Establishment of nursing care goals *Patient care activities that require professional nursing knowledge, judgement and skills. A registered nurse MAY delegate: * Feeding a client * Taking vital signs * Hygiene Care

The 5 delegation rights: 1. Right task 2. Right Circumstance 3. Right Person 4. Right Direction and Communication. 5. Right Supervision

Prioritization: Typical prioritization question look like the following: 1. What is the most important? 2. What is the initial action of the nurse? 3. What is the best nursing action? 4. Which client would the nurse care for first? Here are some tools that will help answer these types of questions: 1. MASLOW’S HIERARCHY OF NEEDS: * Physiological Needs (survival) * Safety Needs (Physical and psychologicval) * Psychologicla Needs (Care and Belonging) * Self Actualization Next is their Nursing Process APIE Assessment Plan Implement Evaluate

Next back to the old ABC’s: Airway Breathing Circuation

When dealing with fire use RACE: REMOVE the client Sound the ALARM CALL the fire department EXTINGUISH the fire. ADULT PHYSIOLOGICAL INTEGRITY

20

Comprizes 46-54% of the test. They lay it out according to system. Remember 1. Maintaining the cts airway is always #1. 2. There is always something you could do before calling the doctor. THE NERVOUS SYSTEM. The nervous system is comprised of the CNS, PNS &ANS. CENTRAL NERVOUS SYSTEM – Brain and spinal cord. PERIPHERAL NERVOUS SYSTEM – Cranial and spinal nerves. AUTONOMIC NERVOUS SYSTEM – Controls “automatic” function of the body like breathin and our heartbeat. It also maintains a stable internal environment. The A.N.S. branches in the SYMPATHEIC and the PARASYMPATHETIC Nervous Systems SYMPATHETIC PARASYMPATHETIC “Fight or Flight” Maintains normal body functioning Increases respiratory rate Normalizes heart rate and blood pressure Decreases Peristalsis Increases peristalsis Secretes Epinephrine and Nor epinephrine Secretes Acetylcholine Dilates pulmonary bronchi Constricts pulmonary bronchioles NEUROTRANSMITTERS (acetylcholine, serotonin, epinephrine, norepinephrine and dopamine) transfer information from one neuron to another across a synapse. AFFERENT IMPULSE (To CNS). EFFERENT IMPULSES (from CNS). The BRAIN Frontal Lobe • Personality, behaviour

• MOTOR Function • BROCA’S AREA (Aids formation of

words) • Concentration, abstract thought, memory

Temporal Lobe • Hearing, taste , smell • WERNICKE’S AREA (Interpretation of

language) • Interpretive Area- Junction of temporal,

parietal, and occipital lobes (Integration of somatic, auditory, and visual associations occur here).

Parietal Lobe • Sensation – Determination of size, shape, weight, and texture of sensory input.

• Orientation of space and space perception (propriception)

Occipital Lobe * Vision (reception and interception). BRAIN STEM

• Nerve pathways connecting the brain and the spinal cord • Cardiac, vasomotor and respiratory centres

DIENCEPHALON= THALAMUS AND HYPOTHALAMUS, located between the brain stem and the cerebrum. THALAMUS = Interpretation of SENSATION (Pain, temperature and touch).

21

HYPOTHALAMUS = Temperature control, water metabolism, control of hormonal secretion, heart rate, peristalsis, appetite control, thirst centre, sleep-wake cycle. THE 12 CRANIAL NERVES I Olfactory Smell II Optic Vision III Oculomotor Eye movement IV Trochlear Eye Movement V Trigeminal Chewing, Facial sensation VI Abducens Eye movement VII Facial Taste, facial movement VIII Vestibulocochlear Hearing, BALANCE IX Glossopharyngeal Taste (Posterior Tongue),

Swallowing X Vagus Pharynx,Respiratory, cardiac and

circulatory reflexes XI Spinal Accessory Shoulders, head movement XII Hypoglossal Tongue movement The American Exam will require any more knowledge than the above about the cranial nerves. A good one would be III, IV and VI all control eye movement. NEUROLOGICAL ASSESSMENT What is the first thing a nurse should assess to determine the presence of neurological changes? (LOC) THE GLASGOW COMA SCALE EYE OPENING + BEST MOTOR RESPONSE+ BEST VERBAL RESPONSE = SCORE 3 is worst score, 15 is best. DECORTICATE = Arms turned in and up, toward CORTEX. DECEREBRATE = Extension, limbs away from body. CEREBRAL DYSFUNCTION 1. GNOSIA – Inability to recognize common objects 2. APRAXIA – Inability to perform a skilled motor task, assuming the ct is not paralyzed. 3. APHASIA - The inability to communicate. (EXPRESSIVE – speak, RECEPTIVE – Understand speech). BLOOD LABORATORY VALUES: Hbg=12-18g/dl Platelets=130-400 thos/mcl MCH=27-33pg Hct – 37-54% (↑=dehydrat) RBC=3.9-5.6 mill/mcl WBC=3.8-10.8thos/mcl PT=10-14sec aPTT=20-40 sec Neut=48-73% (baby wbcs) Lymph+18-48%(↑=viral) Monocytes=0-9%(↑=Leuk) Eosinophils=0-5%↑=Allerg Na=135-146 mEq/L K=3.5-5.5mEq/L Cl=95-112 mEq/L Ca=8.5-10.3 mEq/dl Po=2.5-4.5 mEq/dl AST=0-42U/L ALT=0-48 U/L ALP=20-125 U/L Bilirubin=0-1.3mg/dl

22

BUN=7-25mg/dl Creat=0.7-1.4 mg/dl Uric Acid=2.5-7.5mg/dl Spec grav=1.015-1.030 Osmolal=600-1400 Glucose=70-115mg/dl Chol=120-240mg/dl LDL=62-130mg/dl HDL=35-135mg/dl Triglycerides=0-200mg/dl HgA1c=5-7.5% Protein=6.0-8.5 g/dl Albumin=3.2-5.0 g/dl ESR=<20mm/hr ABG’s pH 7.35-7.45 pCO2 35-45 pO2 80-100 HCO3 21-27mEq/L O2 Sat 95-99% BIRTH - Gestational age assessments must be done at birth and repeated at 24 hours in order to be accurate. - Infants are expected to grow approximately one inch every month for the first 6 months - Milia are exposed sebaceous glands, usually on the nose - may be associated with physiologic jaundice, because extra red blood cells are destroyed within the cephalhematoma - The correct answer is molding which is caused by overriding of the cranial bones during labor and birth. - - Caput succedaneum is a localized soft edematous area of the scalp and overrides suture lines. - The nurse should be able to auscultate bowel sounds after one hour. It is too late for the initial assessment to auscultate bowel sounds anytime after two hours - The correct answer is Erb-Duchenne paralysis (Erb's Palsy) which results from trauma to the brachial plexus during a difficult birth - The normal range for heart rate is 110-150 beats per minute for a quiet, healthy full term newborn. 150 beats per minute is too high for a newborn during sleep. During sleep it is normal for the heart rate to drop to 85 beats per minute. Crying can also increase heart rate, 180 is normal for a crying newborn. - Normal HCT range is 43% to 63%, 40% is too low and should be reported to the physician. Normal range for Sodium is 126-166mEq/L. Normal range for potssium is 5.6-12.0mEq/L.. - Forceps may cause facial bruising or cephalohematoma which can increase the amount of bilirubin for the liver to handle - Encouraging glucose feedings helps to increase calories that are needed to form hepatic binding proteins - The usual skin sensor placement site is on the newborn's abdomen, preferably over the liver and not over ribs - A newborn with facial grimacing and mucus may be in distress and is the correct answer - The correct way to stimulate respirations in a baby is to vigorously stroke the spine - Voiding should occur within 24 hours and the first stool can occur up to 48 hours after birth - It is correct to assess every 30 minutes for at least two hours for abnormal bleeding (circ) - The correct answer is that newborn C with pathological jaundice would benefit most from breastfeeding - The correct answer is to check the bloodglucose level every hour until stable (for IDB) - The correct answer is to perform glucose testing no later than two hours of age

23

Section ONE 1. A client who received Extracorpeoreal Wave Lithotripsy will expect: a. Pain at site - correct b. Hematoma at site After ESWL, most patients have transient hematuria and intermittent colic as fragments pass. Proper hydration and analgesia are essential during the immediate postoperative period. Hematoma at site is a sign of complication. 2. A client is experiencing menstrual cramps. What will you advice? a. Increase potassium in the diet a week before menstruation b. Increase protein the diet - correct. c. Eat carbohydrates liberally d. Decrease carbohydrates The question sounds confusing, I mean the choice of answers. Ca and Mg play important role in cramps, but no word about them... Low sugar is a contributing factor for cramps, but what means "liberally"? 3. A patient has an active PTB. What will the nurse advice? a. Close the bedroom windows at night - room with negative pressure, how the windows could be open or closed? b. Wash utensils in warm water - doesn't address the question. c. Use surgical mask when talking to friends - not effective d. Cover mouth with tissue when coughing or sneezing - correct 4. The patients were admitted in the ER after a plane crash. Who will the nurse attend to first? a. A 5-year-old boy with soot on the chest of t-shirt and singed eyebrows - stable. No evidence for neck wounds or singed chest. b. Infant with second degree burns on both thighs - correct. Unstable client. 5. What will the nurse teach a patient who will undergo CABG? a. I will start coughing and deep breathing after CABG when I'm in PACU - yes, correct action to prevent atelectasis. b. The nurse will wake up hourly to take my BP - not needed that often c. I will have the chest tube after CABG - not needed 6. In post inguinal hernia repair, what will you tell the patient? a. You will have a pink-tinged urine until 3 days post-op - no way b. You will expect swelling on the scrotal area until 2 days post-op - true. Swelling usually subsides in 48 hours post-op c. Other options

24

7. A charge nurse is making assignment for a HIV (+) patient. Who will be assigned to this patient? a. It's okay for the Nursing Aide with colds to take of this patient - no, the pt is immunosuppressed. b. Nursing Aide with psoriasis can put on gloves and give care to the patient - the same reason as above. c. Pregnant Nursing Aide can take care the patient - correct, nothing is wrong 8. Which of the following is high risk of colon CA? a. 50-year-old male with prostate CA and taking colace - unrelated b.60-yr-old male who has a history of surgery for Hirschsprung's disease when he was 2 years old - that was too long ago c. 48-year-old smoker - big risk factor. 9. A patient is on CPAP with apnea. Purpose of CPAP is: a. To breathe for patient when patient stops breathing - wrong. Definition is close to SIMV, but not sure. b. To keep respiratory tract open - correct. c. Other options 10. In a 2-day-old infant, what is normal? a. Touch the left cheek of the infant, will turn head to the left, flexing left extremities and extending right extremities - correct b. Hold the infant upright and infant steps with one foot alternating with other foot - too early for 2 days 11. Herbal medicine- Echinacea a. St. John's Wort b. Take with caution; it interferes with Coumadin - correct statement c. Garlic decreases BP Echinacea shouldn't be combined with other drugs that can cause liver damage. And because this herb may stimulate the immune system, it may interfere with the effects of immunosuppressants. 12. What would you instruct a caregiver taking care of patient with G-tube at home? a. Dress G-tube with NSS - dressing should be dry b. Inject 5 cc of air before feeding to check if G-tube is in place - correct, check the placement first. c. Raise the head of the bed - also necessary, but placement first. 13. A boy has hemophilia. What is the best activity for him? a. Biking with helmet? - correct. Some of the more commonly recommended activities for children with hemophilia include swimming, bicycle riding, walking, jogging, tennis, golf, dancing, fishing, sailing, and bowling. b. Playing soccer with knee pads? - boy should avoid contact sports. c. Playing jungle in a sandbox or sand area - low activity

25

14. Mother of a 4-year-old is newly diagnosed with leukemia. Which of the following statements need follow up? a. I will seek a second opinion - disbelief, normal process of grieving. b. This hospital is giving us a hard time - sort of denial, but not sure... c. I will celebrate and make a party and invite all his friends when we get home - should be correct. What's the reason for the party? The word "all" is suspicious to me. The child is immunosuppressed, can catch any opportunistic disease from the invited friends. 15. A client is with bulimia nervosa and admitted in the hospital. The nurse would a. Make a contract with client that no more weight loss will occur - correct upon discharge. but in real life doesn't work at all. 16. An old woman has mouth sores and anorexia. What is the best diet for her? a. A glass of cranberry juice - can be irritable b. Milkshake - correct; provides nutrients plus needed calcium for depleted bones in elderly women. 17. How can the nurse tell that a woman in labor is experiencing utero-placental insufficiency? a. Early deceleration - wrong b. Late deceleration - correct (found in the internet) http://www.nursingceu.com/NCEU/courses/placentanc/ c. Variable deceleration - wrong 18. A woman was admitted in the delivery room with prolapsed cord. What will the nurse do? a. Cover the cord with dressing moistened with NSS - wrong, there is no purpose for that. b. Place the mother in knee-chest position - correct, decreases the pressure on cord. 19. For 2 days post-op BKA with diabetes. What action will the nurse question? a. Patient teaching crutch walking to a 2-day post-op BKA - normal finding b. Social worker talking to patient about home health after discharge - normal finding c. Dietician leaving pamphlets on diet restriction at patient's bedside to read - correct, teaching should be done by RN d. Other option - wrong 20. A patient who is receiving radiation therapy has nasopharyngeal cancer. The nurse would assess? a. Dry mouth - correct, the mouth contains hundreds of different bacteria, some helpful and some harmful. Chemotherapy and radiation therapy can cause changes in the lining of the mouth and production of saliva and upset the healthy balance of bacteria. These changes may lead to mouth sores, infections, and tooth decay. b. Difficulty swallowing - this also could be affected

26

21. A patient with moderate Alzheimer's disease was just discharged from the hospital. The nurse's best teaching to the family would be: a. Provide limited time to maintain and restructure activities - correct, but I don’t personally like the word “limited” b. Unlock the doors in case of fire to provide safety - sounds like correct, because it addresses safety issues, but… Alzheimer’s patients tend to wondering around, often get confused, and in the case of fire it’s not enough for family members just to unlock the doors, they have to rescue the patient. 22. Respiratory assessment of a 25-year-old: a. Squeaky sound - wrong b. Bronchovesicular sound heard at the lung bases -wrong Bronchovesicular sounds are heard in the posterior chest between the scapulae and in the center part of the anterior chest. Bronchovesicular sounds are softer than bronchial sounds, but have a tubular quality. Bronchovesicular sounds are about equal during inspiration and expiration; differences in pitch and intensity are often more easily detected during expiration. c. Other option - could be correct 23. A patient with TB will manifest the following signs and symptoms (enumeration) a. Dry cough b. Diaphoresis c. Temperature- 37.50C d. Lethargy A, B, C - correct. Dry cough can be than turn into productive cough with bloody secretions or greenish sputum. 24. A patient with a thoracic injury is complaining of restlessness and headache. What will you assess further? a. Blood pressure - correct, because low BP is an indication of internal hemorrhaging. 25. A patient is under chemotherapy. What will you assess further? a. Hydration status b. Nutritional status Both need to be monitored and assessed, but I would answer A, based on the principle of priority. 26. A nurse who is 10 weeks pregnant can take care of a patient with: a. Fifth disease who is taking motrin wrong. Pregnant women and people with impaired immune systems or certain blood disorders (such as sickle cell disease, or thalassemia are at high risk for developing complications from fifth disease. These people need close monitoring by a health professional after exposure or if they develop symptoms of infection. Medical treatment for complications sometimes requires hospitalization. b. AIDS patient with candidiasis - correct, the disease not that contagious.

27

27. Patient had BKA 24 years ago. How do you prevent hip contracture? a. Elevate the stump - wrong, doesn’t address the problem, it’s a preventive measure for swelling b. Keep in prone position for a short period of time - correct, number of hours in prone position should be in doctor’s orders 28. A newborn baby with hypothermia is kept in a radiant warmer. What is the next step to be done? a. Apply skin probe to the abdomen - wrong, even the nurse places the infant in a fancy warmer that provides body temperature measurements, they are not 100% correct, skin probes can become detached, be defective and often provide false readings; measuring the temp is nursing priority. b. Take vital signs - correct, VS are taken frequently, especially the temp. 29. A newborn baby with acrocyanosis and harlequin sign. What will the nurse do next? a. Check APGAR score - correct. This is done to evaluate color, reflex activity, tone, heart rate, respiration - done in delivery room to determine need for resuscitation b. Weigh the baby - doesn’t address the problem c. Determine if Vitamin K was given - same as above 30. A patient had colonoscopy with bowel perforation. What will the nurse assess further? a. Abdominal distention and hyperactive bowel sound - wrong, these are normal findings. ABD distention could be because of the nature of the colonoscopy - small amounts of air could be injected to facilitate the advancement of the scope into the colon. b. Abdominal pain and tenderness - correct, indication of bleeding and peritonitis. BP should also taken 31. In a patient having a hemodialysis, what will the nurse assess? a. Elevated BP - correct, means that the machine doesn’t filter the blood properly, leaving toxins in the circulation. Decreased BP is normal, but not elevated. b. Leaking dialysate - wrong, it’s impossible. Dialysate is used in peritoneal dialysis, not in hemodialysis. 32. A patient with a skeletal traction fell down, with traction hanging on the floor. The nurse noticed moderate serous discharges from the pin site. What is the nursing diagnosis? a. Risk for infection - wrong, infection comes after injury b. Risk for injury - correct, pt fell down, probably misalignment took place, risk for infection comes after that. 33. A Muslim patient is terminally ill. The nurse's best expectation is: a. A minister will stay with the patient - correct, b. A family member of the same gender will bathe the body when the pt. dies - wrong, the body should not be washed. Islamic washing of the body is done before burial (Ghusl before burial). If no relatives are available then the Islamic Council should be contacted. Reference taken from http://www.health.qld.gov.au/multicultural/pdf/islamgde.pdf

28

34. A Muslim with diabetes can have the following diet: a. Vegetables - correct, these are approved in HALAL (good food) b. Hotdog - wrong, may contain pork - HARAM (bad food, or unacceptable food) c. Hamburger - wrong, the same reason as above 35. What is the best health teaching in a pt using tampons? a. Rashes or allergy reaction - wrong, do not use tampons if you have a skin infection near the genitals. b. Use super absorbent for heavy menstruation - wrong, contraindicated because of the risk for toxic shock syndrome. c. Change tampon every 8 hours - wrong, Tampons should be changed every 4 hours and PRN d. Other option- good personal hygiene - correct, good hygiene is always good. 36. A breastfeeding mother is complaining of soreness of both breasts. Which of the following health teachings need further attention? a. Apply Vaseline ointment to the areola after feeding the baby - wrong, this doesn’t require nurse attention. This is done correctly. Also instruct the pt to “avoid soap and lotions or creams that have alcohol. But, you can put a thin layer of lanolin-based ointment on your nipples and areola after feedings. And, do not use plastic-lined bras or bra pads. Also, change your bra and bra pads as soon as possible when they get wet in order to keep your nipples dry.” b. Emptying the breasts - correct. Soreness of the breasts usually occurs because of improper positioning of the baby during the feedings and this needs attention of the nurse. 37. Patient is receiving TPN thru a PICC line. What will the nurse do? a. Use the sterile pad when removing the PICC line b. Apply pressure for 5-10 minutes The question is not quite understood. May be the questions asks about discontinuation of the PICCline? If so, both answers fall into the “correct” section. But… being a student nurse I observed how they d/c PICCline. Physician did the procedure, and RN applied pressure for 5-10 minutes. Again, the question is not quite understood. 38. What should be assessed in Extracorporeal Lithotripsy? a. Hematoma on the site b. Presence of blood in the urine for the first 24 hours 39. How to position a newborn after feeding a. Put on his back - wrong, could be predisposing factor for GERD b. Turn to the left side - correct, prevents burping and aspiration. 40. Growth and development of a 7-month-old a. Just beginning to turn from his abdomen to back -wrong, this happens earlier b. Transferring an object from one hand to another - wrong, this happens earlier c. Standing and holding onto a furniture - correct

29

41. Who will you discharge first? a. A COPD patient with oxygen, elevated PCO2, pulse oxymetry reading is 92 % - correct, this is the most stable pt b. Diabetic patient with blood sugar of 360, HCO3 is elevated - wrong, 360 is too much. c. Renal patient with urine specific gravity of 1.010; BUN- 24 - wrong, BUN elevated 42. What is the cause of mononucleosis? a. Caused by cytomegalovirus in the air b. Caused by cytomegalovirus thru droplet/secretions - correct 43. OB patient- who needs the most attention? a. 18 weeks complaining of muscle cramps - wrong, not the priority b. 21 weeks complaining of headache - wrong, this is common in pregnancy. c. 34 weeks complaining of shortness of breath - correct, SOB is a priority 44. What patient condition needs further teaching? a. Patient with bipolar disease embracing another patient with depression 45. Antisocial patient needs more attention when she: a. Complains about unit rules to another patient 46. Difference between slander and libel a. Healthcare worker talking about incompetence of a nurse - correct, this is slander. slander is the spoken false defamation of a person or entity b. Healthcare worker giving negative comments to a group of nurses - wrong, just expression of anger. 47. Health teaching to a pregnant woman a. Drink 4 glasses of milk per day - wrong, too much milk b. Eat a kind of vegetable and fruit each meal - correct, fairly enough veggies and fruits in the diet. 48. How will a father of an 18-month-old assess a child with otitis media? a. How frequent is the baby drinking milk during the night? - correct, because when drinking, child's ears "pop" when yawning or swallowing, the eustachian tube is adjusting the air pressure in the middle ear. 49. What will you assess in a patient with post-epidural anesthesia? a. Hypotension 50. OB patient, 38 weeks pregnant whose cervix is 5 cm dilated, intact membranes, and bulging presenting part. 0-station, asks if she can go for a walk. The nurse's best response: a. I think it would be best for you to stay in bed - wrong, contains the words “I think” b. It's okay for you to walk, but if the membranes will rupture, go back to your room-correct statement. c. It would be best for you to lie down - wrong, lying down increases blood pressure.

30

51. What is the intervention of a prolapsed cord? a. Insert finger into the vagina to prevent pressure and push head 52. Patient had mastectomy; what will worry the nurse? a. Husband is more intimate, looking at the operative site - wrong answer, shows good husband adjustment to the wife’s illness. b. Wake up at 2:00 am, and can't go back to sleep -correct, this should alert the nurse. A sign of depression. Further interventions needed. 53. Chest tube malfunction: a. Continuous bubbling in the water seal - correct, indicates air leak b. Continuous bubbling in the suction chamber - wrong, this is normal finding. c. Intermittent bubbling in the water seal -wrong, this is normal finding. This usually occurs during pt’s cough, sneeze, or deep breathe. 54. Effectiveness of hemodialysis in a patient who is employed: a. Increase energy level after toxins were removed from the body 55. Hepatitis C; how to draw blood a. Needleless syringe 56. Burping a baby; what is the best position? a. Put baby in a sitting position 57. Health assessment history of a patient with endocarditis, ask: a. Did you have any infection?- correct. Bacterial infection is the most common source of endocarditis b. Did you have a sore throat 2 weeks ago? -wrong, why should it be limited to 2 weeks? 58. A patient on chemotherapy is complaining of anorexia. What food will you give? a. grapefruit juice - wrong, very acidic, may worsen anorexia and induce vomiting. b. Orange juice - wrong, somewhat helpful, but not the best choice among given. c. Eggnog and milkshake - correct, provide maximum nutrients and calories 59. What will you ask to a pt. with sickle cell disease? a. Does anybody in the family have the disease? 60. What will you expect in a patient with allergic rhinitis? a. Presence of mucous secretions in the throat 61. In a child with asthma, what health teaching needs follow-up? a. Playing with wooden puzzle - wrong, not contraindicated b. Sleeping with stuffed toy - correct, usually stuffed toys are great sources of dust, dust mites, etc. 62. A patient with BKA, how will you assess that the patient is coping? a. Fitting the prosthesis - wrong, pt. accepts this as doctors insist. b. Inspecting the stump, looking at the mirror every morning - correct, the same as with pts with mastectomy, who looks at the incisions and JP drains.

31

63. An adolescent with diabetes says, "There's nothing to worry about my illness". The nurse's best response is: a. You sound not worried 64. A patient with cancer is complaining of pain in a scale of 9 out of 10. The nurse noticed that the patient is talking and laughing with visitors. What will the nurse do? a. Hold the pain medication - wrong, because pt’s pain is a priority for the nurse. b. Give the pain medication as ordered - correct, pain of the patient is ONLY his or her perception. Maybe the pt is experiencing acute pain, but hiding it from the visitors… 65. A child with hemophilia; play activity is: a. Play soccer with an elbow pad - wrong, all contact sports are contraindicated b. Ride in a bicycle wearing a helmet - correct, addresses safety measures and this is the best activity possible for a child with hemophilia. c. Playing in a sandbox - wrong, children should be encouraged to play actively 66. A patient is trying to pull out his NG tube. The nurse was ordered to put restraint after the patient calms down. What would be the most appropriate statement to tell the patient? a. I am sorry that we have to use the restraint because you leave us no choice - wrong explanation of the restraints. b. It's fortunate that you did not hurt other patients, but if you do it, we will restrain you - wrong, because this sounds as a threat to the pt. c. Other option - could be right, something like: “I know, this is an uncomfortable procedure, but this is done for your safety and maintaining your health.” 67. What is the assessment that will concern the nurse most after the patient had bronchoscopy? a. Absence of gag reflex -wrong, not a priority, gag may return soon. b. Diminished breath sounds - correct, this indicates bronchospasms, and priority at this time. 68. A non-complaint TB patient has an order of INH/Rifampin. How do you know that the patient is taking the medication? a. Color of urine is red-orange 69. What is true about BSE? a. It is done 1 week after the end of menses - correct statement. Week before or week after menses this should be performed. b. It is done after ovulation - wrong, this is hard to detect. c. Face the mirror, put hands behind your hip, lean forward - wrong, d. face the mirror, raise the corresponding hand - wrong 70. A mother has history of premature labor at 35 weeks. Now she's pregnant for 32 weeks. What will you tell the patient to prevent premature labor? a. Avoid nipple manipulation - correct, this could induce Oxytocin production, and induce premature labor. b. Other option 71. What is true about mammogram? a. No deodorant on the day of the test

32

72. A patient is taking lactulose. Choose all that apply. a. Check the ammonia level - correct, check. b. Check the urine - correct, check, lactulose is excreted in urine. c. Given after antacid - wrong, Antacids may decrease the effects of lactulose d. Given between meals - wrong, it can be taken anytime, no diet restrictions on that. 73. A patient is being discharged. What will you teach the patient about ferrous sulfate? Choose all that apply a. Given between meals - correct, this med should be taken on an empty stomach. 1 hour before meals or 2 hours later. b. Tell patient to avoid organ meats - wrong, organ meats is a good source of iron c. Tell patient to call MD if there's a dark stool - wrong, this is normal appearance of the stool while taking this medication. 73. What is true about borderline personality disorder? a. They have history of physical abuse 74. What is true about Norplant? This medication has been taken off the market in the United States a. It is repeated every seven years b. Can get pregnant in six mos. if removed c. Spotting from time to time 75. BKA after 24 hours (position) a. Patient on his tummy several times a day 76. A patient with closed head injury complains of polydipsia. What will be the priority to monitor? a. Capillary refill - wrong, not a priority, but needs to be assessed. b. Blood glucose -wrong, not a priority, but needs to be assessed. c. Specific gravity of the urine - correct, during CHI putitary gland could be damaged, and SG should be monitored closely 77. What is a normal development in a 7-month-old baby? a. Can sit with support - wrong, should sit without support. b. Can pass object from one hand to the other - correct, normal finding. c. Can push self in upright - wrong 78. Which one is your priority for a 2-hour-old baby? a. Baby with acrocyanosis - correct, this indicates circulatory insufficiency. Can be fixed by straightening the limb. b. Vitamin K to baby with petechiae - wrong, petechiae is normal in babies 79. What assessment would the nurse be most concern of? a. Continuous bubbling in the water seal 80. What is your priority with a seizure patient? a. Padded side rails

33

81. Newly admitted manic patient, what activities would your provide to facilitate good assessment? a. Card games with the patient - correct, good assessment could be done through this simple procedure. Concentration could be evaluated, mental assessment, etc. b. Drawing activities - wrong, usually this is used by shrinks to assess the pt. c. Put him in group therapy - wrong, does not facilitate assessment. d. Walking - wrong, but somewhat correct, only walking pattern could be assessed. 82. A patient is depressed. What is the statement that would most concern the nurse? a. I am not going to see you again because I feel much better 83. What is the priority nursing diagnosis in multiple myeloma? a. Pain related to osteoporosis 84. Know the IV insertion sequence 1. Change client’s gown for easier removal. 2. Open sterile packages correctly. 3. Prepare IV infusion tubing and solution:

a. Checked “five rights” of drug administration. b. Open infusion set. c. Place roller clamp about 2 to 5 cm (1-2 inches) below drip chamber. d. Remove protective sheath over IV tubing port on IV solution bag. e. Insert infusion set spike into fluid bag or bottle. f. Prime infusion tubing by compressing drip chamber and filling to 1/3 to 1/2 full. g. Remove protector cap on end of tubing (if necessary), released roller clamp, and allow fluid to fill tubing. h. Remove air bubbles. i. Replace protector cap on end of infusion tubing.

4. Prepare heparin or normal saline lock for infusion. 5. Apply clean gloves. 6. Identify accessible vein. 7. Apply flat tourniquet above proposed insertion site. 8. Select appropriate vein for IV insertion.

a. Stroked extremity from distal to proximal below site. b. Had client open and close fist. c. Lightly tap over vein. d. Apply warmth to area for several minutes.

9. Release tourniquet temporarily. 10. Place needle adapter end nearby on sterile surface. 11. Replacetourniquet and checked client’s distal pulse. 12. Cleanse site with alcohol or povidone-iodine and allow to dry. 13. Perform venipuncture:

a. Anchor vein by placing thumb over vein and stretching skin distal to the selected site. b. Warn client of sharp, quick stick. c. Insert over-the-needle catheter (ONC), IV catheter safety device, or butterfly needle with bevel up at a 20- to 30-degree angle slightly distal to the actual site in the direction of the vein.

14. Stabilize catheter/needle with one hand and released tourniquet with the other hand. 15. Remove stylet of ONC; did not recap stylet. Glide protective guard over stylet of IV safety device. 16. Connect needle adapter of infusion tubing set of heparin/saline lock adapter to hub of ONC or butterfly tubing. 17. Intermittent infusion:

a. Hold heparin/saline lock firmly with one hand and cleansed with alcohol. b. Inserte prefilled syringe with flush solution into injection cap. c. Flushed injection cap slowly with solution. d. Withdraw syringe while still flushing.

34

18. Continuous infusion: Connect tubing and open roller clamp to allow solution to infuse. 19. Tape or secure catheter/butterfly needle. 20. Apply sterile gauze or transparent dressing to site. Avoid taping over insertion site and connections. 21. Recheck flow rates of IV fluid infusions. 22. Wrote date and time of IV insertion and catheter/ needle size on dressing. 23. Dispose of sharps in appropriate container. 24. Instruct client how to move around without dislodging the IV. 25. Change peripheral access site per agency policy. 26. Replace IV solution when less than 100 ml remained in bottle or bag. 85. A telemetry patient with a pulse of 73-52. Which one will you question about the doctor's order? a. Propranolol everyday - correct, Inderal is contraindicated in pts with irregular heartbeat and slow heart rate. b. Digoxin everyday - wrong, this med is likely to be prescribed for the patient. c. Tagamet 300 mg everyday (?) - wrong, nothing is contraindicated for tagamet (heartburn relief) d. V/S every 8 hours - wrong, this can’t be questioned. 86. How many ml can you give to an infant patient in IM injection? a. 0.5 ml - correct. b. 10 ml - wrong, too much even for adult c. 15 ml - wrong, the same reason as above. 87. Where can you hear vesicular sounds? a. Trachea - wrong, this is for bronchial sounds. b. Bronchi - wrong, c. At the base of the lung - correct definition. 88. Client with active TB. What will the nurse advice? a. Cover mouth with paper tissue when coughing and sneezing - correct preventive measure. b. Use surgical mask when talking to a friend - wrong, not necessary, friend should ware the mask himself, but not a surgical one ☺ 89. What do you expect in an elderly? a. Decreased salivation - wrong, this is usually due to the side effect of medications they take. b. Decrease in anteroposterior diameter - correct, normal aging process. 90. Patient is on chemotherapy. What indicates understanding of the therapy? a. I will avoid parties while under chemotherapy 91. Patients with anorexia nervosa should be weighed weekly 92. A nurse found an unresponsive patient. What is the priority action? a. Open airway - correct, Airway first. b. Check carotid artery - wrong, this is second

35

93. A patient with subtotal gastrectomy is on intermittent suction. What is the color of the secretion? a. Red-brown b. Greenish c. Yellowish d. Coffee-ground I don’t know what to choose, because all med books say that it should appear cloudy and green, tan or brown in color. 94. A patient underwent total gastrectomy. How would you know that the patient understood his condition? Patient needs: a. Injection of Vitamin B12 for life 95. A patient in 3rd trimester has painful bleeding. What is the suspected diagnosis? a. Abruptio placenta - correct, this is classical case b. Placenta previa - wrong, previa-painless (good to remember P_P) c. H-mole - wrong, even don’t know what is that☺ 96. A patient underwent ECT. After the procedure, the pt slept. What will you do? a. Let the patient rest - wrong b. Wake the patient up and orient - correct, this is the part of the treatment, wake the pt up and orient. 97. Blood level of digoxin is 1.8. What will you do? a. Give the next dose - correct, 0.5-2.0 - is a safe range. b. Withhold and notify the MD - wrong, not necessary. 98. A patient with continuous bladder irrigation has bloody output with blood clots. What is your nursing action? a. Increase the irrigation b. Stop the irrigation c. Notify the physician 99. A patient is in sickle cell crisis. What is your priority? a. Oxygen - wrong, in this case it’s not a priority. b. Fluids - correct, always a priority in SCC. 100. A patient is experiencing itchiness during blood transfusion. Your first action is to: a. Stop transfusion

Section TWO

1. Phenolketonuria (PKU) diet a. No milkshake

36

2. A patient's glycosylated hemoglobin assay result is 10%. What is your priority nursing diagnosis? a. Risk for injury - wrong, b. Ineffective management of the disease - correct, it is above 7% and considered abnormal. c. Risk for aspiration - wrong, totally wrong, can’t be an indication for that. d. Ineffective patient teaching - wrong 3. Potassium chloride IV is given to patient. What do you check first? a. Blood pressure b. Urinary output c. Respiration d. Pulse - correct, apical pulse should be checked first. 4. Visual examination is done to a 2-year-old child. What would you use? a. Snellen's chart b. Allen color chart c. Cover right side of the eye then the left side 5. A client is in active labor. Contraction is 30-45 seconds, interval is 3-5 minutes. What would you do first? a. Reposition client to the left side b. Check the blood pressure - correct c. Apply an external fetal monitor d. Apply an internal fetal monitor 6. An infant with Trisomy 21 was admitted 3 days ago. Which behavior by one of the parents would suggest effective coping? a. One of the parent says, "I need to find a second job to cope with the needs of our family" b. The mother states, "He's so cute, I want to hug him all the time" c. The mother cries frequently d. One of the parent states, "I will stay at home all the time because the baby have some special needs.” - correct, addresses the problem. 7. For a 2-year-old, what is the appropriate toy to give? a. Push-pull toy - wrong, this toy is needed in earlier age b. A racking horse correct, appropriate for the age of a child c. Playing cards - wrong, too early. d. A picture puzzle - wrong, too early. 8. What observation in an infant needs further assessment? a. A 7-month-old with negative tonic neck reflex - wrong, normal finding. b. A 12-month-old who loves staring at the colorful lanterns - correct, this needs further assessment. At this time an infant usually explores fine motor skills, not just lying in cribs. c. A 6-month-old who needs support when sitting - wrong, this doesn’t require an assessment.

37

d. An 8-month-old with positive babinski reflex - wrong, the same as above. 9. Post surgery cleft lip/palate. What statement by the mother would suggest that she understands the instructions? a. I will let my child lie prone to promote drainage of secretions - b. I will let him lie on his back to promote comfort - wrong, risk for aspiration. c. I will irrigate/clean the mouth of my baby with water before and after feeding 10. A patient has a new colostomy. What indication would indicate that colostomy is starting to function? a. Flatus in the colostomy bag - correct, flatus comes first b. Moderately formed stool from the site c. No bowel sounds - wrong, indicates that peristalsis didn’t return after surgery. 11. A client has new ileal conduit in place. What observation needs further assessment? a. Dark purple color of the stoma - correct, indicates ischemia. b. Cloudy drainage c. Blood in the drainage bag d. 30 ml output 12. A rape victim continuously tells the staff what has happened to her. What will you do? a. Offer her alternative measures of how to cope with the crisis b. Refer her to rape crisis center c. Continue listening to her - correct d. Encourage her to sue the assailant - wrong, not the business of the RN 13. A patient has heart failure with low cardiac output. What is your management to correct the problem? a. Elevate legs at the level of the heart - wrong, should be above. b. Elevate legs above the level of the heart - correct, this promotes venous return and will pull some fluid from the legs into circulation. c. Raise the head of bed at least 30 degrees - wrong, of no use. d. Place client in prone position - wrong, this is unsafe. Suppresses vena cava. 14. A patient with C6 injury is sweating, uneasy and diaphoretic. What will you assess? a. Check his respiration - correct, respiratory compromise symptoms b. Check patency of the urinary catheter c. Reposition client in high-fowler's position d. Reposition client side-lying 15. A patient 8 hours post-surgery complains of pain and request scheduled medication for pain. While giving the medication, the client laughs while watching TV. What will you do? a. Tell the client to decrease the dosage because it could cause addiction b. Assess the client's pain level and give the medication at the peak c. Do not give the next dose of medication d. Give the medication as prescribed - correct, the pain is measured by client’s perception, not by nurse’s judgment.

38

16. A client, 32 weeks gestation, asks why she has to take betamethasone (Celestone). Tell her it is to: a. Treat premature labor contraction - b. Help lungs of the unborn to mature - c. Prevent infection - correct, d. Decrease clients anxiety - wrong, doesn’t help with that. 17. A client, 3 hours postpartum, is bleeding profusely. Her fundus is firm and at the level of the umbilicus. What will you assess? a. Standing orders for an Oxytocin drip b. Massage the fundus c. Check the perineum (for lacerations) d. Check for bladder distention - correct, this should be assessed first 18. A client on hypothermic blanket. What observation needs further observation? a. RR= 20 b. BP=120/80 c. Pulse= 80 d. Temperature before is 38.0, now 39.70 - correct, this requires immediate attention. 19. Appropriate use of restraints a. 16-year-old, uneasy and agitated, curses staff and other patients - wrong, no evidence for harm to the staff, patients or self. b. 65-year-old patients with Alzheimer's disease who is confused and wanders at night - wrong, this client needs close attention, not restraints. c. 1-month-old post-circumcision - correct, d. Other option 20. A patient with Bell's palsy understands his condition when he states: a. That the cause of his condition is viral - wrong statement. b. That situation will resolve in a couple of weeks -wrong, without treatment it doesn’t happen. c. That surgery will correct his problem - correct, decompressing of the 7th nerve will correct the problem. d. That frequently tearing of his eye is caused by blockage of the lacrimal duct - wrong, the lacrimal duct works properly, but the tears can’t be spread on the eye due to the lack of blinking reflex. 21. A patient has Cushing's disease. What is the appropriate diet? a. Low caloric, low sodium - correct. b. Low protein, high sodium c. High fat, high caloric d. High caloric, high sodium 22. Who is high risk of colorectal cancer? a. A client with rectal polyp two years ago - correct. Classical case. b. A patient with duodenal ulcer with complaints of pain

39

c. A patient with Hirschsprung's with rectal bleeding 23. A patient developed fat emboli. What will you observe first? a. Restlessness - correct, this comes first, then - increased BP. b. Increased BP - wrong, this comes second c. Decreased urine output - wrong, not indicative d. Decreased LOC - very late symptom. 24. Contraction monitoring is used for: a. Contraction and effacement b. Dilatation c. Contraction and fetal well being - correct. d. Contraction and dilatation of the cervix 25. A patient has arterial insufficiency a. Smokes 2-4 cigarettes per day - correct b. Eat fatty foods frequently - correct as well, risk for high cholesterol levels. c. Eats banana and cantaloupe d. Has a sedentary lifestyle 26. Patient on ventilator is to be weaned off with tracheostomy with cuff in place. What is the priority nursing diagnosis? a. Risk for aspiration b. Ineffective breathing pattern c. Risk for airway obstruction - correct, mucous plug can form and also because of excessive secretions. d. Decreased cardiac output 27. 12 hours after amniocentesis, client calls the hospital and complains that she's not feeling well. What will be an appropriate response by the nurse? a. Do you have fever? - indicates sepsis, which occurs after 12-24 hours after the procedure. b. Do you have difficulty breathing? 28. A patient with bulimia nervosa needs further assessment when she: a. Express anxiety upon discharge b. Comments that her dress are to fit for her - correct, always obsessed with that. c. Exercises 1 hour everyday for her to keep fit 29. Which child will be assessed for hearing loss? a. A child who has otitis media and was treated with antibiotic - correct, antibiotics are usually ototoxic. b. A child with tonsilitis with low-grade fever c. A child with URTI d. A child with asthma

40

30. A patient has fractured femur. Two days after application of a long leg cast, he becomes agitated and restless. What should you do initially? a. Administer prescribed O2 and notify the physician b. Raise the side rails of bed and notify the physician c. Place the patient in low fowler's position - correct, after that - call the doc. d. Administer meds for pain 31. A patient with Hodgkin's disease best room assignment a. A patient with MRSA with draining pus on skin lesions b. A patient with chickenpox c. A patient with osteomyelitis d. A patient with simple fracture of the right leg - correct. 32. A patient is post-liver biopsy. What will you do first? a. Position to right side b. Monitor or check blood pressure - correct. c. Start an IV infusion d. Administer prescribed medications for pain 33. A patient ileostomy in place. The patient understands instruction is he states: a. "I will allow the drainage bag to be 3/4 full before replacing or draining" b. "Connect the tube to a bedside drainage bag when sleeping" - correct. 34. A 3-year-old with cystic fibrosis will begin postural drainage. What is your initial nursing action? a. Check RR b. Check PR c. Check BP d. Determine or assess the part of the lung that has the greatest obstruction - correct. Then report the findings to the physician. 35. You are about to begin abdominal examination. What is the proper procedure? a. Auscultate first before palpation - correct. The perfect sequence would be: observe, auscultate, palpate, percuss. b. Give prescribed pain medication c. Palpate first before inspection d. Uses bell of the stethoscope 36. A patient complains of fatigue and pallor. What should you assess? a. BUN b. Creatinine c. RBC - correct. They carry oxygen to the cells, hypoxemia results in fatigue an pallor. d. WBC 37. An infant is on phototherapy. What is an appropriate nursing action? a. Cover the genitalia during phototherapy b. Cover the infant with blanket to prevent hypothermia c. Feed the infant while on phototherapy - correct.

41

38. What is the clinical manifestation of a patient with pneumothorax? a. Absence of lung sounds - correct. b. Rales c. Crackles at lung bases d. Rhonchi 39. A patient with schizophrenia paranoid type is seen grimacing and clenches fist. Suddenly the patient cries "NO" in aloud manner. What will you assess first? a. Reorient and explore client's feelings and help verbalize his emotion b. Assess for adverse side effects of prescribed medications c. Assess client if he is experiencing delusions - correct. 40. Upon assessment of a 12-year-old girl, what is the appropriate occurrence at this age? a. Developed breast b. Regular menses c. Height increases 3 inches within a period of 9 months d. Has delicate features - correct. 41. High risk for iron deficiency anemia a. 12-year-old early menses - correct. b. 65-year-old chronic alcoholic - wrong, already anemic, not “at risk for…” c. 45-year-old post menopausal d. 35-year-old multipara 42. Hospitalized patient has a husband who is an alcoholic. What statement by the patient indicates that she will not be a co-dependent? a. Says that she will find and get all the stocks of alcohol and destroy it b. Will encourage husband to exercise more and to drink less - correct, fully understands the problem of drinking. 43. Patient is post left radical mastectomy. What statement by the client indicates the understanding of discharge teaching? a. States that she will let her blood pressure be taken on the left arm b. I will continue my regular breast exam on my right breast every month - correct. c. I can do heavy lifting in my right arm as long as I have enough rest 44. A patient has chicken pox. What symptom needs further assessment? a. Skin eruptions at the back and chest b. Slight elevation of temperature c. Painful skin ulcerations at the buccal mucosa and lips - correct. d. Lesions rupture very easily 45. A patient with mitral valve regurgitaion has surgery with mechanical valve prosthesis. Which of the following findings need further assessment? a. There is a loud clicking sound when breathing - correct. b. Temperature elevation 38 degrees centigrade - wrong, this is expected during 48 hrs.

42

46. Before administering epidural anesthesia, the priority nursing action is: a. Hydrate the patient b. Place on prone position c. Check blood pressure - correct. Hypotension is a major concern during EA d. Check the respiratory rate 47. A patient has a cataract on his right eye. Signs and symptoms include: a. Halos around lights - correct. b. Blurring of vision c. Unsteady gait and loose balance easily 48. Priority health teaching for a 65-year-old patient a. Eats high fiber foods - correct, risk for constipation. b. Consume more of simple sugars c. Increase calcium intake 49. A patient with Parkinson's disease is taking meds. What are the side-effects of his medications? a. Pill rolling tremors -correct, sign of extrapyramidal s/e b. Increased RR c. Decreased PR d. Decreased BP 50. Safety measures by parents to prevent poisoning a. Secure covers of medicines tightly b. Place medicines in a secured locked cabinet - correct, c. Place medicines in the highest possible cabinet holder - wrong, nothing impossible for the children☺ 51. A patient has arterial insufficiency. Assess patient for: a. +1 pedal pulses - correct b. +3 pedal pulses - wrong, this is normal c. Edematous - wrong, this is in venous insufficiency. 52. A child has acute glomerulonephritis. What observation needs immediate attention? a. Severe headache, decreased urine output - correct, early sign of renal insufficiency b. Hematuria c. Proteinuria d. Polyuria 53. A patient is on a non-rebreather mask. Which of the following needs further observation? a. PaO2= 90 % b. O2= 4 L/min c. PCO2= 65 - correct. d. PO2=92

43

54. Which of the following patients should you see first? a. A patient for hemodialysis, AV shunt not working and a potassium level of 5.8 Meq/L- correct, needs immediate attention. b. A patient with liver disease has bleeding esophageal varices and is vomiting 55. A patient with PVD has arterial insufficiency in his right leg. The nursing intervention is: a. Elevate head of the bed and legs in dependent position - correct, b. Elevate legs above the level of the heart - wrong, this is a good implementation in venous insufficiency. c. Apply elastic hose to the affected limb - wrong, same as above d. Raise foot of the bed - wrong, same as above. 56. Patient is on TPN via central line. What should you report? a. Fever of 38.0 C 57. Which would be a high fiber diet? a. Tofu and cantaloupe - correct. b. Bread and coffee c. Baked chicken and pork wrong, this is high protein diet 58. Gout diet a. No organ meat 59. A patient was taking heroine. The patient will order: a. Narcan 60. Oxytocin side effect Side effects with oxytocin are not common. Serious side effects include:an allergic reaction (shortness of breath; closing of the throat; hives; swelling of the lips, face, or tongue; rash; or fainting); difficulty urinating; chest pain or irregular heart beat; difficulty breathing; confusion; sudden weight gain or excessive swelling; severe headache; rash; excessive vaginal bleeding; or seizures.

Section THREE

70. A patient has attended a medication class for tricyclic antidepressants. He demonstrates he understands the teaching by stating: 1. "I must exercise and eat foods high in fiber." 2. "I must have my blood drawn at least once a month after discharge." 3. "I must stay out of the sun or wear sun block." 4. "I cannot eat cheese or drink wine."

44

71. Multiple Sclerosis is characterized by which of the following: 1. A progressive neurological disorder characterized by the degeneration of basal ganglia in the cerebrum 2. A progressive demyelinating disease which affects fibers brain and spinal cord 3. A progressive and sometimes fatal disease which results in degeneration of the motor neurons 4. A progressive disease involving neuro-muscular transmission of impulses of voluntary muscle 72. You are assigned to care for a client diagnosed with Bell's Palsy. Which of the following would be an INCORRECT nursing action or intervention?: 1. Offer small frequent feedings of soft foods 2. Apply a facial sling to support facial muscles 3. Teach the patient to close the lid(s) of the affected eye(s) periodically, and to instill artificial tears 4. Vigorously massage muscles of the affected side at least twice daily, to restore circulation and muscle tone 73. In caring for a client with ALS (Amyotrophic Lateral Sclerosis), the nursing diagnosis with the HIGHEST priority would be: 1. Impaired Physical Mobility 2. Altered Role Performance 3. Potential for Ineffective Airway Clearance 4. Potential for Impaired Verbal Communication 74. A client is admitted to your unit with a diagnosis of COPD. The client tells you that the 2L/min of 02 that he is now receiving are not adequate, and requests that you increase the liter flow to 6-8L/min. The nurse's FIRST RESPONSE or INTERVENTION in this situation would be to: 1. Increase the liter flow to 6-8L/min, as per patient's request 2. Not increase the liter flow knowing it will likely result in atelectasis 3. Maintain the liter flow as ordered 4. Contact the patient's physician to get an order to increase the liter flow 75. A patient is on Respiratory Isolation for Tuberculosis (TB). Which of the following would be an indicator for removal of Isolation Precautions? 1. Absence of adventitious breath sounds 2. Patient has been on Anti-Tubercular Drug Therapy with INH for one month's time 3. Patient has no infiltrates on chest x-ray 4. Sputum Culture is negative for AFB, following a course of INH and PAS 76. You are assigned to work in the Endocrine Clinic. A teenage patient with Type I Diabetes comes for monthly evaluation. Her serum glucose is 175mg/dl and the Glycosylated Hgb is 25%. Based on these lab values, which of the following determinations can the nurse make about the patient's control of her diabetes? 1. That it is in good control 2. That it is being poorly controlled and further evaluation is warranted 3. Serum Glucose is within an acceptable range. Glycosylated Hgb is not significant

45

4. That lab values likely reflect the fact that teenagers usually do not comply with diet at all times, and therefore control is unlikely 77. A client is being seen in the Emergency Room. A diagnosis of Congestive Heart Failure is made. Upon auscultating the client's lungs the nurse hears crackling sounds bilaterally at the bases. In documenting these findings, the nurse would state that which of the following was heard: 1. Rhonchi 2. Wheezing

3. Rales 4. Atelectasis

78. A client with Congestive Heart Failure is placed on several medications including the Beta Blocker Atenolol. The nurse understands that Beta Blockers exert their influence by: 1. Reducing myocardial Oxygen demand 2. Cause vasodilation of coronary vessels 3. Increase heart rate and force of contraction 4. Decrease heart rate and force of contraction 79. A patient in CCU (Coronary Care Unit) is receiving Digoxin (Lanoxin) and Furosemide (Lasix). In assessing the patient's lab values, which of the following might the nurse expect to see? 1. Increase specific gravity of urine 2. Hyperkalemia 3. Hypokalemia 4. Hypernatremia 80. A patient in CCU is diagnosed as having Third Degree (3 deg. A-V Block). Which of the following is NOT an EKG finding in this dysrhythmia? 1. No relationship between P wave and QRS complex 2. Widening QRS complex greater than 0.10 sec. 3. Normal configuration of P waves 4. Heart rate of 60-100 bpm 81. You are caring for a client just diagnosed with 3 degree A-V Block. Which of the following should the nurse be prepared to do? 1. Assist in possible insertion of a pacemaker 2. Administer Digoxin 3. Administer Lidocaine 4. Assist in doing CPR 82. A client has a demand pacemaker inserted. The nurse knows that which of the following is true concerning such: 1. It is unaffected by spontaneous heart beat 2. Only fires when heart rate falls below pre-set minimum rate 3. Uses a sending and pacing electrode in the atria 4. It is always a temporary modality to provide electrical stimuli to the heart

46

83. A client is discharged on Digoxin (Lanoxin) following hospitalization for Atrial Fibrillation. In preparing a Discharge Teaching Plan, the nurse would NOT include which of the following? 1. Take pulse correctly and count for one full minute 2. Report any signs and/or symptoms such as ocular disturbances, anorexia, etc., to M.D. promptly 3. Take another dose of medication if first dose is vomited 4. Withhold drug if heart rate falls below 60 bpm 84. You are caring for a client s/p cardiac catheterization. Which of the following signs/symptom(s) that the patient may experience DO NOT necessarily indicate a possible serious complication requiring intervention by the nurse? 1. Mild pain at the insertion site 2. Decrease in pedal pulses on affected side 3. Development of an expanding hematoma at the insertion site 4. Development of bradycardia 85. A client with a history of Polycystic Kidney Disease is admitted to the Renal Unit for evaluation for dialysis. Which of the following lab values would be MOST significant in determining renal function? 1. Creatinine 8.7 mg/dl 2. BUN 90 mg/dl 3. Serum K+ 7.0 MEq/l 4. Uric Acid 7.5 86. A 2 year old diagnosed with Hirschsprung's disease is being interviewed by the nurse. During data collection, the parents described the child's stools as "strange". Which of the following stool types would most likely fit the parents description? 1. Light yellow, frothy and foul smelling 2. Currant jelly-like 3. Narrow and ribbon-like 4. Green liquid 87. A mother calls the clinic and asks to speak to the nurse regarding her 6 month old daughter who has been vomiting for the past 24 hours. The mother states that the baby is on Digoxin for a congenital heart defect and she is concerned that this may be a serious problem. What instructions should the nurse give to the mother over the phone? 1. Check the infant's pulse for a HR <100 BPM 2. Increase the infant's fluid intake 3. Check the infant for a wet diaper 4. Redose the digoxin 88. Which of these statements made to the nurse by a 9 year old with acute appendicitis would require immediate action? 1. "I am afraid to have surgery." 2. "I feel hot and thirsty." 3. "I feel better with my legs up towards my chest." 4. "My pain has gone away."

47

89. A nurse assigned to a child with Acute Glomerulonephritis is picking up doctor's orders to put in the Kardex. Which of the orders should the nurse question? 1. Bed rest 2. Daily weights 3. Daily blood pressure 4. Strict I & O 90. A 4 year old with Celiac Disease is in the hospital with an exacerbation of Celiac Crisis due to improper dietary intake. When teaching the mother the dietary restrictions for her child, which of the following foods must be completely eliminated from the child's diet? 1. Whole milk, ice cream and cheese 2. Rice, corn and soybeans 3. Bread, oatmeal and pretzels 4. Beef, liver and veal

Answers-Section Three 70. 1. Choice 1 is the best choice. Constipation is a very common anti-cholinergic side effect of tricyclic antidepressant. These side effects can be effectively treated with proper diet, fluids and exercise. Choice 2 Routine blood levels need to be drawn on patient's taking Lithium, to maintain therapeutic blood levels; Choice 3 - Patients on Phenothiazines must stay out of the sun to prevent photosensitivity reaction; Choice 4 - Cheese and wine need to be avoided for patients on MAO inhibitors, to prevent a hypertensive crisis. 71. 2. This is the best choice which describes M.S., and it is important for the nurse to be able to distinguish between these different neurological disorders in order to plan care. Choice (1) describes Parkinson's Disease, Choice (3) refers to ALS (Amyotrophic Lateral Sclerosis), and Choice (4) refers to Myasthenia Gravis. 72. 4. Choices (1), (2) and (3) would be appropriate interventions for such a client. Bell's Palsy is a disorder involving Cranial Nerve VII, and results in facial paralysis or paresis, patients may experience a variety of symptoms including pain, drooping of the mouth on the affected side, and an inability to close the eye on the affected side. Choice (4) would NOT be appropriate, as vigorous massage could further increase pain, etc. Gentle massage, and the application of heat are preferred interventions. 73. 3. ALS is a progressive and usually fatal disease which affects the motor neurons; as they die, muscular function is affected resulting in progressive weakness, atrophy, spasticity of the muscles including the muscles of respiration. While Choices (1), (2) and (4) are all potential diagnoses, choice (3) would be the diagnosis with the highest priority. REMEMBER A PATENT AIRWAY IS ESSENTIAL TO LIFE, AND MUST BE MAINTAINED. 74. 3. Oxygen is a drug, and must be administered carefully. Clients with COPD, have adapted to breathing with lower 02 concentrations. Administering 02 at liter flow greater than 2-4L min, can result in the absence of the patient's drive (hypoxic drive) to breathe. This will cause further C02 retention (CO2 Narcosis), and ultimately respiratory depression. ABG's will also help to determine the liter flow. The nurse should explain this to the patient. Therefore, Choice (1) would NOT be an appropriate intervention. Choice (2) would be correct but the reason is incorrect, and Choice (4) you should let the M.D. know, but this question asks for a first nursing response. 75. 4. Clients who have been on anti-TB drug regimes for at least 2-3 weeks and have absence of AFB in at least two successive sputum cultures, no longer need to be on Respiratory Isolation. Taking medication alone, or the absence of adventitious breath sounds such as rhonchi, rales, etc, or the absence of infiltrates on chest x-ray, usually seen with Pneumonia would not be a reason to D/C Isolation, making choices (1), (2), and (3) incorrect. 76. 2. Glycosylated Hemoglobin (Hgb) is a lab value which reflects glucose combining with Hgb and attaching to the red blood cells (RBC's) for the life of RBC. This test, therefore, is indicative of control of blood sugar, regardless of increase or decreases in serum glucose values. A value of 10% or less is considered good control. Choices (1) and (3) are incorrect, although (4) might seem likely and may be true, it is NOT correct in this instance. 77. 3. As rales are abnormal lung sounds, which are described as crackling in nature. Choice (1), rhonchi, are also adventitious, but are characterized by sonorous, dry coarse sounds which may clear if patient coughs. Sometimes called gurgles. Choice (2), wheezes, are high pitched sounds which are continuous, and can be auscultated during both inspiration and expiration, secondary to narrowed air passages. Choice (4), refers to either incomplete expansion or collapse of air passages but is not in and of itself a breath sound. 78. 4. Beta blockers act to decrease heart rate and force of contraction and reduce vasoconstriction by antagonizing Beta receptors in the myocardium and vasculature. Choices (1) and (2) refer to the action of nitrates as well as Calcium Channel Blockers such as Diltiazem. Choice (3) is not applicable. 79. 3. Loop diuretics such as Lasix result in potent diuresis. The most common side effects are electrolyte imbalances such as hypokalemia, hyponatremia making choices (1), (2) and (4) incorrect. In addition, Digoxin taken in combination with loop diuretics can result in digitalis toxicity, so the nurse should be alert to this and normal serum lab values for Digoxin (Norm: 0.5-2.0 ng/ml). The nurse should be aware that there is a very small variance between therapeutic and toxic levels of this drug.

48

80. 4. 3 degree A-V block is a conduction defect with A-V Junction that impairs conduction of atrial impulses to ventricular pathways. It is the highest degree of A-V Block and is characterized on EKG by choice (1), (2) and (3). Choice (4) is NOT an EKG finding, as in 3 degree A-V Block. The heart rate is slowed to 40-60 bpm, or inherent ventricular rate. 81. 1. A patient in 3 degree A-V Block has all sinus impulses blocked and abnormal ventricular conduction, as well as a heart rate of 40-60 bpm or less and will therefore need support in the form of a pacemaker and/or use of meds such as atropine or Isoproterenol (Isuprel). Choice (2) Digoxin, a cardiac glycoside would only further decrease heart rate; Choice (3) Lidocaine is used in treatment of PVC's; Choice (4) may seem a possible option, however, with proper medical management a heart rate sufficient to support perfusion can be maintained making CPR unnecessary. 82. 2. Demand Pacemakers will fire ONLY when the heart's spontaneous beat falls below the minimum rate. Electrodes for sensing and pacing are placed in the ventricles. Choice (1) refers to an Asynchronic Pacemaker which is unaffected by heart regular beat; Choice (3) refers to Synchronous and A-V Sequential type Pacemakers, however, it is the sensing electrode ONLY and both sensing and pacing electrodes are placed in the atria in the A-V mode. Choice (4), a Pacemaker may be a temporary modality, but can also be permanently implanted, making this choice incorrect. 83. 3. A client should NOT repeat a dose if first dose is vomited, as one would not know how much of the original dose was absorbed, and could possibly lead to excess Digoxin levels, which can cause arrhythmias, or slow heart rate below 60 bpm. Choices (1), (2) and (4) would be included in a teaching plan. 84. 1. Following cardiac catheterization, the nurse must make several crucial evaluations v.s. must be monitored and peripheral pulses (distal to insertion site) must be checked q 15 min x 1 hr. Then, if stable, re-assess q 1/2-1hr till stable. In addition, the site must be checked for hematoma formation and/or bleeding. The color, temperature, etc. of the extremities must also be assessed, as well as any unusual pain, numbness, or tingling in the extremity. The nurse should also be alert for any dysrhythmias, especially bradycardia (a vasovagal response to the procedure), chest pain, oliguria, etc. are ALL potential complaints or observations that require IMMEDIATE intervention by the nurse. Mild pain at the insertion site can be expected following this procedure and analgesics are administered p.r.n. However, any unusual pain should be assessed and reported and would require IMMEDIATE INTERVENTION. 85. 1. Although BUN is a measure of kidney function, patients who are dehydrated (without kidney disease) can show an elevation in Blood Urea Nitrogen (BUN). Creatinine is a specific indicator of renal function and/or failure. 86. 3. This finding is consistent with Hirschsprung's disease. Due to the aganglionic portion that is very narrow and failure of the internal anal sphincter to relax the stool that passes is very narrow and may appear to look like a long ribbon. Option 1 is Foul smelling stool = fat in stool, option 2 is currant jelly stool = circulation problems, option 4 is liquid stool = malabsorption. 87. 1. Signs of Digoxin Toxicity in infants often presents with vomiting since increased drug levels stimulate the emetic control center in the medulla. Digoxin in toxic levels significantly suppress the SA node and cause slowing of the heart rate, therefore having the mother assess the child's pulse will help to determine if the child is exhibiting a toxic reaction to Digoxin. Increasing fluid intake is inappropriate until it is determined whether or not the child is toxic. Checking for wet diapers would indicate renal response to digoxin, not toxicity. Redosing the digoxin is inappropriate until toxicity is ruled out. 88. 4. The classic finding when an appendix ruptures is a sudden cessation of pain. A ruptured appendix requires immediate intervention to prevent serious complications. Options 1, 2 and 3 are expected findings for a child of this age who is diagnosed with acute appendicitis. 89. 3. Blood pressure elevation is a serious and frequent complication associated with Acute Glomerulonephritis. The nurse should expect to assess blood pressure every 4 to 6 hours with vital signs. Options 1, 2 and 4 are appropriate orders for a child with Acute Glomerulonephritis. 90. 3. Celiac disease is caused by an intolerance to gluten, which is a protein found in wheat, oats, barley and rye, All the foods in option 3 contain gluten. Option 1 would be eliminated if the child had a lactose intolerance, option 4 would be eliminated if the child had a fat intolerance.

Section FOUR 21. A breast feeding mother develops mastitis in the left breast and is put on an antibiotic for seven days. She asks the nurse if she can continue breast feeding. The nurse's best answer would be: 1. "Only breast feed from the right breast." 2. "Do not breast feed or stimulate the breasts until the infection is resolved." 3. "Continue breast feeding, this is not a contraindication." 4. "Pump the breasts and discard the milk until the infection resolves." 22. On a tour of the labor and delivery suite, a prospective couple asks the nurse when do you put the erythromycin ointment in the baby's eyes. The correct response would be: 1. "It is only done if the mother has a chlamydia infection at the time of delivery." 2. "It is only used if the baby has signs or symptoms of an eye infection." 3. "It is placed in the eyes immediately after the delivery." 4. "It is placed in the eyes after the parents have had a chance to hold the baby."

49

23. A woman has been in labor for 13 hours and is 4 cm dilated, 50% effaced. She is on a Pitocin drip. The nurse is aware to discontinue the Pitocin if: 1. The woman complains of pain 2. The contractions are 90 seconds in duration 3. The contractions are 3 minutes apart 4. The contractions are not causing cervical dilation 24. When teaching a 36 week primigravida signs of true labor, the nurse would be sure to include: 1. When she feels pain that radiates from the lower back to the abdomen 2. When the contractions are felt more in the abdomen 3. When there is loss of the mucous plug 4. When she experiences lightening 25. A woman who is 12 hours post partum has the following findings: Temperature = 100.4, Pulse = 94, Respirations = 20, BP = 100/60. An appropriate nursing intervention would be to: 1. Call the doctor 2. Get her up and ambulating 3. Offer her fluids to drink 4. Prepare her for a blood transfusion 26. A patient on a locked psychiatric unit asks the nurse if he may have a razor to shave. The nurse is aware that razors are considered sharps and therefore must be: 1. Used under the direct supervision of a nurse each time used 2. Only given to patients who are extremely responsible 3. Only given if a suicide assessment has been performed first 4. Signed out to responsible patients and signed in when returned 27. A patient being treated for schizophrenia is started on Thorazine 200mg qid. The doctor has ordered Cogentin for this patient. The nurse is aware that the Cogentin is given: 1. To decrease the incidence of seizures 2. To reduce side effects of the Thorazine 3. To potentiate the action of Thorazine 4. To improve and stabilize mood 28. A patient diagnosed with antisocial personality is admitted to the psychiatric unit. She is told that all meals are served in the dining room and patients are expected to be dressed in street clothes when they come in to eat. She complies with the rule for lunch and dinner but demands that she be served breakfast in bed. What action should the nurse take? 1. Bring the breakfast tray as requested 2. Warn her that her behavior will be reported to the doctor 3. Tell her that you will do it once and then she must promise to comply with the rules 4. Tell her that the rules apply to everyone

50

29. An important aspect of planning care for patients with a conversion disorder is the nurses awareness that: 1. Reality testing is often severely impaired 2. The symptoms are ego-syntonic 3. Secondary gains can interfere with achieving desired treatment goals 4. The disorder originates in problems occurring during toilet training 30. A patient with major depression has been placed on Parnate. The nurse teaches the patient the dietary restrictions required while taking an MAO inhibitor. The following statement made by the patient indicates the patient needs more teaching: 1. "I should avoid stimulants like caffeine and any drugs with epinephrine." 2. "I can't have smoked fish or pickled vegetables." 3. "I should eat fresh foods and keep my lunch refrigerated when I go to work." 4. "I should use over the counter cold remedies when I get sick." 31. A young woman has been seeing a therapist for about a year following the sudden death of her husband. Which behavior would indicate that the woman has achieved reorganization following this major crisis in her life? 1. She keeps all her dead husband's things in the same place they were and keeps a candle lit next to his picture every day 2. She feels comfortable talking about her husband and how they fell in love 3. She begins to explore work options and moving to a smaller place that she can afford on her income 4. She is no longer angry that her husband has left her 32. A patient is on special observation for signs of delirium from alcohol withdrawal. What physical changes would the nurse note first? 1. Hypotension and delusions 2. Temperature elevation and tremulousness 3. Runny nose and stomach cramps 4. Angina and confabulation 33. A patient is admitted to the Psychiatric Unit for chronic alcoholism. The treatment team diagnoses the patient with Wernicke's encephalopathy. The team is concerned that if this syndrome is left untreated, the patient will develop which one of the following irreversible syndrome with the cardinal symptoms of confusion and amnesia? 1. HIV encephalopathy 2. Delirium Tremens 3. Korsakoff's syndrome 4. Down's syndrome 34. An elderly man is admitted to the Geriatric Unit for his forgetfulness and severe behavioral changes. He is diagnosed with Alzheimer's Disease. Which of the following should be the primary goal of nursing intervention? 1. Keep him away from problems of daily living 2. Keep his capacity for self care activities to the optimum 3. Keep him isolated 4. Keep all available resources to increase his dependency

51

35. A patient on the psychiatric unit has been in an acute manic state for several days. She is very active, rarely sleeps and never appears to be hungry. The nurse's goal of highest priority based on these observations would be: 1. Slow the patient's rate and quantity of speech 2. Improve self esteem and self worth 3. Maintain optimal rest and hydration 4. Remain in areas with low stimulation 36. Which of the following is an INCORRECT statement regarding diet therapy for a patient in renal failure? 1. Limit dietary protein 2. Provide a diet high in carbohydrates 3. Limit Sodium (NA) intake 4. Provide a diet high in Potassium rich food 37. A patient with Chronic Renal Failure is being maintained on Peritoneal Dialysis. Which of the following is NOT an indication that the patient is developing possible Peritonitis? 1. Slightly blood tinged drainage after the first exchange 2. Rigid abdomen with abdominal pain 3. Decreased rate of fluid return 4. Nausea and vomiting 38. A patient with a bowel obstruction has a Salem Sump Tube (N/G) tube) in place. You are to irrigate this tube q. shift. Which of the following solutions should the nurse use? 1. Tap Water 2. D5%W 3. Sterile Water 4. Normal Saline 39. A patient has a Salem Sump Tube. When the nurse goes to irrigate the tube she notices that the gastric drainage is dark brown. Which of the following is the FIRST intervention the nurse should take upon noticing this? 1. Check the pH of the gastric contents 2. Perform a Hemoccult Test on the contents 3. Irrigate the tube and then check the returns 4. Remove the tube from suction 40. You are assigned to speak to a group of High School students about HIV and AIDS. In discussing transmission the nurse knows that the highest concentration of the HIV virus in infected patients is in the: 1. Saliva 2. Semen 3. Blood 4. Cerebrospinal Fluid

52

41. In teaching High School students about health practices that promote the prevention of spread of the HIV virus, the nurse should include which of the following: 1. Use a latex condom and water soluble lubricant during intercourse 2. Abstain from intercourse if the female is menstruating 3. Following oral intercourse, use an over-the-counter mouthwash so to destroy the HIV virus 4. Shower immediately with an antibacterial soap after intercourse, so to destroy the HIV virus 42. You are assigned to care for a patient with SIADH (Syndrome of Inappropriate Secretion of Antidiuretic Hormone). In developing a nursing care plan, which of the following needs would have the HIGHEST PRIORITY: 1. Oxygenation 2. Nutrition 3. Activity Intolerance 4. Safety 43. A patient is admitted to the Surgical Intensive Care Unit following a motorcycle accident in which severe head trauma was obtained. Which of the following signs would be indicative of increased intracranial pressure? 1. Increased pulse, increased respirations, increased BP 2. Increased pulse, decreased respirations, increased BP 3. Decreased pulse, decreased respirations, increased BP 4. Decreased pulse, decreased respirations, decreased BP 44. A patient is being monitored for signs of increased intracranial pressure. An Intraventricular catheter has been placed. Which of the following would indicate normal intracranial pressure? 1. An ICP of less than 20mmHg and CPP of 10mmHG 2. An ICP of 30mmHg and a CPP of 20mmHg 3. An ICP of more than 20mmHg and a CPP of 30mmHg 4. An ICP of less than 20mg and a CPP of 60mmHg or more 45. Of the following signs/symptoms, which is the EARLIEST sign of increasing ICP (Intracranial Pressure)? 1. Papilledema 2. Lethargy 3. Change in vital signs (ie: Increased BP) 4. Absence of reflexes

53

46. A homeless individual is brought to the Emergency Room after having been out in subfreezing temperatures for three to four days. The toes of the patient's right foot appear hard and cold with mottling, and are unresponsive to touch. Which of the following would NOT be included in the initial management of this patient by the Emergency Room nurse: 1. Rewarm the extremity with controlled and rapid rewarming until the injured part flushes 2. Wrap the affected extremity in a blanket and apply moist heat 3. Place sterile gauze between the affected digits 4. Elevate the affected extremity 47. In caring for a patient with a tracheostomy which of the following would be an INCORRECT ACTION by the nurse when providing tracheostomy care? 1. Checking the cuff pressure 2. Provide humidified oxygen 3. Remove the outer cannula for cleaning q. shift 4. Place sterile gauze between the outer wings of the tube before tying strings or tape to secure it 48. You are assigned to teach a nursing student how to suction an adult patient with a tracheostomy. Which of the following actions by the nursing student would be INCORRECT? 1. Pre-oxygenation of the patient with a Resusibag at 100% 02 several times before suctioning 2. Maintains wall suction pressure at 110-150mmHG 3. Does not suction for greater than 10-15 seconds at a time 4. Applies gentle intermittent pressure and rotates catheter during insertion phase of suctioning 49. Following a Vasectomy, a patient complains of pain and discomfort. Which of the following measures can be safely employed by the patient to aid in the relief of this discomfort? 1. Use an ice bag intermittently 2. Apply a heating pad to the scrotal area 3. Have the patient take a warm sitz bath 4. Give ASA grains X, q4h 50. Following a Vasectomy, a patient notices discoloration of the scrotal area and becomes alarmed. In responding to the patient's concern the nurse should: 1. Explain to the patient that this is a normal finding and usually responds to warm sitz baths 2. Note this is an abnormal finding and notify M.D. to see the patient 3. Know that this is not a likely occurrence, and is probably in the patient's imagination 4. Note that this is an indication of adequate blood flow to the area, and no intervention is necessary.

54

51. A woman who is one day post partum tells the nurse she doesn't have any milk yet. The nurse instructs her to: 1. Supplement the formula until the let down reflex occurs 2. Continue breast feeding 3. Place ice packs on her chest 4. Discontinue breast feeding and choose a commercially prepared formula 52. On a first prenatal visit, a woman has an alphafetoprotein test. The nurse is going to explain the reason for this test. The nurse would be sure to include: 1. The test detects Down's Syndrome 2. The test detects congenital heart defects 3. The test detects neural tube defects 4. The test detects the baby's sex 53. During the neonatal assessment, the nurse notes the baby has a cephalohematoma. Which type of delivery would be most likely to have caused this? 1. NSVD 2. C-Section 3. Vacuum extraction 4. Breech 54. A mother who has just delivered is a known cocaine abuser. During the neonatal assessment, the nurse would assess the baby for the following complications: 1. Drowsiness, respiratory depression, sluggish reflexes 2. Irritability, jitterness, tachycardia 3. Fever, irritability, difficulty breathing 4. Poor muscle tone, persistent peripheral cyanosis, sleepiness 55. A breast feeding mother has begun to complain that her nipples have become sore. The nurse would instruct her to: 1. Discontinue breast feeding until they improve 2. Put olive oil on her nipples 3. Utilize different positions for the baby during breast feeding 4. Pump the breasts and give the baby the expressed milk 56. A newborn is admitted to a special care nursery awaiting surgery for a myelomeningocele. Which of the following nursing diagnoses would be the priority in the plan of care for this baby? 1. Altered parenting 2. Altered skin integrity

3. Potential for infection 4. Potential altered elimination

57. The nurse has just admitted a 4 month old infant to the Recovery Room after a repair of a cleft lip. In transferring the infant from the stretcher to the bed, the nurse would position the infant in the following position: 1. Trendelenburg 2. Prone with the head turned to the right 3. Supine with head of bed elevated 30 degrees 4. Prone with head elevated slightly

55

58. The parents of a child with Tetralogy of Fallot have been given discharge instructions. Which of the following situations would the parents be instructed to avoid? 1. All infant contact with persons outside the home 2. Infant contact with persons who have mild colds 3. Infant contact with persons who have severe allergies 4. Routine immunizations 59. A nurse's teaching plan for a family with a child with Sickle Cell Anemia includes information on prevention of Sickle Cell Crisis. Which of the following situations would the nurse instruct avoiding in order to prevent a crisis? 1. Weight loss without dehydration 2. Midrange altitudes

3. Exposure to respiratory infections 4. Overhydration

60. A 14 year old girl has been hospitalized with Sickle Cell Anemia in vasoocclusive crisis. Which of these nursing diagnoses should receive priority in the nursing plan of care: 1. Impaired social interaction 2. Alteration in body image

3. Pain 4. Alteration in tissue perfusion

61. A patient with an Antisocial Disorder has a long history of arrests related to drugs and petty theft. This patient is admitted to the hospital for evaluation. The nursing staff should assume the following approach: 1. Authoritarian 2. Punitive

3. Unified 4. Flexible

62. A patient has been placed on Prozac [Fluoxetine] to treat a major depression. The nurse is aware that Prozac is an SSRI which is different from a tricyclic antidepressant. Two advantages of Prozac related to side effects would be: 1. Facilitates weight loss and doesn't potentiate seizures 2. Improves sleep and builds bone density 3. Strengthens immune system and improves sleep 4. Improves mood and stabilizes mood swings 63. A patient attends a medication class on Lithium. The following statement made by the patient indicates she needs additional medication teaching: 1. "I will need to have my Lithium blood level monitored." 2. "I should have salt in my diet." 3. "I should take a diuretic if I feel bloated. 4. "I should tell the nurse if I have diarrhea and vomiting." 64. A schizophrenic patient has been placed on Prolixin. He complains that he hates having to take medication because it reminds him that he's ill. The nurse reports this to the treatment team and recommends the doctor prescribe Prolixin Decanoate. The advantage of Prolixin Decanoate is: 1. The dose is minimal with few side effects 2. It is given by injection once every 1-2 weeks 3. It is given by injection once every 3-4 weeks 4. The dose can be given once a day

56

65. An elderly patient has been living in a nursing home for several years. The nursing staff has begun to notice a change in her behavior. All of the following are symptoms of depression except: 1. Changes in sleep patterns 2. Changes in eating patterns with weight loss 3. Excessive fatigue and increased concern with bodily functions 4. Hyperorality 66. A patient is brought to the Psychiatric Emergency Room by the police for threatening neighbors to blow up their car for stealing his secret formula. The patient is very agitated and paranoid. He is diagnosed with Paranoid Schizophrenia. The doctor orders Haldol for this patient over Thorazine because: 1. Haldol works faster in violent patients 2. Haldol has less side effects than Thorazine 3. Haldol is less sedating than Thorazine 4. Haldol is more sedating than Thorazine 67. A manic patient is exuberant and restless. He has been on medication for a few days but he continues to be somewhat agitated. The nurse recommends which of the following activities as most therapeutic at this time: 1. Playing a card game with the nurse 2. Volleyball 3. Jigsaw puzzle alone 4. Jazz dance group 68. A patient suffering from major Depression has done poorly on medication. The decision to give her ECT [electroconvulsive therapy] is made with her consent. All of the following are priorities in teaching her about this therapy to help her make an informed decision except: 1. ECT may cause death & memory loss 2. ECT may take several treatments

3. ECT may cause a manic episode 4. ECT involves having a seizure

69. A patient has been acting out on the Psychiatric Unit for days, being verbally abusive and threatening to staff. The following observation made by the nurse indicates he is improving: 1. The patient approaches each staff member and apologizes 2. The patient offers to help the staff clean up the living room 3. The patient begins to verbalize feelings 4. The patient states he feels better and is ready for a pass home 70. A young woman is admitted to the Psychiatric Unit. She had made a serious cut to her abdomen following her boyfriend threatening to terminate their relationship. Upon admission she began verbally abusing one nurse while being extremely sweet to a mental health worker. The patient's diagnosis is Borderline Personality Disorder. The primary defense mechanism used by this borderline patient with the nursing staff is: 1. Regression 2. Splitting

3. Inflation 4. Depression

57

71. A patient has Alzheimer's Disease and keeps the nurse in the room for extended periods of time while reminiscing about the past. Which of the following interventions by the nurse would be MOST therapeutic? 1. Keep the patient focused on the present and future only 2. Take the patient to group therapy with others in a similar age group 3. Set aside time in planning care in which you can let the patient reminisce about the past 4. Offer diversionary activities that will free you to do your work, and reduce patient's talking about past 72. Which of the following measures would be MOST EFFECTIVE to prevent disorientation in a minimally confused, elderly, hospitalized patient? 1. Orient to the room nightly prior to the patient going to sleep 2. Leave a small night light on in the room 3. Make rounds q1h to ensure that the patient is safe 4. Put the side rails up and call bell within reach 73. Following Gastric Resection, patients are prone to developing Dumping Syndrome. Which of the following types of dietary intake by the patient would be MOST helpful to either reduce or prevent this syndrome from developing? 1. Moderate fat, low carbohydrate 2. High fat, high carbohydrate 3. Low fat, low carbohydrate 4. Moderate fat, high carbohydrate 74. Following Total Gastrectomy patients will require vitamin replacement. Of the following, which vitamin is ESSENTIAL and MUST be given throughout life: 1. Vitamin C 2. Vitamin B6

3. Vitamin D 4. Vitamin B12

75. A patient with Pseudomonas is placed on Gentamycin Sulfate (Garamycin). Which of the following assessment parameters is NOT indicative of a potential serious adverse/toxic effect of this pharmacological agent? 1. Patient complains of decreased hearing or hearing loss 2. Elevation of serum BUN and/or Creatinine 3. Urine output of less than 600ml daily 4. Nausea and vomiting 76. You are taking care of a patient on Amphotericin-B. Which of the following electrolyte disturbances should the nurse be alert for: 1. Changes in Serum Potassium and Serum Sodium 2. Changes in Serum Potassium and Serum Magnesium 3. Changes in Serum Sodium and BUN 4. Changes in BUN and Creatinine

58

77. You are assigned to work in the Infectious Disease Clinic. A patient is diagnosed as having Second Stage Syphilis. Which of the following signs/symptoms is NOT a clinical manifestation of Second Stage Syphilis? 1. Low grade fever and malaise 2. Macular type rash

3. Chancre 4. Arthralgia

78. A patient presents to the Rheumatology Clinic where a diagnosis of Rheumatoid Arthritis is made. Which of the following would NOT be an assessment finding by the nurse: 1. Presence of Heberden's Nodes 2. Warm, tender, painful joints 3. Serum RF, and elevated ESR 4. Pt. c/o increased pain and stiffness in the morning 79. Following amputation of a lower extremity, a patient with prosthesis should be educated by the nurse to: 1. Wear the prothesis daily, but remove immediately when discomfort is experienced 2. Adjust the fit of the prosthesis by wearing a heavier sock to insure a tight fit 3. To put the prosthesis on immediately upon arising in the morning & keep it on all day 4. To apply oil or lotion to the stump before applying the prosthesis 80. You are assigned to care for a patient with a Below the Knee Amputation (BKA). Among the patient's orders is one which states that the patient should be placed in the prone position twice daily. The nurse knows that the reason for this is: 1. Changing the patient's position will help to prevent skin breakdown 2. To observe the stump for signs of infection 3. To assist the patient in doing ROM (Range of Motion) exercises 4. To stretch the flexor muscles and prevent flexion contractures 81. A young athlete injures his knee in football practice. The team doctor prescribes Ibuprofen (Advil) for pain, and to reduce inflammation. Following three weeks of therapy the patient develops petechiae, and a blood test reveals a Platelet Count of 9,000. The nurse can expect that: 1. The patient will be able to continue therapy with this medication 2. That the dosage will be reduced from 6 to 3 tablets daily 3. That the medication will be discontinued, and an alternative therapy will be prescribed 4. That the doctor will order repeat lab studies, as they are likely in error 82. You are caring for a patient who is s/p Colostomy. In preparing a teaching plan for this patient which of the following would be an INCORRECT statement? 1. Irrigation is not necessary since the fecal contents are liquid 2. That the stoma should be dark pink to red in appearance 3. That the bag should be checked when starting new medication to be sure that it is completely dissolved 4. That the bag/appliance should be changed q 2-3 days

59

83. You are caring for a patient receiving Hyperalimentation (Total Parenteral Nutrition). The flow rate ordered is 60cc/hr. After two hours the patient complains of feeling extremely nauseous, and of having a bad headache. Which of the following would be the MOST APPROPRIATE intervention by the nurse? 1. Stop the infusion immediately 2. Increase the flow rate as the patient is likely hypoglycemic 3. Decrease the flow rate and observe the patient 4. Check the patient's glucose level and urinary output 84. In caring for a patient with DVT (Deep Vein Thrombosis), which of the following nursing interventions would be INAPPROPRIATE? 1. Elevate the foot of the bed 2. Apply elastic stockings to both lower extremities 3. Apply warm, moist heat to the affected extremity 4. Teach patient to use a heel-toe gait when ambulating 85. You are providing care for a patient with Arterial Occlusive Disease, and writing a Nursing Care Plan. One of your interventions is to position the patient's legs below the level of the heart. From which of the following Nursing Diagnoses is the above intervention MOST LIKELY derived from: 1. Potential for Activity Intolerance 2. High risk for pain 3. Potential for Altered Respiratory Function 4. Altered Tissue Perfusion 86. A 4 year old with advanced HIV disease is anorexic from oral candidiasis. Administration of antifungal medications will help to resolve the Candida. Which of the following nursing interventions will help to promote optimal nutrition: 1. Offer favorite foods several times a day 2. Give foods in liquid form 4. Provide high calorie, protein meals and snacks 87. An 8 year old girl is admitted with R/O Acute Glomerulonephritis. Considering the usual prescribed treatment for this diagnosis which would be the earliest clinical manifestation of a response to treatment: 1. Decreased blood pressure 2. Increased urine output

3. Decreased edema 4. Increased serum protein

88. During a well baby check up, the nurse would expect the mother of a 7 month old to report that the baby: 1. Walks while holding on to things 2. Sits alone for brief periods 3. Pulls himself to a standing position in the crib 4. Just began to hold his head erect while sitting with support

60

89. A child in the Emergency Room is diagnosed with an acute episode of Croup (Acute laryngotracheo - bronchitis). During the initial assessment, which of the following finding would the nurse expect to find? 1. Diffuse expiratory wheezing 2. Inspiratory stridor with a brassy cough 3. Decreased aeration in lung fields 4. Shallow respirations 90. A 7 month old with congenital hip dysplasia had a spica cast applied. Which of the following instructions should be given to the parents in caring for their child. 1. Do not diaper the baby. 2. Feed only in prone position. 3. Observe the child's respiratory patterns. 4. Apply baby powder to the edges of the cast. 91. A woman delivered a set of twins 2 hours ago via C-Section and is now in the Recovery Room. The following fundal assessment findings would be expected: 1. Fundus at umbilicus, hard and midline 2. Fundus 1-2 finger breadths above umbilicus, hard and midline 3. Fundus 1-2 finger breadths below umbilicus, hard and midline 4. Fundus would not be assessed because of the C-Section 92. A woman is admitted to the Labor Room in preterm labor. She is 30 weeks pregnant and having contractions. Her cervix is 5 cm dilated, 50% effaced. The doctor orders Celestine (Betamethasone) to be administered stat. The patient asks why this drug is being given to her. The nurses' best explanation for this order would be: 1. To stop her contractions 2. To relieve pain 3. To increase maturity of the baby 4. To prevent infection. 93. A woman was admitted to the Labor Room in the active phase of labor. Which of the following assessment findings would confirm that she is in the active phase of labor? 1. Cervical dilation of 3cm, contractions lasting for 45-50 seconds with a frequency of 20 minutes apart 2. Cervical dilation of 4cm, contractions lasting for 60 sec. with a frequency of 10 minutes apart 3. Cervical dilation of 8cm, contractions lasting for 50 seconds with a frequency of 3 minutes apart 4. Cervical dilation of 6cm, contractions lasting for 45 seconds with a frequency of 3 minutes apart 94. The nurse is assessing a patient who is on a fetal monitor. On the last tracing, 2 late decelerations have occurred. The nurse is aware that late decelerations are: 1. Not worrisome and indicate head compression 2. Not worrisome and indicate cord compression 3. Worrisome and indicate uteroplacental insufficiency 4. Worrisome and indicate head compression

61

95. During the transitional phase of labor, the patient begins to scream and grabs the nurse with each contraction. The most appropriate nursing intervention would be to: 1. Look for the patient's coaching partner and get them to assist the patient 2. Establish eye contact and breathe with the patient 3. Give the patient pain medication 4. Tell the patient to begin pushing 96. The nurse is caring for a 9 month old in Bryant's Traction. When the nurse enters the room she observes that the baby is in the crib with the buttocks elevated slightly off the bed and the hips are flexed at a 90 degree angle. The appropriate nursing action to take would be to: 1. Call the Orthopedic Department to adjust the traction 2. Reposition the patient to the correct position 3. Chart the observation 4. Loosen the traction so that the buttocks rest on the bed 97. A toddler with Cystic Fibrosis is admitted with Pneumonia. Which of the findings in the child's history is directly related to the diagnosis of Cystic Fibrosis: 1. Developmental delay in walking 2. Tripling birth weight at one year 3. Meconium ileus 4. Two previous admissions for dehydration. 98. An 18 month old with Tetralogy of Fallot has a "tet" spell after having an invasive procedure. To improve the child's cardiac status which of the following interventions should the nurse do initially: 1. Place the child in a knee chest position 2. Begin chest compressions 3. Administer oxygen 4. Position with HOB elevated 99. A 2 1/2 year old who had a cleft palate repair is in the playroom and is crying because she wants the elbow restraints removed. The most appropriate action by the nurse to take would be to: 1. Distract the child with a toy 2. Remove the restraints and supervise the child 3. Remove the child from the playroom 4. Give the child pain medication 100. Which of the following actions taken by the mother of a 6 month old who is experiencing discomfort associated with teething, indicates that the nurse's teaching plan has been successful: 1. She gives the baby an ice cube to suck on 2. She reports that the baby was less irritable after placing an Aspirin against the erupting tooth 3. She continues to apply a solution made by her grandmother that contains whiskey and sugar on the baby's gums 4. She offers the baby a hard rubber toy to bite on

62

101. You are caring for a patient with a diagnosis of Sepsis. One of the goals you have set for your patient is that there will be adequate tissue perfusion within the first 24 hours. Which of the following assessment parameters would NOT be indicative that this goal is being met? 1. CVP of 8cm H2O 2. Urinary output of 30ml/hr or more 3. Heart rate of 106 bpm 4. Absence of mental confusion and/or lethargy 102. You are assisting a physician in removing a chest tube from a patient. Which of the following will the patient be asked to do when the physician is ready to remove the tube? 1. Exhale and hold breath, or bear down 2. Inhale and hold breath, or bear down 3. Breathe normally 4. Inhale and cough 103. Which of the following is an ABNORMAL finding when observing Water Sealed Chest Drainage for proper functioning? 1. Bubbling initially with coughing and deep inspiration 2. Continuous bubbling where the water seal is maintained 3. Water level fluctuations with breathing 4. A collection chamber that is less than 1/2 full 104. A 70 year old patient sustained a hip fracture and is placed in Buck's Traction while awaiting a surgical fixation. Of the following, which would be the PRIORITY intervention in providing care for this patient? 1. Turn and change the patient's position q2h 2. Check traction ropes, weights and pulleys q shift 3. Assess neurological/sensory and circulatory status q 2 h 4. Release traction intermittently 105. Following Total Hip Replacement, the nurse should position the patient: 1. Recumbent with the affected extremity in abduction 2. Recumbent with the affected extremity in adduction 3. Recumbent on unoperated side with affected leg straight 4. Recumbent on operated side with unaffected leg at 45 degrees 106. In caring for a patient with Total Hip Replacement the nurse must assess for signs and symptoms of possible joint dislocation. Which of the following symptoms would NOT indicate a possible/probable dislocation of the hip joint: 1. Severe hip pain 2. Inability to move affected extremity 3. Shortening of the extremity 4. Positive Babinski Reflex

63

107. A patient with Insulin Dependent Diabetes comes to the doctor's office with a chief complaint of fever, a "bad cold" and flu like symptoms. On exam her Temperature is 101.2, Pulse 96 and Respirations 16. Her WBC (White blood count) is 15,000/mm3. Which of the following can the nurse expect in regard to the patient's Insulin dosage: 1. Insulin should be withheld 2. The dose should be decreased 3. No change is necessary 4. The dose should be increased 108. A patient with Insulin Dependent Diabetes (IDDM) is admitted to the hospital following a three day history of productive cough, fever and chills. A diagnosis of Pneumonia is made. VS on admission are Temperature 103.2, Pulse 112, Respirations 32 and are deep and rapid. The nurse's FIRST ACTION should be to: 1. Administer oxygen at 2L/min via nasal cannula 2. Obtain a blood sample for Glucose and Acetone 3. Administer 5U of Regular Insulin 4. Give orange juice with sugar packets added 109. You are changing a dressing on a patient who is s/p Nephrectomy. When assisting the doctor in changing the dressing, you observe the drainage to be thin and light red in color. When documenting this, the nurse would describe the drainage as: 1. Serous 2. Sanguinous 3. Serosanguinous 4. Purulent 110. You are caring for a patient with a diagnosis of Meniere's Disease. Which of the following would NOT be an appropriate nursing intervention for this patient? 1. Restrict salt intake 2. Give vasodilating drugs such as Priscoline or Banthine 3. Give diuretics 4. Increase carbohydrate intake 111. Which of the following classifications of drugs WOULD NOT be utilized in the treatment of Asthma? 1. Calcium Channel Agonists 2. Beta Agonists 3. Methylxanthines 4. Corticosteroids 112. A patient is receiving Prednisone, a corticosteroid. Which of the following possible changes in lab values should the nurse be alert for? 1. Increased K+, decreased Ca++, increased glucose 2. Decreased K+, increased Ca++, decreased glucose 3. Decreased K+, decreased Ca++, increased glucose 4. Increased K+, increased Ca++, decreased glucose

64

113. A patient has received thromboembolytic therapy following a Myocardial Infarction with Streoptokinase. Which of the following drugs should the nurse have on hand if the patient develops excessive bleeding or hemorrhage? 1. Protamine Sulfate 2. Aminocaproic Acid (Amicar) 3. Vitamin K 4. Heparin 114. A patient is receiving Incentive Spirometry post-operatively. Which of the following would demonstrate misunderstanding on the part of the nurse regarding this treatment modality? 1. The patient should be medicated for pain, p.r.n. prior to beginning the treatment 2. The head of the bed should be elevated to at least 45 degrees 3. The therapy should begin on the second or third post-op day 4. The patient should be taught to hold their breath following inspiration, and then to exhale slowly 115. You are working in the Dermatology Clinic. A patient has a Basal Carcinoma In Situ removed from his left lower leg. When he returns for follow-up in one week, you note that the wound has healed with minimal scarring. Which type of healing process does this represent? 1. Primary intention 2. Secondary intention 3. Tertiary intention 4. Dehiscence 116. A mother has requested that Hepatitis B vaccine be given to her newborn, because all her other children have been vaccinated. The most appropriate response by the nurse would be to: 1. Tell the mother it will be administered 2. Explain to the mother that it is not given until the two month check-up 3. Explain to the mother that Hepatitis B vaccine is not given to infants 4. Tell the mother to ask her Pediatrician what his/her opinion is 117. A woman in labor begins to have variable decelerations on the fetal monitor tracing. The nurse's most appropriate intervention would be to: 1. Decrease IV fluids 2. Increase IV fluids 3. Increase Pitocin 4. Prepare the patient for a C-Section 118. A woman who is 31 weeks pregnant is having contractions that are 5 minutes apart. She has not ruptured membranes. Which of the following interventions should be started first? 1. Give IV fluids 2. Give Ritodrine (Yutopar) to cause uterine muscle relaxation 3. Place her on a fetal monitor 4. Place her in a left lateral recumbent position

65

119. A woman with placenta previa is being discharged from her prenatal visit. Which of the following instructions should the nurse be sure to include for her? 1. Use condoms during coitus 2. Coitus is contraindicated 3. Only use the missionary position during coitus 4. During coitus do not use any position that puts pressure on the uterus 120. A pregnant woman who is HIV+ is being told about a new treatment to decrease the incidence of transmission of HIV to the baby. The nurse would explain that AZT is: 1. Given only to the mother during the third trimester of the pregnancy 2. Given only to the baby after birth 3. Given in the third trimester, during the delivery and to the baby for the first 6 weeks after birth 4. Given during the entire pregnancy and during the delivery

Answers-Section FOUR 21. 4. Unless the antibiotic is safe for the infant, it is best not to feed the baby, but in order to continue establishment of milk flow, the breasts must be stimulated. 22. 4. The medication may irritate the baby's eyes, thereby, the bonding process should be initiated before the medication is instilled in the eyes. 23. 2. One of the toxic effects of Pitocin is uterine tetany. Contractions that last more than 60 seconds may indicate uterine tetany. 24. 1. Choice 2 is an indication of false labor; Choices 3 and 4, are signs of approaching labor. 25. 3. The nurse should understand that these findings are signs and symptoms of mild dehydration. 26. 4. In the therapeutic milieu, the nurse is responsible for maintaining safety of all patients. Utilizing a sign out and sign in sheet for razors keeps track of who has the razor, when it was taken and assures that it is returned. Sharps count at the end and beginning of each shift provides documentation that the items deemed sharps or dangerous are accounted for. 27. 2. Thorazine blocks the neurotransmitter dopamine resulting in side effects that look like Parkinson's disease. Cogentin reduces side effects such as stiffness, pill rolling tremor, mask-like face and cogwheeling rigidity associated with the Thorazine. 28. 4. Antisocial patients believe that rules do not apply to them so they frequently get into trouble. The nurse reinforces the rules of the unit and helps the patient deal with delaying his/her need for immediate gratification of personal needs. 29. 3. The primary goal of a conversion disorder is to reduce intolerable anxiety. The secondary gain involves meeting dependency needs. When dependency needs are met, the individual is comfortable and may have no motivation to change behavior. 30. 4. Over the counter cold remedies often contain stimulants or other ingredients that would cause a hypertensive crisis. The patient must consult their doctor before taking ANY drug. 31. 3. Reorganization implies the individual has begun to move on with their life. This involves making life decisions and or beginning new relationships indicating they have accepted the loss and are coping effectively to begin living again. 32. 2. The cardinal signs that a patient withdrawing from alcohol may be going into delirium tremens are elevated temperature, elevated blood pressure, elevated pulse and tremulousness. 33. 3. Korsakoff's syndrome is a form of dementia secondary to the thiamine deficiency known as Wernicke's encephalopathy. Both the short and long term memory are affected leaving the patient very confused and having amnesia of many if not most events. 34. 2. Alzheimer's disease is a progressive disease that takes years until the patient is completely incapacitated. It is best to support the individuals self esteem by helping them maintain as much control of their life through independent functioning of self care and activities of daily living. 35. 3. It is essential that the patient receive fluids, calories and rest to avoid cardiac complications related to dehydration and exhaustion. 36. 4. Patients with renal failure should have a diet that provides (high biologic value) proteins rich foods such as eggs, dairy products and meats. These are necessary to maintain a positive nitrogen balance. Foods high in calories are also necessary, and sodium intake should be limited. Foods high in Potassium should be AVOIDED due to decreased ability of the kidney(s) to filter and excrete Potassium. 37. 1. One of the possible complications of Peritoneal Dialysis is Peritonitis. The signs and symptoms may include: nausea, vomiting, abdominal pain or tenderness or rigidity, and cloudy dialysate. Blood tinged or slightly bloody drainage may be observed during the first one to two exchanges, and is not considered to be abnormal unless seen after such, making choices (2), (3), and (4) possible signs of Peritonitis, but NOT choice (1). 38. 4. Since an isotonic solution must be used 0.9 or Normal Saline MUST be used in order to irrigate a Nasogastric Tube. Use of a solution that is not isotonic could result in the undue removal of sodium and acids etc. from the stomach, and lead to potential fluid and/or electrolyte imbalances in the patient. 39. 2. The presence of dark brown "coffee ground" drainage may indicate the presence of bleeding or blood in the GI tract. Because this maybe the case the nurse should FIRST perform a Hemoccult Test to determine this. Choice (1) checking the pH will only help to determine gastric acidity; Choice (3) and (4) would not be correct nursing actions in this case. 40. 3. The HIV virus has been found and isolated in all of the above body fluids, as well as in the stool and urine. However, the highest concentration is found in the blood of infected individuals.

66

41. 1. Although abstinence is still the best protection against spread of the HIV virus, the use of a latex condom with a H20 soluble lubricant is the most effective means. All other choices given have no proven validity against the spread of the HIV Virus. 42. 4. Although the patient with SIADH has many needs, the one with the highest priority is that of safety. The patient with SIADH is at risk for dilutional hyponatremia which may result in seizures. Therefore, of the diagnoses given Choice (4) would have the highest priority. Choice (1) - Oxygenation - is not a problem in these patients, and Choice (2), Nutrition, although dietary changes may be needed to manage fluid and electrolyte disturbances associated with this syndrome, it does not take priority over safety. 43. 3. Signs and symptoms that may indicate an increase in Intracranial Pressure may include headaches, visual changes, seizures, vomiting, stiff neck etc. VS are also an important indicator, as indicated by Choice (3) the triad of decreased pulse and respirations, and a rising BP, also called Cushing's Triad are very important indicators of increased intracranial pressure (ICP) and MUST be monitored carefully by the nurse. 44. 4. Maintaining an ICP of less than 20mmHg, and a CPP (Cerebral Perfusion Pressure), usually at least 60mmHG, or as ordered are considered to be within normal limits. Intracranial Pressure monitoring is done through the placement of a catheter in the ventricle of the brain. Normal ventricular pressure is 10mmHg. Pressures of 11-20mmHg are considered to be mildly elevated, and pressure above 20mmg is to be high, and a definitive sign of increased ICP. 45. 2. Changes in level of consciousness, and responsiveness are among the earliest signs, with lethargy being the earliest sign or indicator of increased ICP. The other choices are also associated with increasing/increased ICP, however they are usually later manifestations of such. 46. 2. This patient is suffering from frostbite, due to prolonged exposure to sub-freezing temperatures without proper protection. Frostbite is a condition in which there is trauma to the tissues without actual freezing of tissue fluids. Exposed areas of the body such as hands, feet, earlobes, etc. are all subject to this. The affected part becomes hard, cold, and is not sensitive to touch, and mottled bluish-white in color. The aim of nursing care is to restore normal temperature and circulation to the part. All other choices would be included in the management of such a patient. Choice (2) is incorrect in that wrapping of the part could further constrict circulation. If blankets are used, it should be supported with a foot cradle and not in direct contact with the extremity. Prolonged applications of heat should also be avoided due to the patient's diminished sensitivity, and could result in a burn if not carefully monitored. 47. 3. All choices, except choice (3) are essential parts of the nursing care of a patient with a Tracheostomy. In tracheostomy tubes with both an inner and outer cannula, it is ONLY the inner cannula which is removed for cleaning. Newer plastic tubes have disposable inner cannulas that are changed as ordered. 48. 4. In suctioning a patient with a Tracheostomy, the nurse should employ all of the above choices, except choice (4). However, when the catheter is inserted it should be done gently, and to a depth of 10-12.5cms (4-5"s) or until the patient begins to cough. Suction should never be applied when inserting the catheter, not should it be rotated during this period. Suction should be applied by occluding the Y-port with the thumb of the unsterile gloved hand, while the catheter is rotated gently during withdrawal. The patient should never be suctioned for more than 10 seconds at one time to avoid the development of hypoxia. 49. 1. Following a Vasectomy a patient may experience localized discomfort. The patient is advised to apply ice bags intermittently for several hours after the surgery, to the scrotal area, to relieve mild pain and discomfort. Choice (4) may also initially seem to be correct, however, analgesics are usually administered on a p.r.n. basis, vs. q4h. Choices (2) and (3) are not correct interventions for pain relief in this situation. 50. 1. Abnormal discoloration of the scrotal area following Vasectomy is not unusual, and usually responds well to warm sitz baths. The nurse should reassure the patient of such. However, ecchymoses, swelling, superficial wound infection of the vas-deferens or epididymitis, or sperm granulomas, are all potentially serious complications, and the M.D. should be notified of any unusual findings immediately. Therefore, choices (2), (3) and (4) are incorrect. 51. 2. The first three days will produce colostrum. The "let down reflex" usually occurs around the third post partum day and then milk will flow. 52. 3. Elevated AFP may indicate a neural tube defect. 53. 3. Pressure on the head during this type of delivery will likely create a cephalohematoma. 54. 2. SE of Cocaine include vasoconstriction, stimulation of neuro system. 55. 3. Changing the baby's position will distribute the pressure on the nipples more evenly and will make it less likely to cause soreness. 56. 2. Skin integrity is correct because the myelomeningocele is only covered by a thin membrane and damage to the membrane can cause severe spinal cord injury and infection. All of the other options are appropriate concerns for a child with this condition, but are not as life threatening. Potential problems never take priority over actual problems. 57. 3. This position in the immediate post operative period allows for constant observation of the airway and prevention of injury to the suture line. Options 1, 2 and 4 will risk injury to the suture line. 58. 2. Children with cardiac disease have lowered resistance to upper respiratory infections and should avoid any circumstances that may expose them to even mild infections. Option 1 is unrealistic and option 3 would not put the child at risk since allergies are not contagious, option 4 is wrong because children with cardiac conditions do receive routine immunizations. 59. 3. Avoiding known sources of respiratory infections decrease the risk of hypoxia, which will cause decreased oxygen tension and sickling of RBCS. Option 4 is related to CHF, Option (1), weight loss, with dehydration could contribute to a crisis. Midrange altitudes do not cause problems it is high range altitudes that cause problems. 60. 4. In a vasoocclusive crisis tissue perfusion to the vital organs is threatened. Vital organs require perfusion of blood and oxygen in order to perform their functions and maintain homeostasis. All the other options are important concerns when planning care for an adolescent with Sickle Cell Disease, but are not life threatening. 61. 3. The antisocial patient manipulates the environment to get needs met. The nursing staff must act in a unified and consistent manner to help the patient learn to follow rules and function within limits imposed on him, as it will be, when he lives in the community 62. 1. Tricyclic antidepressants tend to cause weight gain and potentiate seizures. 63. 3. Lithium is a salt and it is stored in the body fluids. Loss of body fluids by diuretics would make Lithium levels dangerously high. In addition, loss of body fluids through vomiting and/or diarrhea would also make the Lithium level dangerously high. 64. 2. Prolixin Decanoate is good for non-compliant patients as it's a time released medication lasting 1-2 weeks. 65. 4. Hyperorality is a symptom of Dementia associated with the 3rd stage of Alzheimer's disease. 66. 3. Haldol is good with paranoid patients because it is less sedating. Very sedating medications tend to increase suspicions of the patient that the staff are out to hurt or poison them.

67

67. 1. A focused game of cards with the nurse would be most therapeutic. This activity limits competition with others, stimulation from others and allows for consistent limit setting in the event the patient begins to escalate. 68. 3. It is highly unusual for ECT to cause a manic episode. 69. 3. Putting feelings into words is a sign of improvement for this patient who has shown his angry feelings only through verbal abuse and threats. 70. 2. Splitting is a primitive defense mechanism in which persons see themselves or others as all good or all bad, failing to integrate the positive or negative of the self or others into a cohesive whole. 71. 3. Patients with Alzheimer's Disease often focus a great deal on the past. It is important for the nurse to allow the patient to do this in order to maintain self-esteem and self-concept. The nurse should set aside a certain amount of time in providing daily care in which the patient is allowed to reminisce about the past. 72. 2. Elderly patients, even if only minimally confused, are at a higher risk for sustaining injuries, especially in unfamiliar surroundings. While other choices are potential interventions that the nurse could implement, choice (2) would allow the patient to better visualize the surroundings, delimiting possible accidents or falls. Orienting the patient, as well as checking the patient, and keeping side rails up (judgement has to be made if such a measure is necessary) are also important. Each patient must be assessed individually to determine which measure(s) should be employed. 73. 1. One of the more common problems following Gastric surgery is Dumping Syndrome. Dietary management is the key to reduce or prevent this potential problem from developing. The diet should contain moderate amounts of fat, as well as be low in carbohydrates, especially small molecular carbohydrates such as sucrose and glucose. These dietary modifications will result in decreased hypertonicity of the intestinal contents, and prevent osmotic pull of extracellular fluid into the intestinal area, lessening the possibility for Dumping Syndrome to develop. 74. 4. Following Total Gastrectomy the production of Intrinsic Factor is permanently destroyed. This is necessary (Intrinsic Factor) for the absorption of Vitamin B12 from the GI tract. As a result patients MUST receive Vitamin B12 by parenteral route throughout life, or a condition known as Pernicious Anemia will develop, and can prove to be fatal. Regular IM injections on a monthly basis of 100-200ug is the usual therapeutic dose. 75. 4. The nurse should be alert for the potentially serious adverse ototoxic and nephrotoxic effects this drug may cause. Signs and symptoms of such as those in Choices (1), (2) and (3), are especially important in assessing a patient on this therapy. Choice (4) - nausea and vomiting - are side effects, but are usually not considered to be adverse or toxic to the patient if they should occur. 76. 2. Amphotericin-B is an antifungal/antimycotic drug, and is the preferred drug of choice for the treatment of severe systemic mycotic infections. Side effects and adverse effects which the nurse must be alert for include fever, chills, nausea, vomiting, etc. Electrolyte disturbances especially Hypokalemia and Hypomagnesemia are frequently observed. The drug is also potentially nephrotoxic and therefore changes in BUN and Creatinine are also significant. However, the question specifically asks for ELECTROLYTE disturbances making Choice (2) correct. 77. 3. The chancre, Choice (3), is seen in the Primary Stage, the stage at which the disease is most infectious. These lesions and the area in which they present are usually related to the pattern of sexual activity. The Secondary Stage, from a few weeks to several months in duration, sees the treponeme infecting agent traveling to various body systems and organs resulting in the signs and symptoms listed in Choices (1), (2) and (4), as well as others. 78. 1. Rheumatoid Arthritis is a chronic inflammatory Connective Tissue Disease in which the synovial joints of the hands, wrist, knees, feet are usually affected. RA also has systemic effects as well. Choices (2), (3) and (4), are all potential assessment findings in a patient with this diagnosis. Heberden's Nodes, however are associated with Osteoarthritis, and appear as bony nodules on distal finger joints. 79. 3. The prosthesis should be worn as described in Choice (3), to prevent swelling of the stump. The use of lotions or creams should be avoided as they can result in excessive softening of the skin. They can also set up a potential area for bacteria formation and infection. The prosthesis should NEVER be adjusted by the patient ONLY by a trained professional, making the other options (1), (2), and (4) incorrect. 80. 4. In the post-op period following an amputation of a lower extremity, nursing interventions are aimed at preventing deformities, building and maintaining muscle strength, and mobilizing patient's joints. Placing the patient in the prone position twice daily is specifically aimed at stretching the flexor muscles, and preventing flexion contractures of the hip. The other choices (1), (2) and (3), are also important parts of the nursing care, but do not answer the question. 81. 3. Side effects of NSAIDS are GI upset, rash, easy bruisability, etc. More serious side effects and adverse responses to therapy can include purpura, petechiae and more serious blood dyscrasias, among them Thrombocytopenia (decreased platelet count). The lab value of 9,000 in this situation denotes severe thrombocytopenia, (Normal values 150,00-400,000). 82. 1. A colostomy is one form of bowel diversion created through surgical intervention. Choices (2), (3) and (4) are all proper statements concerning colostomy care, and should be included by the nurse in the teaching plan. However, Choice (1) refers to an Ileostomy, as the contents of such are liquid unlike the colostomy, and therefore irrigation is not required. 83. 4. Patients on TPN must be monitored closely, and the rate of the infusion maintained as ordered. If the rate is too fast, hyperosmolar diureses occurs, and the patient may complain of headache, nausea, chills and may have an elevated temperature. The rate of the infusion must NEVER be increased or decreased abruptly, nor should it be changed. In this situation, the nurse should suspect Hyperglycemia, and should immediately check the patient's glucose level through (Finger Stick), as well as urinary output, as diuresis is likely. Notification of the M.D. of the patient's symptoms, as well as your assessment findings are also part of the nursing interventions. 84. 2. Nursing care for patients with DVT include bed rest, elevation of the extremity, application of warm, moist heat, anticoagulation therapy, and elastic stockings only if edema is present once the patient resumes ambulation. For the patient on bed rest, elastic stockings (TEDS), are applied to the UNAFFECTED EXTREMITY ONLY. Note in elevating the legs NEVER use pillows under the knees and/or knee gatch the bed. Teaching the patient how to ambulate should also be included in your patient care. 85. 4. Patients with Arterial Occlusive Disease have decreased perfusion of tissues. By placing the patient in a dependent position, blood flow to the lower extremities is enhanced, and, therefore, the nursing diagnosis for this intervention would be Altered Tissue Perfusion. Choices (1), (2) and (3), do not relate to the given problem of Arterial Occlusive Disease. 86. 4. All options may help to increase a child's intake, but in order to promote optimal nutrition the nurse must provide foods that are high in calories and protein. 87. 2. When the inflammatory process begins to resolve renal function improves. Urinary output must increase before the blood pressure or edema decreases. Serum protein loss is not the problem in acute glomerulonephritis.

68

88. 2. At 6 months babies sit with support and by nine months they sit unassisted, therefore a child between this stage would be able to achieve sitting alone for brief period. This is a demonstration of cephalocaudal development. Option 1 is expected of a 12 month old, option 2 is expected of a 9 month old and option 4 is expected of a 3 month old. 89. 2. Croup is an upper airway obstruction and the signs and symptoms are because of difficulty getting air past the upper airway. Wheezing is found with Asthma, decreased aeration in lung fields is found with Pneumonia. Shallow respirations are unlikely, the child may exhibit retractions, but not shallow respirations. 90. 3. Spica casts are body casts and cover the trunk of the body. If the cast is too tight the child's respiratory effort would be compromised. Diapering babies in spica casts is indicated as the perineal area is exposed. There are no feeding restrictions with spica casts and applying baby powder is contraindicated as it may cake and cause skin breakdown. 91. 2. Choice 2 is the normal post partum fundus, at two hours, regardless of mode of delivery. 92. 3. Celestone prevents Respiratory Distress Syndrome in a premature neonate by stimulating surfactant production. 93. 4. Cervical dilation of 6cm, contractions lasting for 45 sec. with a frequency of 3 minutes apart is the active phase of labor. Choices 1 and 2 are the latent phase of labor; Choice 3 is the transitional phase of labor. 94. 3. Uteroplacental insufficiency reduces 02 to the fetus and causes fetal hypoxemia 95. 2. During this phase of labor, the patient may experience strong forceful contractions which will distract the patient from concentrating. The nurse can redirect the patient and reduce the stress of this time of labor. 96. 3. This is the correct positioning of Bryant's traction, When a nurse observes an appropriate treatment the nursing action to be taken is to document the findings. 97. 3. The increased viscosity of mucous in the GI tract is frequently responsible for development of meconium ileus in the newborn period. Developmental delay in walking is not directly related to Cystic Fibrosis, tripling birth weight is a normal milestone expected of all children. History of dehydration may or may not be related to Cystic Fibrosis. 98. 1. A "tet" spell is when the child is having difficulty meeting oxygen demands. The knee chest position reduces venous blood return from the lower extremities and increases vascular resistance to divert blood flow to the pulmonary artery. Option 2 is to be initiated for cardiac arrest. Options 3 and 4 would not help to oxygenate a child with TOF. 99. 2. Elbow restraints can be removed with supervision and children wearing restraints should have them removed every 2 hours to allow for ROM exercises. Developmental stimulation with play activities must be encouraged so the child should not be removed from the playroom. Distraction might work but it is not the optimal choice. Giving a child pain medication would be inappropriate since the child is not complaining of pain. 100. 4. Although all the above suggestions may make the baby feel less pain the only safe practice is to allow the child a hard rubber toy to bite on. The toy should be larger than the child's fist and easy to clean. An ice cube may be aspirated. Aspirin is contraindicated to be given to children and the action of aspirin is systemic not local. All alcoholic beverages are contraindicated to be given to children in any form. 101. 3. Patients with Sepsis are subject to coagulation problems and blood loss, which may also result in poor or inadequate tissue perfusion. Choices (1), (2) and (4) are all useful assessment parameters in determining if tissue perfusion is adequate. An increased heart rate above 100bpm is not an indication of adequate tissue perfusion. Trends and patterns in levels of consciousness, skin color and turgor, urinary output above 30ml/hr, BP, CVP, etc. are all important parameters to be observed for in a patient with Sepsis. 102. 1. When removing a chest tube the physician will ask the patient to exhale and hold their breath, or exhale and bear down. This will serve to increase intrathoracic pressure, as well as prevent air from entering the pleural space. 103. 2. Water sealed chest drainage is designed to remove air and/or fluid from the pleural cavity, and to restore negative pressure in the pleural cavity, which promotes the re-expansion of the lung. In observing the water-seal drainage system, the nurse can expect to see all of the above with the exception of Choice (2), as continuous bubbling can indicate a possible air leak in the system. 104. 3. While choice (1) and (2) are important parts of nursing care for the patient in this type of traction, Choice (3) would be the priority in order to detect complications, if any, early on. Use of a special mattress to prevent skin breakdown and checking the ropes, pulleys, etc. q shift, and traction tapes for tenderness or pressure, are also important nursing responsibilities. Traction should NEVER be released, nor change in tension or weight. 105. 1. Following Total Hip Replacement, the patient should be kept flat when in the recumbent position with the affected extremity placed in abduction. The nurse can accomplish this by placing a wedge or pillow between the patient's legs. Dislocation could result if the affected leg were to be placed in adduction. If positioning the patient on the unoperated side, it should only be to a maximum of 45 degrees, while maintaining the operated side in abduction. 106. 4. Choices (1), (2) and (3) are possible signs of a hip joint dislocation which can be the result of poor positioning (extreme adduction or flexion of affected joint), or infection. Choice (4) would NOT be associated with joint dislocation. A Babinski Reflex is movement of the big toe upward (dorsiflexed) vs. downward flexion, and fanning of the other toes. It is usually seen with upper motor neuron damage, as in patient's with stroke, etc. 107. 4. For Diabetics, certain conditions will cause a rise in the patient's blood sugar (hyperglycemia), among them is infection. As this patient has an elevated temperature, and elevated WBC of 15,000 (Normal is 5,000-10,000/mm3) indicative of an infectious process, the nurse can expect that the patient is likely to be hyperglycemic and the Insulin dose would need therefore, to be increased. Dosage will be determined by patient's usual dose and blood glucose value(s). 108. 2. In addition to infection, this patient is also demonstrating Kussmaul's Respirations, which are deep, rapid, blowing like respiration, associated with ketoacidosis in the diabetic. The deep, rapid respirations are the body's attempt to compensate for this acidotic state by blowing off Carbon Dioxide. The nurse should recognize these signs and symptoms as well as be aware that the patient will also have an increase in the number of circulating ketone bodies (a breakdown product from the metabolism of fat), contributing to the acidotic state. Therefore, blood should be obtained immediately for acetone and glucose, so that the body's hemeostatic state can be restored. Choices (1) and (3) may also be part of subsequent nursing interventions and choice (4) is unlikely. 109. 3. The nurse would term this type of drainage serosanguinous, composed of fluid (clear and watery) serous drainage, and cells that escape from blood vessels (red/red tinged). Choice (1) - Serous drainage is watery and clear in its appearance. Choice (2) Sanguinous drainage is blood like and bright red in appearance if fresh, and darker if the blood is old. Choice (4) purulent drainage refers to drainage which is made up of cellular debris, white blood cells and bacteria. 110. 4. Medications to suppress the vestibular system and other drugs such as those in choice (2) serve to improve tinnitus. Restricting dietary intake of sodium and adequate hydration have proved useful in some patients to diminish vertigo. Choice (4) has no proven place in the treatment of Meniere's. 111. 1. Choice (2) Beta Agonists, are usually the initial drugs used and serve to dilate bronchial smooth muscle. Methylxanthines are among the most commonly used bronchodilators and include Aminophylline and Theophylline. Corticosteroids, which may be

69

administered IV, or through inhalation help to reduce inflammation and reduce bronchospasm. Calcium Channel Blockers are not used in the treatment of Asthma. 112. 3. Among the most commonly lab values disturbances encountered with patients on long term Corticosteroid therapy are hypokalemia, hypocalcemia and hyperglycemia. The nurse should also be aware of the physical signs and symptoms associated with the above laboratory changes such as nausea, vomiting and muscle weakness, associated with hypokalemia, serum value of less than 3.5mg/dl, elevated BP, sodium and water retention, osteoporosis, especially in the elderly, and excessive thirst, hunger and/or urination associated with hyperglycemia. 113. 2. The major complication of thromboembolytic therapy is hemorrhage, and the antidote for this is Amicar, which aids in the stoppage of bleeding by inhibiting plasminogen, which inhibits thrombolysis. This drug should be available for any patient on this type of thromboembolytic therapy. Choices (1) and (3) are the antidotes for Heparin and Coumadin, respectively. Choice (4), Heparin, is not correct, as it is an anticoagulant. 114. 3. Incentive Spirometry is used post-operatively especially after thoracic and abdominal surgery to prevent collapse of the air passages or atelectasis. In assisting the patient, the nurse should employ choices (1), (2) and (4) and although it is more effective with the head of the bed elevated, it can be performed from any position. It should be started immediately as atelectasis can start as soon as one hour post-operatively. 115. 1. Surgical incisions, and small sutured incisions usually heal by what is called Primary Intention. In surgical incisions and the like where the edges of the wound can be approximated, healing takes place in this way. In larger wounds, when the edges cannot be approximated Secondary Intention is the usual manner of healing, the Tertiary intention occurs when there is an extended period of time between the wound and when it is sutured. 116. 1. Hep B is now a required immunization. The first dose is given at birth. 117. 2. Increasing IV fluids will address a possible hypovolemia and therefore increase fetal perfusion. 118. 3. Evaluating fetal well being is of highest priority and will direct subsequent interventions. 119. 2. Seminal fluid contains prostaglandins which causes cervical dilation. Intercourse may stimulate cervical dilation. 120. 3. This is protocol 076 and has been found to decrease transmission of HIV from 30% to 7%.

Section FIVE

1. A nurse is reinforcing self-care instructions with a patient who has non-insulin dependent diabetes mellitus (NIDDM). Which of these statements, if made by the patient, should indicate to the nurse that the instructions were understood?

a. “I will avoid watching flickering lights.” b. “I will eliminate raw foods from my diet.” c. “I will increase my medication when I feel shaky.” d. “I will inspect my feet daily.”

2. A nurse should reinforce which of these discharge instructions for a patient who has a

lumbar laminectory? a. ”Refrain from wearing restrictive clothing.” b. “Hold the weight close to your body when lifting.” c. “Do not sit for prolonged periods of time.” d. “Place a pillow at your back while in bed.”

3. A client on the oncology unit is refusing further treatment and wishes to sign out against-

medical-advise (AMA). The nurse calls the client’s doctor who tells the nurse to restrain, if necessary in order to keep the client in the hospital. The nurse refuses to accept the order knowing that to restrain this client would be: (a) Battery (b) Breach of confidentiality (c) Invasion of privacy (d) False imprisonment

70

4. A nurse is preparing to administer pilo-carpine hydrochloride (Pilocar) 1 gtt OD q 6h as prescribed for a patient. Which of these actions is correct?

a. Administer one drop on the cornea of both eyes. b. Place one drop in the conjuctival sac of the right eye. c. Drop the medication into the inner canthus of the left eye. d. Instill the medication onto the sclera of the affected eye.

5. A nursing home resident has been confined to a geriatric chair for two hours. Which of

these measures should the nurse take at this time? a. Give the resident a bed bath. b. Sit and talk with the resident for ten minutes. c. Walk the resident around the unit. d. Encourage the resident to socialize with his roommate.

6. A nurse is reinforcing self-care instructions with a patient who has been treated for

hepatitis type B. Which of these instructions should be emphasized? a. “You will have to take frequent rest periods.” b. “You will have to disinfect the toilet seat after each use.” c. “You will have to use disposable plates and utensils.” d. “You will have to take an antiviral medication for about a year.”

7. A resident persistently hoards pieces of string and other useless items. Which of these

actions should the nurse take? a. Tell the resident that the items are useless and take them away. b. Give the resident a container for the items so that they can be neatly stored. c. Remove the items from the resident, saying that they will be kept in a safe

place. d. Remove the items when the resident is not looking because they will be

forgotten. 8. A female patient is transferred from a nursing home to a psychiatric unit for treatment of

depression. When the nurse asks the patient to do anything, such a self grooming, the patient replies, “I can’t.” Which of these approaches would be most effective in dealing with the patient’s behavior?

a. Wait until she expresses willingness to act independently. b. Get a mirror and have her look at her untidy appearance. c. Go with her and assist as necessary. d. Tell her that a bath and food will improve her health and self-esteem.

9. A person who had a cerebrovascular accident (CVA) is admitted to a long-term care

facility. Which of these measures should the nurse include in the resident’s care plan to prevent development of foot drop?

a. Putting sheepskin foot protectors on the resident’s feet. b. Having the resident wear high-topped tennis shoes in bed. c. Propping the resident’s feet in an internally rotated position with sandbags. d. Placing trochanter rolls along the lateral margins of the resident’s thighs and

legs.

71

10. The daughter of an elderly resident who is confined to bed says to the nurse, “My mother’s bed was wet with urine when I came to see her today.” The patient has not previously been incontinent. Which of these actions should the nurse take in response to the complaint?

a. Get more information from the patient. b. Explain to the daughter that the nurses have been extremely busy. c. Notify the physician. d. Reduce the patient’s fluid intake.

11. A nurse should remind a patient who has had a cataract extraction to avoid which of

these activities? a. Drinking more than three cups of coffee a day. b. Bending over to put shoes on. c. Sleeping on a pillow. d. Watching television.

12. A resident with non-insulin dependent diabetes mellitus (NIDDM) is receiving

glyburide (Micronase). Which of these symptoms, if exhibited by the resident, should indicate to the nurse that the resident is experiencing a side effect of the medication?

a. Diaphoresis. b. Kussmaul’s respirations. c. Euphoria. d. Constricted pupils.

13. A patient who had had a cerebrovascular accident (CVA) has left-sided weakness. To

help maintain the patient’s affected bodily structures in functional position, the nurse should use which of these articles?

a. A bedboard under the patient’s mattress. b. A small pillow under the patient’s knee. c. A donut under the patient’s heel. d. A rolled washcloth in the palm of the patient’s left hand.

14. When counseling a sexually active teenage girl who is receiving isotretinoin (

Accutane) po for treatment of acne, which of the following instructions should be provided by the nurse?

a. “Consistently use birth control while on the medicine.” b. “Do not apply a spermicide during sex as long as you are on the

medicine.” c. “Do not use this medicine on the days you plan to have sex.” d. “Be prepared for a heavy menstrual flow during the course of treatment.”

15. A patient who has cirrhosis of the liver develops ascites. Which of these measures

would be important for the nurse to include in the patient’s plan of care? a. Measuring the abdominal girth. b. Encouraging foods high in protein. c. Keeping the legs elevated. d. Maintaining bed rest.

72

16. A nurse should recognize that a patient with acute renal failure is receiving hemodialysis for which of these purposes?

a. To improve kidney function. b. To increase urinary output. c. To remove nitrogenous waste. d. To enhance glomerular filtration.

17. A 16-year-old boy who has gonorrhea is most likely to have which of these

symptoms? a. A painless ulceration on the penis. b. A discharge from the urethra. c. Vesicular eruption on the perineal area. d. An erythematous macular rash on the inner aspects of the thighs.

18. An elderly resident who has congestive heart failure has an order for digoxin

(Lanoxin) elixir 0.125 mg to be administered via a nasogastric (NG) tube. The medication is available in 0.05 mg per mL. How many milliliters of the medication should the nurse administer?

a. 0.5 b. 2.5

c. 5.0 d. 6.25

18. The nurse is frequently asked about the probability that a premature baby will survive and be normal. What information is most accurate when counseling the obstetric client in preterm labor?

a. A baby born prematurely will not have as high an intellectual capacity as if he had been born full-term.

b. The earliest time at which an infant can be born with a prospect of survival is seven months gestation.

c. With each successive week after the fetus is viable, the chances of life become progressively greater.

d. A baby born before term is better able to perform some vital functions than those born at term.

19. Which equipment is found in a delivery room with a pre-eclamptic client?

a. Television with the volume turned high b. Bright overhead lights to observe papillary reaction c. Emergency cart in case of cardiac arrest d. Plastic airway

20. What is the physiologic effect of magnesium sulfate in the pre-eclamptic client?

a. Lowers the fetal heart rate b. Promotes hypofibrinogenemia c. Raises blood pressure by causing vasodilatation d. Controls convulsions by action on the central nervous system and the

myoneural junction

73

21. What complication is aquamephyton (Vitamin K) given to the neonate immediately after birth to prevent?

a. Hemorrhagic disease b. Hemolytic disease c. Ophthalmia neonatorum d. Erythropoiesis

22. Newborns are treated with an antibiotic ophthalmic ointment to prevent which condition?

a. Jaundice b. Ophthalmia neonatorum c. Erythema neonatorum d. Vitamin K deficiency

23. What historical data does the practical nurse obtain from the parents for the initial

care of the child who is admitted to the emergency department after a motor vehicle accident?

a. Grade in school b. Allergies to medications c. Weight gain d. History of previous fractures

24. The mother of a twelve year old who is admitted to the emergency department after a

motor vehicle accident reports that he has had three DPT immunizations to date. Which intervention is indicated?

a. DT 0.5 cc IM b. Tetanus Immune Globulin 2.0 cc IM c. DT and Tetanus Immune Globulin d. No immunization needed

25. The client admitted the emergency department after a motor vehicle accident is upset

and attempts to sit up on the treatment table. The nurse prevents this action by restraining him carefully. What is the rationale for this intervention?

a. Movement is contraindicated until X-rays of the cervical spine have been obtained and a fracture ruled out.

b. Sudden movement will further displace the client’s fractured femur. c. The client may become disoriented and fall off the treatment table. d. The client should lie down until his IV has been inserted.

26. In caring for the child with a head injury, which nursing intervention is the highest

priority? a. Take an oral temp b. Cleanse the laceration with betadine and apply Neosporin ointment c. Determine vital signs and level of consciousness d. Administer a sedative to promote relaxation

27. Which sign indicates a rise in intracranial pressure in the client with a head injury?

a. Elevated pulse rate b. Decreased pulse rate

c. Increased respiratory rate d. Pupils reactive to light

74

28. The ten year old boy is recovering from a motor vehicle accident. He is in traction for fracture of the femur. His mother asks the practical nurse what gift is most appropriate based on his age and physical status. What suggestion is most appropriate?

a. A basketball b. A Star Wars model c. A new bicycle d. New pajamas and a bathrobe

29. What is the priority in preoperative nursing care for the term infant born with an

omphalocele? a. Maintain moist sterile covering of the defect. b. Perform passive range of motion exercises to the lower extremities. c. Prevent dehydration by feeding small amounts of D5W. d. Relieve abdominal distention by keeping a nasogastric tube to low suction.

30. Two nursing assistants are talking about the client and another baby with

omphalocele they had cared for. The nurse asks them to get up and restock the formula cabinet instead of chatting. Why is this request made by the nurse?

a. The parents would have been distressed to overhear people talking about their baby.

b. There was work yet to finish before the shift was over. c. The baby and family rights to confidential treatment would have been

breached if other parents had heard the conversation. d. The client has a right to individual care and shouldn’t have been compared

with any other baby. 31. A 16-year-old unmarried woman who is in the third trimester of her first pregnancy

has been diagnosed as having mild pregnancy-induced hypertension (PIH). Which of these factors in her history placed her at risk for developing PIH?

a. Her age. b. Her parity. c. She is the eldest child in her family of origin. d. She is allergic to milk products.

32. Phenytoin sodium (Dilantin) is prescribed for a patient who has a seizure disorder.

The nurse should reinforce which of these instructions about a side effect of the medication?

a. “Protect your skin from sunburn.” b. “Drink plenty of fluids.” c. “Limit your intake of dairy products.” d. “Maintain a comprehensive oral hygiene program.”

75

33. A nurse is preparing to change a patient’s sterile dressing, and proceeds in the following way before putting on sterile gloves. Which action would contaminate a surface that should remain sterile?

a. The nurse opens the sterile dressing tray without touching the inner surface of the wrapper.

b. The nurse removes the indicator tape from a package of sterile 4X4’s and opens the first flap with a motion away from the nurse’s body.

c. The nurse spills sterile saline on the sterile field. d. The nurse handles the inside of the sterile gown when putting it on.

34. A patient who has cervical chlamydiasis is taking tetracycline hydrochloride

(Achromycin) and complains of gastric distress. The nurse should reinforce instructions for the patient to take the medication with

a. An antacid. b. Milk. c. Citrus juice. d. Non-dairy meals.

35. Which of these measures is most important for the nurse to include in the plan of care

for a patient who has Parkinson’s disease? a. Providing a cool environment for the patient. b. Encouraging the patient to breathe deeply and cough. c. Limiting physical activity for the patient. d. Increasing the amount of fiber in the patient’s diet.

36. A multipara says to the nurse, “I’m worried that my 3-year-old daughter is going to

be very jealous of this baby.” Which of these measures by the mother is likely to be most helpful to the 3-year-old daughter?

a. Having the daughter go to a day care center. b. Spending time with her daughter. c. Giving the daughter more responsibility for her own care. d. Encouraging the daughter to talk about why she feels neglected.

37. A nurse is reinforcing self-care instructions with a patient who has a hiatal hernia.

Which of these statements, if made by the patient, should indicate to the nurse that the instructions were understood?

a. “I will eat small, frequent meals.” b. “I will rest in a recumbent position after meals.” c. “I will drink several glasses of fluids with my meals.” d. “I will reduce carbohydrates in my meals.”

38. A patient who has thrombocytopenia has a very low platelet count. For the safety of

this patient, the nurse should include which of these measures in the patient’s care? a. Having the patient use a bacteriocidal soap for bathing. b. Observing the patient for signs of bleeding. c. Taking the patient’s apical and radial pulses at the same time. d. Encouraging the patient to eat foods high in iron.

76

39. A primigravida is in labor and her cervix is 8 cm dilated. The nurse enters the room and hears the woman say to her husband, “Don’t touch me. Leave me alone.” Which of these actions would be appropriate for the nurse to take in relation to the husband?

a. Encourage the husband to leave the wife’s room. b. Tell the husband to watch the nurse give care to his wife. c. Explain that his wife’s behavior is related to her stage in labor. d. Find out exactly what the husband did to elicit this response from his wife.

40. A nurse is reinforcing dietary instructions with a patient who has cirrhosis of the

liver. Which of these statements, if made by the patient, should indicate to the nurse that the instructions about the diet are understood?

a. “I have to include more foods containing carotene.” b. I need to increase fat intake in my diet.” c. I need to restrict the amount of calories I consume.” d. “I have to limit the amount of meat that I eat.”

41. A nurse is caring for a patient who has a nursing diagnosis of altered body image

related to modified right radical mastectomy. On the basis of this diagnosis, which of these actions by the nurse should be given priority?

a. Have the patient inspect her incision. b. Suggest that the patient explore the possibility of reconstructive

procedures. c. Encourage the patient to verbalize her feelings. d. Inform the patient that it takes time to adjust to the change.

42. A nurse should recognize that which of these measures would provide the greatest

relief of pain for the patient with rheumatoid arthritis? a. Providing a warm bath. b. Encouraging active range-of-motion exercise for affected joints. c. Maintaining bed rest. d. Massaging affected joints.

43. A child and his parents are instructed in the side effects of phenytoin sodium

(Dilantin), which include a. Decreased hearing. b. Drowsiness. c. Thinning hair. d. A metallic taste in the mouth.

44. An elderly man who is addicted to alcohol is being admitted to the hospital for

treatment after abstaining from alcohol for two days. He is given diazepam (Valium). Which of these behaviors suggest that the medication is having the desired effect?

a. He eats soup without spilling it. b. He puts his belongings away without complaining of muscle spasms. c. He says he is free of nausea. d. He says alcohol is ruining his life and has to quit.

77

45. A patient develops a neurogenic bladder as a result of a spinal cord injury. To initiate a bladder training program for the patient, the nurse should

a. Restrict fluids throughout the day. b. Compress the abdomen before each voiding. c. Pour water over the perineum. d. Observe for patterns of incontinence.

46. An elderly widow who lives alone comes to the clinic for immunizations prior to

taking a trip abroad. She tells the nurse that she is a volunteer in a children’s hospital where she cares for boarding babies for four hours every week. The nurse should recognize that the widow’s volunteer activity indicates that the widow is

a. Denying dependence b. Reliving “better days.” c. Gaining self-esteem. d. Demeaning adults who require assistance.

47. A resident who has had a cerebrovascular accident (CVA) can move the left arm and

leg voluntarily, but has left-sides weakness. Which of these behaviors will prepare the patient to participate in self-care?

a. Wearing a sling on the affected arm. b. Practicing sitting balance. c. Becoming tearful when frustrated. d. Ignoring the affected side.

48. A 13-year-old student tells the atepartal clinic nurse, “My period is a month late.”

Which of these questions should the nurse ask next? a. “Have you ever missed your period before?” b. “Are you using any protection when having sex?” c. “Are you ovulating?” d. “Have you gained any weight lately?”

49. A patient is scheduled for a radioactive iodine uptake scan (I131). Which of these

actions is essential for the nurse to take? a. Evaluate ability to swallow. b. Determine a history of hypertension. c. Check for allergies to shellfish. d. Note presence of skin lesions.

50. A patient who has chronic obstructive pulmonary disease (COPD) uses oxygen. The

nurse should recognize that which of these measures would be a safety hazard to the patient?

a. Having plants or cut flowers in the room. b. Wearing clothing made of 100 percent cotton. c. Using humidified oxygen d. Using oxygen at six liters per minute.

78

51. Which of these measures should the nurse take in order to ensure adequate nutrition for a resident with second stage Alzheimer’s disease who is restless and agitated?

a. Establishing set mealtimes. b. Offering small portions of finger foods frequently. c. Serving easily digested foods. d. Encouraging socialization during meals.

52. An elderly resident is receiving levothyroxine sodium (Synthroid) therapy. Which of

these symptoms, if reported by the resident, should indicate to the nurse that the prescribed dosage may need to be decreased?

a. Dry skin b. Weight gain.

c. Lethargy. d. Palpitations.

53. A nurse is reinforcing discharge teaching about taking a nonsteroidal anti-

inflammatory drug (NSAID) with a patient who has rheumatoid arthritis. The nurse should include which of these directions?

a. “Notify physician if fever occurs.”

b. Increase fluid intake.”

c. “Take with food.” d. “Monitor pulse rate.”

54. A nurse is reinforcing instructions about breast self-examination with a

premenopausal woman. Which of these statements, if made by the woman, would indicate that she needs further instructions?

a. “I will use a circular pattern to feel for abnormalities of my breasts.” b. “I will examine both of my breasts weekly while showering.” c. “I will look in a mirror for changes in contour of my breasts.” d. “I will gently compress the nipple of my breasts to check for discharge.”

55. A patient who has chronic obstructive pulmonary disease (COPD) is admitted to the

hospital. To prevent the patient’s secretions from becoming tenacious, which of these measures should be included in the care plan?

a. Encouraging the patient to do diaphragmatic breathing exercises tid. b. Encouraging the patient to drink approximately three liters of fluids every

day. c. Instructing the patient to walk the length of the hall bid. d. Instructing the patient to turn, breathe deeply, and cough every two hours.

56. A man is admitted to nursing home following a cerbrovascular accident (CVA). He is

ambulatory and able to talk, but he sometimes had difficulty understanding what is said to him. Which of these measures should the nurse try when helping him with morning care?

a. Showing him the implement needed when reminding him to carry out a task.

b. Asking him to think about what to do next when he is unable to proceed. c. Having him start again at the beginning when he helplessly repeats an

action in the middle of a task. d. Giving him a written outline of the steps to be taken and have him refer to

it as necessary.

79

57. The multigravida client tells the practical nurse that she has had short labor with her

previous delivery. She expresses concern about getting to the hospital before the baby is born because their home is 20 miles from the hospital. What anticipatory guidance is appropriate?

a. Appearance of show b. Dialing 911 for assistance

c. Rupture of membranes d. Timing of contractions

58. The practical nurse monitors the client during an oxytocin challenge test. During her

assessment the practical nurse observes that uterine contractions are lasting 90-95 seconds and the fetal heart rate slows during the contraction and returns to normal 30 seconds later. What is the practical nurse’s first action?

a. Notify the charge nurse at once. b. Check for sustained, sharp pain. c. Change the mother’s position. d. Administer oxygen to the mother.

59. What anticipatory guidance is appropriate for the obstetric client prior to an

ultrasound? a. Even though it is a very accurate test, it also has serious side effects. b. It measures the diameter of the fetal skull which facilitates estimating fetal

size in utero. c. Ultrasonography is an inaccurate index of fetal maturity. d. As an invasive procedure, ultrasonography may precipitate labor.

60. What is the practical nurse’s role during the administration of the oxytocin challenge

test? a. Administer analgesics for pain. b. Assist with amniotomy. c. Interpret the findings from the test. d. Perform a biophysical profile.

61. The practical nurse at a Poison Control Center receives a call from a mother whose

two year old has just drunk turpentine. What instruction does the nurse give to this mother?

a. Watch carefully for nausea and vomiting b. Give 1 oz. syrup of Ipecac with 8 oz. water c. Take the child to the emergency department immediately d. Give burnt toast and milk while driving to the emergency department

62. After successful open-heart surgery for a ventricular septal defect, which sign

indicates that a four year old child is recovering well? a. Increased apetite b. Interest in playing c. Tachycardia and tachypnea d. Talking to the doctors and nurses

80

63. An eight year old child is hit by a car while on his bicycle. At the hospital emergency department he is diagnosed with a fractured skull. Which sign indicates increased intracranial pressure?

a. A decrease in systolic blood pressure while diastolic remains normal b. Increased pulse and respiration c. Increased blood pressure, decreasing pulse and respiration d. Subnormal temperature and shallow tachypnea

64. What intervention is appropriate for the immediate nursing care of the client with a

cerebral vascular accident? a. Prevention of skin breakdown b. Prevention of contractures c. Development of alternative communication system d. Maintenance of life functions

65. Which nursing intervention is most effective in assisting the client with aphasia to

communicate? a. Provide lots of environmental stimuli to force communication b. Use signs made by the nurses and encourage visitors to talk with the client c. Encourage the client to respond as quickly as possible to questions d. Use yes or no questions and allow time for response

66. Which outcome criterion related to mobility for the client after a cerebral vascular

accident is appropriate in the nursing plan of care? a. Walk unassisted to bathroom/commode b. Perform basic daily living activities c. Keep affected arm in sling d. Bedrest until paralysis resolves

67. Which neurological symptom is an early warning sign of cerebral vascular accident?

a. Riniging in the ears b. Acetone breath c. Mental confusion d. Periorbital ecchymosis

68. During the acute phase of a cerebral vascular accident, what nursing care is most

appropriate? a. Stimulating the environment b. Antibiotic therapy c. Frequent neurological assessments d. Reduction of environmental stimuli

69. Which condition is common for the client after a cerebral vascular accident?

a. Hematuria b. Polyphagia c. Dysphagia d. Polyuria

81

70. Which positon does the practical nurse place the client in with a cerebral vascular accident for lumbar puncture?

a. Sitting on a stool with her elbows supported on the examination table in front of her

b. Lying flat on her abdomen c. Lying on her side with both legs extended d. Lying on her side with her back arched and knees and head drawn together

71. The physician prescribed meprobamate (Equanil) for the client with an anxiety

disorder. Which is an indication of drug effectiveness? a. Decreased nausea b. Decreased anxiety c. Regained use of her legs d. Inability to sleep

72. Which effect indicates that meprobamate (Equanil) is beginning to be effective in the

client with an anxiety disorder? a. Lack of seizures b. Sleep c. Increased restlessness d. Improvement in weakness in the arms

73. Which is an initial assessment in the client with hepatitis type A?

a. Tender liver b. Urine color c. Splenomegaly d. Swollen gums

74. The 43 year old client is seen in the doctor’s office with the chief complaint of

frequent stumbling and dropping items she is carrying. After further examination a diagnosis of multiple sclerosis is made. The practical nurse understands that multiple sclerosis is characterized by which pattern?

a. A steady downhill course which rapidly ends in death b. Gradual deterioration and progressive motor function loss, only c. Periods of remissions and exacerbations d. Only loss of motor and sensory functions in upper extremities

75. As the multiple sclerosis progresses, the client may develop the condition of

nystagmus. What is nystagmus? a. Bulging of the eyeball b. Double vision c. Involuntary movement of the eyes d. Deviation of the eyes to one side

76. Which is considered a presumptive sign of pregnancy?

a. Chadwick’s sign b. Positive pregnancy test

c. Hegar’s sign d. Urinary frequency

82

77. A nurse is reinforcing discharge instructions with a patient who is on warfarin sodium (Coumadin) therapy. Which of these instructions should be emphasized? (a) “Brush your teeth after meals with a firm toothbrush.” (b) “Use an electric razor for shaving.” (c) “Avoid use of Acetaminophen (Tylenol) while taking this medication.” (d) “Include green, leafy vegetables in your diet.”

78. Which physiological change occurs during the first trimester?

a. The uterus rounds out to globular form; the walls become softer and thicker

b. Relaxation of the sacroiliac joints and the symphosis pubis creates a “waddling gait”

c. Upward displacement of the diaphragm causing shortness of breath d. Milk production is complete

79. Which physiologic changes occur during the second trimester of pregnancy?

a. Continuation of “morning sickness” b. Blood volume increases to peak c. Linea nigra d. Lightening

80. A patient who is being treated for Parkinson’s disease makes all of the following

statements to the nurse. Which one would require further investigation? a. “I’m afraid I will not get better.” b. “My medications haven’t taken away all stiffness.” c. “I’m upset that I can’t smile anymore.” d. “Life is not worth living like this.”

81. Which of these actions should the nurse take when a patient is having a tonic-clonic

seizure? a. Monitor the patient’s vital signs. b. Protect the patient from injury. c. Place a plastic airway in the patient’s mouth. d. Obtain an order for a sedative for the patient.

82. When lighting a barbecue grill at a nurse’s picnic, an individual sustains a full-

thickness burn of the arm. Which of these emergency treatments by one of the nurses would be appropriate?

a. Pack the arm in ice. b. Apply petrolatum (Vaseline) to the arm. c. Cover the arm with a cool moist dressing. d. Place a tourniquet on the arm above the involved area.

83. The most effective way to emphasize strengths of a patient with low self-esteem is to

a. Provide tasks that the patient can complete successfully. b. Remind the patient of previous performance and encourage doing a little

better this time. c. Praise the patient whether or not stated goals are achieved. d. Suggest explanations for any of the patient’s failures.

83

84. A nurse is caring for a patient who requires pharyngeal suctioning. When performing this procedure, the nurse should apply suction when the

a. Catheter is inserted b. Catheter is being

withdrawn

c. Patient is inhaling d. Patient is exhaling

85. A patient is returned to the surgical unit after an appendectomy. Which of these

measures should the nurse give priority in the patient’s immediate postoperative care plan?

a. Increasing intake of oral fluids b. Assisting with ambulation c. Encouraging deep breathing and coughing d. Assessing the abdomen for bowel sounds

86. When a mother complains that her daughter has frequent urinary tract infections, the

nurse should suspect a. Child’s appetite has decreased b. Child wipes from front to back after toileting c. Child takes bubble baths d. Child’s panties are too loose

87. While an 18-month-old boy is in a mist tent, which of these toys would be most

appropriate for him? a. Crayons & coloring book. b. A wooden telephone.

c. Colored balloons. d. A mobile.

88. A nurse is reinforcing teaching with a patient who has acute lymphocytic leukemia

and who has a nursing diagnosis of “high risk for infection”. Which of these statements, if made by the patient, would indicate that instructions related to the nursing diagnosis were understood?

a. “I will avoid crowded areas.” b. “I will sleep with the window open.” c. “I will keep the room temperature at 80°F (26.7°C).” d. “I will include raw vegetables in my diet.”

89. A pregnancy test would be positive in the presence of which of these hormones?

a. Estrogen. b. Progesterone. c. Follicle stimulating hormone. d. Human chorionic gonadotropin hormone.

90. A nurse should reinforce which of these instructions with the family of a patient who

has a propulsive gait related to Parkinson’s disease? a. “Restrict the patient’s area of ambulation.” b. “Keep the floor in the patient’s surroundings free of obstacles.” c. “Assist the patient when walking.” d. “Install assistive railings in the hallways of the patient’s home.”

84

91. The answer to which of these questions would be most likely to reveal disorientation in a patient with primary dementia of Alzheimer type?

a. “What is your name?” b. “What is your birth date?” c. “What did you do for a living?” d. “What did you have for breakfast?”

92. A mother has been given instructions about the appropriate type of pet for her child

who is asthmatic. Which of these comments, if made by the mother, would indicate the need for further teaching?

a. “I plan on getting a fish tank filled with goldfish for my child.” b. “My child may have fun playing with the neighbor’s box turtle.” c. “I want to get my child a short-haired cat.” d. “My child may be interested in playing with an ant farm.”

93. A nurse is preparing to administer a soap solution enema to a patient who has not had

a bowel movement in three days. Which of these actions should the nurse include in the procedure?

a. Suspend the solution reservoir 36 inches (90cm) above the anus. b. Give the fluid slowly to prevent rapid distention of the colon. c. Insert the rectal tube 10 inches (25 cm) to promote evacuation. d. Instill the fluid at 98.6°F (37°) to prevent hypothermia.

94. A nurse is caring for a patient who had a right lower lobectomy and has a closed

(water-sealed) chest drainage system in place. It is essential for the nurse to a. Keep the system below the level of the patient’s waist. b. Change the system q 72h. c. Empty the system q 4h d. Clamp the system when ambulating the patient.

95. When a woman is four hours postpartum, the nurse observes that the woman’s uterine

fundus is soft. She has saturated three perineal pads during the past 15 minutes. The nurse should take which of these actions first?

a. Elevate the foot of the woman’s bed. b. Obtain the woman’s blood pressure. c. Massage the woman’s fundus. d. Apply gentle pressure to the woman’s fundus.

96. A patient who has a history of glaucoma has all of the following orders in preparation

for abdominal surgery. Which order should the nurse question?

a. Meperidine(Demerol) hydrochloride 75 mg IM on call. b. Teach deep breathing and coughing exercises. c. Soapsuds enemas till clear. d. Atropine sulfate 0.4 mg IM on call.

85

97. A nurse should be aware that adolescents are at highest risk for which of these injuries?

a. Motor vehicle accident. b. Gunshot wound. c. Drowning. d. Sexual assault.

98. Which of these measures should the nurse include in the plan of care for a patient

with hyperthyroidism? a. Maintaining the patient in semi-Fowler’s position b. Keeping the patient’s environment cool. c. Placing the patient on voice rest. d. Encouraging the patient to eat foods low in calories.

99. A child’s development is within the normal range for 6-month-old infants. The child

can be expected to demonstrate which of these behaviors? a. Picking up small objects using her thumb and index finger b. Crawling toward a wanted object. c. Waving bye-bye. d. Sitting up with support.

100. A client is newly diagnosed with bipolar mood disorder. At the present time he is in the manic phase. Which activity is most appropriate for the practical nurse to encourage the client to do?

a. Read a book of his own choosing. b. Go for a walk with a staff member c. Play checkers with another client d. Play a game of tennis with a staff member

101. Which drug may not be ordered for the client with bipolar mood disorder to shorten periods of elation and to prevent attacks of either depression or elation?

a. Meperidine b. Chlorpromazine

c. Lithium carbonate d. Diazepam

102. Considering the diagnosis of bipolar mood disorder, which assumption is correct?

a. If the client is in the manic phase, he will soon become depressed. b. Since this is the client’s first episode of manic-depressive illness, close

observation is important c. Since manic-depressive illness is always cyclic, he will experience

problems again in the future. d. The client will never be able to function normally again.

103. What is the most important action to prevent wound infection in the post-surgical client?

a. What signs and symptoms indicate wound infection? b. Swelling and tendernss c. Petechiae and maculopapule d. Crusting

86

104. What intervention is indicated for the client who is in isolation?

a. Apply restraints to keep the client’s hands away from the infected area b. Remove all non-esssential furniture from the room c. Allow the client to come out of the room every two hours d. Provide a staff member to have frequent contact with the client

105. What is the first manifestation of syphilis?

a. A small painless chancre b. Pustular skin lesions c. Painful and burning sensation during urination d. Granulomatous inflammation know as gumma

106. The client is distraught regarding her husband’s treatment towards her. She tells the nurse that she wants to be more assertive, but is fearful of her husband’s response. What is the most appropriate response to the client?

a. “I’m sure your husband would understand.” b. “Tell me more about how you think your husband would respond.” c. “He’s under enough stress now. Wait until he’s feeling better before you

talk with him about this.” d. “This really has nothing to do with him. You should only consider what is

right for you.” 107. What is the most frequently used method of observing uterine contractions during labor?

a. Ask the mother when she has a contraction and its severity b. Place a hand lightly on the mother’s abdomen c. Check the external fetal monitor print out for strength of contractions d. Perform cervical examination every two hours

108. In addition to careful observation and monitoring during the labor induction, what is the next most important nursing action?

a. Open the windows to provide adequate ventilation b. Provide reassurance and emotional support for the laboring couple c. Elevate the foot of the bed d. Prevent unnecessary persons from entering the room

109. The obstetric client’s fundus is boggy and lochial flow heavy, which intervention does the nurse perform first?

a. Elevate the foot of the bed b. Manually compress the placental site c. Massage the uterus and expel clots d. Check Victoria for signs of shock

87

110. After the client is given a preoperative medicine, which nursing action is appropriate?

a. Cover the client’s eyes with a small towel b. Offer the client an opportunity to ambulate to the bathroom c. Dim the room and raise the side rails d. Cover the client with an extra blanket

111. Which is the most important reason for the removal of dentures preoperatively?

a. Lost in surgery b. Broken during anesthesia c. Interfere with respirations d. Interfere with the placement of the endotracheal tube

112. Why is the side lying position recommended for the unconscious patient?

a. Use elastic stocking to facilitate venous return b. Promote passive exercise of the legs every hour c. Gently massage the legs to relieve minor cramps d. Encourage flexion and extension of the feet

113. What is the most important rationale for the nurse to change the position every ninety minutes in the post-operative client?

a. Improve the client’s morale b. Prevent the formation of decubiti c. Facilitate gastric mobility and drainage d. Promote adequate ventilation of the lungs

114. The client with a duodenal ulcer has a history of night pain. Which intervention is appropriate for the plan of care?

a. Advise the patient to give up smoking late in the evening b. Offer a mild pain reliever such as aspirin c. Give the client a light snack one hour prior to the usual onset of pain d. Teach the client relaxation techniques to decrease pain sensation

115. What sign is indicative of gonorrhea in the male client?

a. A swollen urethra and hematuria b. Open sores on his penis and a tender scrotum c. Frequent urination and a yellow urethral discharge d. A fine red rash on the hands and feet with fever and dizziness

116. The physician’s examination and lab studies confirm the diagnosis gonorrhea. What is the practical nurse’s responsibility in care of this client?

a. Assess the client’s knowledge that his sexual behavior is socially unacceptable

b. Report the case to the count Public Health Department c. Counsel the client regarding the situation if his parents find out d. Encourage the client to make new friends

88

117. Immediately after birth the airways are suctioned. Occlusion of the airway and respiratory distress syndrome (RDS) share which physical finding?

a. Neonatal hypoglycemia b. Susternal retractions and nasal flaring c. Neonatal respirations for 20-30 per minute d. Neonatal acrocyanosis

118. Which complication may the practical nurse expect in the infant of a diabetic mother?

a. Post-maturity b. Hyperbilirubinemia c. Hyperglycemia d. Hypoglycemia

119. Which nursing intervention is including the nursing care of an infant of a diabetic mother?

a. Monitor vital signs carefully b. Assess respiratory function frequently c. Maintain body temperature d. Measure frequent blood glucose assessments

120. The infant weighs 7 lbs., 6 oz. (3345 grams) at the well baby visit. Her birth weight was 5 lbs., 15 0z. (2690 Gm.) What does the practical nurse realize about the infant’s pattern of weight gain?

a. Normal rate b. Less than normal c. Greater than normal

d. Greater than the 10th percentile

121. Which suggestion might the practical nurse make to the infant’s mother to assist in the newborn’s feeding?

a. “Let her feed slowly by propping the bottle.” b. “Have your mother come over to feed your baby.” c. “Stop every ounce or so to let her rest and burp.” d. “Flick her feet to keep her awake.”

122. A nurse is reinforcing discharge instructions with a patient who is on warfarin sodium (Coumadin) therapy. Which of these instructions should be emphasized?

a. “Brush you teeth after meals with a firm toothbrush.” b. “Use an electric razor for shaving.” c. “Avoid use of acetaminophen (Tylenol) while taking this medication.” d. “Include green, leafy vegetables in your diet.”

123. Digoxin (Lanoxin) is prescribed for a patient who has congestive heart failure. Which of these symptoms should the nurse recognize as an early symptom of a toxic reaction to Lanoxin?

a. Ataxia b. Hematuria

c. Anorexia d. Tinnitus

89

124. A woman who is in labor is admitted to the hospital. Shortly after admission, the woman’s membranes rupture. Which of these actions is it essential for the nurse to take first?

a. Assess the fetal heart rate b. Obtain the woman’s blood pressure c. Change the wet linen d. Notify the physician

125. A nurse is caring for a patient who just had a thyroidectomy. Which of these observations would be most important for the nurse to make?

a. Monitoring the patient’s urinary output b. Checking the patient for weight gain c. Evaluating the patient’s ability to extend the neck d. Observing the patient for twitching muscles

126. When collecting data about an 8-month-old infant with otitis media, the nurse should expect to observe

a. Pulling of the affected ear. b. An increase in tearing c. Edema of the pinna d. A preference to lie on the abdomen

127. What is the most common feeding interval for the newborn infant?

a. Q 2 hrs. b. Q 3 hrs.

c. Q.i.d. d. Q 8 hrs.

128. Which assessment may indicate to the practical nurse that a twelve month old child may have otitis media?

a. Spike in the temperature of 101 or above b. Pulling his ear c. Reddish drainage from his ear d. Whining or moaning loudly

129. Which site is most appropriate for the injection of penicillin in the twelve month old child?

a. Upper arm b. Hip c. Lateral thigh d. Upper anterior thigh

130. The twelve month old child sees the injection coming and cries and kicks in protest. Which nursing action is most effective for giving the injection?

a. Proceed with the injection firmly and quickly, giving the child a hug when finished

b. Stop and give him a short explanation about why he needs the injection c. Let him up from the examining table and leave him alone to “cool off”

when the injection is over d. Give him the injection and tell him to stop crying with a firm voice

90

131. What is the most important education for the nurse to provide the mother of a child with bilateral myringotomy with tubes?

a. Proper placement of Q-TIPS for cleansing ear b. Signs and symptoms of meningitis c. Prevention of water in ear d. Prevention of colds

132. A nursing home resident complains of feeling cold and asks for a heating pad. The nurse should tell the resident that heating pads are not allowed because elderly people

a. Would lose too much heat as a result of vasolidation b. Have impaired circulation and decreased sensation c. Lack the manual dexterity to set the dial at the proper temperature d. Would become confused as the heat reduces blood flow to the brain

133. Because a child is suspected of having a tumor of the cerebellum, the nurse should expect to see which of these early signs?

a. Drooping of the eyelids b. Projectile vomiting c. Poor muscular coordination d. Changes in speech patterns

134. The nurse should include which of these measures when caring for a patient who has returned from hemodialysis?

a. Obtaining a post-procedure urine sample from the patient b. Increasing the patient’s intake of fluids c. Placing the patient in protective isolation d. Observing the patient for signs of shock

135. An 18-month-old child has been placed in Bryant traction. The traction has been properly applied if the nurse observes that

a. Both legs are at a 35° angle b. Ten-pound weights are in place c. The ropes are slack d. Both buttocks are off the bed

136. Which of these measures should the nurse include in the care plan of a patient who has renal calculi?

a. Restricting fluid intake b. Staining all urine

c. Maintaining bed rest d. Limiting potassium intake

137. Which of these observations of a woman who is in labor would indicate the onset of a complication?

a. Her contractions become prolonged without periods of complete relaxation

b. Her vaginal discharge is blood-tinged and mucoid c. She becomes diaphoretic and vomits 200 mL of undigested food d. She is having discomfort in the lower back during contractions

91

138. A patient who has a long-leg cast complains of tingling sensations of the casted extremity on the day following the application. Which of these actions should the nurse take next?

a. Report the complaint to the physician b. Administer the ordered analgesic c. Assess the neurovascular status of the affected extremity d. Place the affected extremity in a dependent position

139. A nurse teaches an 8-year-old boy who has diabetes mellitus about the inclusion of exercise in his daily schedule. Which of these comments, if made child, would indicate that the teaching was effective?

a. “I will do twenty sit-ups before breakfast.” b. “I will not be able to play with my Little League team.” c. “I will play football after lunch.” d. “I will not eat before I ride my bicycle.”

140. Which of these statements, if made by a patient to the nurse, would indicate that instructions about medications prescribed for tuberculosis were understood?

a. “I will take the medication until I feel better.” b. “I will have to take the medication for about a year.” c. “I will need to have periodic pro-thrombin time tests done.” d. “I will stop the medication if I experience chest pain.”

141. Which of these techniques would be most effective in removing the nits left by lice?

a. Applying a delousing shampoo for three consecutive days b. Brushing hair vigorously c. Covering the head with a knit stocking d. Using a fine tooth comb

142. The nurse should recognize that a patient who has had a perineal prostatectomy is most likely to develop which of these complications?

a. Uretereal stenosis b. Impotence c. Renal calculi d. Glomerulonephritis

143. The nurse discusses with a primigravida symptoms that most likely indicate that labor will begin soon. These symptoms include

a. Intermittent painless contractions. b. Leakage of colostrums from the breasts c. Urinary frequency d. Pink-tinged vaginal discharge

92

144. A nurse is reinforcing discharge instructions with a patient who has nitroglycerine (Nitrostat) sublingual tablets prescribed for angina. Which of these statements, if made by the patient, should indicate to the nurse that further instructions are needed?

a. “I will take as many tablets as I need to relieve the chest pain.” b. “I will rest for fifteen minutes after taking the tablet for chest pain.” c. “I will place the tablet under my tongue when I have chest pain.” d. “I will call the doctor if chest pain is not relieved by medication.”

145. In preparing for a 9-year-old child who has cerebral palsy for a tendon release operation, the ambulatory care nurse should take which of these actions?

a. Give the child a detailed account of the surgical procedure b. Show the child the surgical instruments c. Schedule the child for a preoperative visit to the pediatric unit d. Advise the parents to delay informing the child until the day of surgery.

146. A nurse should recognize that the primary purpose of increasing fluid intake for a patient hospitalized with pneumonia is to

a. Maintain renal function b. Improve cardiac output c. Promote bowel function d. Improve airway clearance

147. A newborn is to receive phototherapy. Which of these measures should be included in the infant’s care plan?

a. Keeping the infant well hydrated b. Using a water-based lubricant on the infant’s skin c. Keeping the infant on either the left or right side d. Using disposable diapers on the infant

148. A patient who has elevated blood glucose level is scheduled for a glucose tolerance test (GTT). Which of these instructions should the nurse reinforce?

a. “Increase your fluid intake for twenty-four hours before the test.” b. “Do not take anything by mouth for twelve hours before the test.” c. “Eat a high carbohydrate diet the evening prior to the test.” d. “Eliminate concentrated sweets from your diet for forty-eight hours prior

to the test.” 149. When a nurse is ambulating a resident who is totally blind, the nurse should stand

a. slightly ahead of the resident with the resident’s hand on the nurse’s arm b. directly behind the resident c. in close contact, with an arm around the resident’s waist d. within reach of the resident but not touching

150. An elderly resident falls and sustains a left hip fracture. The resident is initially placed in skin traction (Buck’s extension) for which of these purposes?

a. To reduce the fracture b. To approximate the edges of the fracture c. To prevent edema around the fracture d. To immobilize the fractured extremity

93

151. Which of these pieces of information should the nurse include in the care plan of a postpartal woman who is to receive ferrous sulfate (Feosol) after discharge?

a. A diet low in roughage will enhance absorption of the medication b. This medication will need to be taken until uterine involution is complete c. Black-colored stools are common side effect of medication d. This medicine will help to increase energy levels within a week.

152. A nurse is caring for a patient who is several hours postoperative following a left modified radical mastectomy. Which of these measures should be given priority in the patient’s care plan at this time?

a. Doing passive range-of-motion exercises of the patient’s left arm b. Splinting the patient’s left arm to the chest c. Encouraging the patient to use the left arm in personal care d. Keeping the patient’s left arm elevated

153. A 16-year-old boy has sudden, excruciating pain in his scrotum and groin. Based on this information, the nurse should suspect that the boy has which of these conditions?

a. Testicular torsion b. Varicocele c. Gonorrhea d. Renal calculi

154. When collecting data from a patient with hypertension, the nurse asks all of the following questions. Which question is related to a risk factor for hypertension?

a. “Do you have children?” b. “How many hours do you sleep at night?” c. “How often do you exercise?” d. “Do you eat a high-fiber diet?”

155. A parent has been instructed by the nurse about the prevention of Lyme disease in her child while on a camping trip. The instruction has been effective if the parent makes which of these statements?

a. “I will require my child to wear a jacket.” b. “I will have my child wear a long-sleeved shirt and his socks pulled up

over his jeans.” c. “I will require my child to wear a hat.” d. “I will have my child wear knee-high socks and sturdy hiking shoes.”

156. A nursing home resident who has chronic obstructive pulmonary disease (COPD) and who is receiving two liters of oxygen by nasal cannula complains of dyspnea. Which of these actions would be correct for the nurse to take?

a. Assist the resident with ambulation b. Obtain an order to increase the resident’s oxygen flow rate c. Encourage the resident to breathe deeply and cough d. Place the resident in a high-Fowler’s position

94

157. A nurse should observe a patient with insulin-dependent diabetes mellitus (IDDM) for symptoms of hypoglycemia, which include

a. Vomiting b. Frequent voiding c. Tremors d. Pinpoint pupils

158. A woman who is 28 weeks pregnant tells the nurse that her fingers are very swollen. In addition to reporting this symptom to the nurse-in-charge, the nurse should

a. Check the woman’s blood pressure b. Find out if the woman has been taking ferrous sulfate (Feosol). c. Test the woman’s urine for acetone d. Measure the girth of the woman’s abdomen

159. A resident is to receive 200 mL of three-quarter-strength tube feeding. Which of these proportions should the nurse administer?

a. 100 mL of feeding and 100 mL of water. b. 125 mL of feeding and 75 ml of water. c. 150 mL of feeding and 50 mL of water. d. 175 ml of feeding and 25 mL of water.

160. An aide asks a frail resident with dementia to sit on a chair while the aide goes across the room to get a wheelchair. Which of these assessments of the aide’s action is accurate?

a. It is considerate because it saves the resident’s having to walk to the wheelchair

b. It is risky because the resident is likely to forget the aide’s request c. It is a safety measure that allows the resident’s blood pressure to stabilize. d. It is thoughtless because the resident may feel abandoned

161. A patient who has sickle cell anemia is admitted to the hospital in occlusive crisis involving the right lower extremity. Which of these measures should be give priority in the patient’s care plan?

a. Encouraging active range-of-motion exercises b. Keeping the affected leg elevated c. Administering prescribed analgesics d. Decreasing fluid intake

162. A nurse should recognize that the purpose of using a pulse oximeter is to measure a patient’s

a. Oxygen saturation b. Blood volume c. Hemoglobin level d. Carbon dioxide saturation

95

163. A woman who is 12 weeks pregnant has been instructed about the alpha-fetoprotein test. Which of these responses, if made by the woman, would indicate that she understood the instructions?

a. "Now I don’t need to worry about having a Down syndrome baby.” b. “This will tell us if my baby will have any visual problems.” c. “This will say if my baby’s skeletal system is okay.” d. “This will help determine if my baby’s nervous system is developing

properly.” 164. A nurse removes an indwelling urethral (Foley) catheter from a patient. Six hours later, the nurse notes that the patient has not voided. Which of these actions should the nurse take?

a. Apply pressure to the patient’s suprapubic area. b. Obtain an order to recatheterize the patient c. Run the tap while the patient is on the toilet d. Tell the patient to call whenever she feels the urge to void

165. A patient has a gastroduodenoscopy under local anesthesia. Following the procedure, the patient’s care plan should include which of these actions?

a. Withhold fluids until the patient’s gag reflex returns b. Maintain the patient in high-Fowler’s position for at least four hours c. Encourage patient activity to increase motility of the intestinal tract d. Examine the patient’s abdomen for a pulsating mass for the next two

hours. 166. Which of these nursing measures should be included in the care plan of a woman who is eight weeks pregnant and is hospitalized with a diagnosis of hyperemesis gravidarum?

a. Encouraging the patient to take fluids by mouth b. Doing passive range-of-motion with the patient c. Checking the patient’s bilateral deep tendon reflexes d. Monitoring the patient’s intake and output

167. Which of these actions should the nurse take to prevent the development of a thrombophlebitis in a postoperative treatment?

a. Raise the knee gatch on the patient’s bed b. Ambulate the patient c. Encourage the patient to increase fluid d. Massage the patient’s legs.

168. Which of these behaviors is commonly observed in children who have been abused?

a. The child lies quietly while blood is being drawn b. The child cuddles a stuffed animal c. The child asks for candy d. The child pushes the nurse away when the nurse comes to take her

temperature

96

169. A nurse is caring for a patient who had a transurethral resection of the prostate (TURP) three days ago. The patient’s indwelling catheter was removed two hours ago and the patient tells the nurse that he is passing blood-tinged urine. Which of these actions should the nurse take?

a. Check the patient’s vital signs b. Collect a urine specimen from the patient c. Call the patient’s physician d. Encourage the patient to increase fluid intake

170. The nurse is caring for a patient receiving intravenous therapy. Which of these observations should indicate to the nurse that the intravenous infusion has infiltrated?

a. Blood backs up in the intravenous tubing b. Cutaneous tissue at the intravenous site is swollen c. A red streak appears along the vein above the intravenous site d. The skin around the intravenous site is warm to touch

171. A nurse is monitoring a resident’s blood glucose level and obtains a reading of 50mg/dL. The resident has non-insulin dependent diabetes mellitus (NIDDM). repiWhich of these actions by the nurse would be most appropriate?

a. Check for an insulin coverage order b. Take the resident’s vital signs. c. Give the resident 4 oz of orange juice d. Encourage the resident to engage in physical activity.

172. When the nurse assesses the woman’s fundus three hours after delivery, the nurse finds the fundus to be firm, displaced to the right side, and two fingerbreadths above the umbilicus. Which of these actions should the nurse take?

a. Encourage the woman to void b. Check to see when the woman last had an oxytocic drug c. Place gentle downward pressure on the woman’s fundus d. Massage the woman’s fundus

173. A child who is suspected of having an attention deficit disorder is most likely to have which of these symptoms?

a. Rapid recognition of objects b. Low frustration tolerance c. Enuresis d. Passivity

174. When a nurse is caring for an elderly patient who has dyspnea, which of these measures should be given priority in the care plan?

a. Splinting the patient’s chest when coughing b. Assisting the patient with ambulation c. Providing frequent rest periods for the patient d. Encouraging the patient to perform deep breathing exercises.

97

175. The nurse should recognize that an elderly resident who has a history of osteoporosis is at greatest risk for developing which of these complications?

a. Bone cancer b. Gall stones

c. Sciatica d. Stress fractures

176. An elderly patient who comes to clinic for a “flu shot” says to the nurse, “I can’t do anything and I’m not worth anything.” These comments should prompt the nurse to investigate whether

a. The patient is suicidal b. The patient’s comments are a realistic appraisal of the situation c. The patient is delusional d. The patient is asking for praise

177. A nurse is preparing a patient who had a hip replacement for discharge. Which of these self-care instructions should be reinforced?

a. “Keep the operative leg extended.” b. “Do not cross your knees.” c. “Sit in an upright position.” d. “Avoid climbing stairs.”

178. A nurse should encourage a patient who is on a low-sodium diet to select which of these foods?

a. Canned peas b. Peanut butter

c. Processed cheese d. Stewed prunes

179. A nurse is hurrying a nursing home resident to the shower. The resident is becoming increasingly resistant and threatens the nurse with a cane. Which of these actions would be best?

a. Return the resident to his chair and postpone the shower b. Get assistance in completing the shower c. Reprimand the resident and take his cane away d. Ignore the incident and continue the shower

180. A nurse is reinforcing instructions with a patient scheduled for a Papanicolaou test (Pap smear) at her next clinic visit. Which of these instructions is appropriate?

a. “Empty your bladder just before the test.” b. “Take a tap water douche the morning of the test.” c. “Do not have intercourse for two days before the test.” d. “Bathe in hot water the morning of the test.”

181. The father of a girl with Down syndrome asks the nurse why his daughter gets frequent upper respiratory infections. The nurse’s response should be based on the understanding that the child has

a. spastic muscles b. a rigid chest wall c. insufficient lung surfactant d. hypotonic muscles

98

182. An elderly resident with non-insulin dependent diabetes mellitus (NDDM) has been giving instructions regarding a 1,800 calorie American Diabetic Association (ADA) diet. Which of these snack selections, if made by the resident, should indicate to the nurse that further instructions are needed?

a. Flavored gelatin b. Fresh orange c. Ice cream d. Graham crackers

183. A patient who is in the hospital is extremely hyperactive. She enters the day room and seizes one patient after another, swings each around for a few dance steps, releases each and goes on to the next. Which of these actions by the nurse is indicated?

a. Recognize that the patient is using up excess energy and overlook the behavior

b. Take the patient to a quieter room c. Encourage the patient to select patients who need stimulation d. Ask the patient to consider how her behavior looks to others

184. Discussion groups for residents in skilled nursing facilities are most likely to foster self-esteem when they focus on

a. Physical complaints b. Past experiences c. Institutional food d. Family visits

185. After administering a parenteral medication to a resident, which of these actions should the nurse take?

a. Recap the needle before discarding b. Replace the needle in the original container before discarding c. Discard the needle uncapped d. Break the needle before discarding

186. Because a 15-year-old girl has been diagnosed as having anorexia nervosa, the nurse should expect the girl to have which of these signs?

a. Nausea b. Amenorrhea c. Ascites d. Dysphagia

187. A nursing home resident with chronic obstructive pulmonary disease (COPD) has a respiratory rate of 30 per minute and is started on continuous oxygen therapy. The nurse should recognize that the treatment has been effective if the respiratory rate has

a. Decreased to 20 b. Increased c. Remained the same d. Decreased to 10

99

188. A nursing assistant in a long-term care facility tells the nurse that a patient with iron-deficiency anemia has not eaten the meat on the lunch and dinner trays for several days. Which of these data should the nurse collect?

a. Have visitors been bringing food to the patient? b. What are the patient’s religious beliefs? c. Is the patient depressed? d. Are the patient’s teeth functional?

189. Which of these instructions should the nurse reinforce with a patient in a long-term care facility who is learning to use a walker?

a. “Use the walker as needed for balance.” b. “Step and move the walker simultaneously.” c. “Move the walker and then step into it.” d. “Glide the walker long the floor with each step.”

190. An infant’s one-minute Apgar score is 10. Which of these findings would result in the infant receiving an Apgar score of 9 instead of 10?

a. Heart rate of 138 b. Blue hands and feet c. Flexion of extremities d. Respiratory rate of 46

191. A nurse who is reinforcing discharge teaching with a patient who has active pulmonary tuberculosis should emphasize which of these measures?

a. Placing used tissue wipes in a paper bag to be burned b. Using a bacteriocidal soap for bathing c. Using disposable dishes d. Excluding visitors

192. A nurse is to obtain a residual urine specimen from an elderly resident whose indwelling Foley) catheter was just removed. The purpose of the procedure is to

a. Promote urinary bladder tone b. Check for urinary retention c. Prevent urinary incontinence d. Measure the amount of urine production

193. A daughter of a nursing home resident says to the nurse, “My mother only gets a bath every other day; why can’t she have one every day?” Which of these explanations would be most appropriate?

a. “We don’t have the staff to do a bath ever day.” b. “Most of our residents don’t want a daily bath.” c. “Daily bathing puts the residents at risk for developing colds.” d. “Daily bathing will further dry her skin.”

194. Cephalexin (Keflex) 0.5 gm is prescribed for a patient. The medication comes in 250 mg capsules. How many capsules should the nurse instruct the patient to take?

a. 2 b. 3

c. 4 d. 5

100

195. A resident is receiving intermittent nasogastric tube feedings. The nurse should aspirate the residual stomach contents prior to administering a scheduled feeding for which of these purposes?

a. To check the pH of the gastrointestinal fluids b. To evaluate the osmolarity of the stomach contents c. To determine absorption of formula in the stomach d. To obtain a specimen of gastric secretion

196. A resident with dementia of the Alzheimer type is admitted to a long-term care facility. During the resident’s first day in the facility, which of these nursing measures would have priority?

a. Determining how much protection the resident needs b. Confirming the resident to the assigned room c. Providing a vest restraint when the resident is in bed d. Assessing the resident’s food preferences

197. The nurse is caring for an elderly patient who had cerebrovasular accident (CVA). Because the patient has facial weakness, the nurse should take which of these measures when feeding the patient?

a. Placing the food in the affected side of the mouth b. Allowing the patient ample time for mastication c. Offering the patient water with each mouthful of food d. Hyperextending the patient’s head and neck

198. When a newborn is 3 hours old, the newborn dies. To assist the newborn’s parents with the grieving process, it would be most appropriate for the nurse to take which of these measures?

a. Providing an opportunity for the parents to hold their baby b. Suggesting that the father remove from their home all articles that have

been prepared for the baby before the mother is discharged c. Reassuring the parents that their feelings of loss will decrease in time d. Telling the parents that it is likely that they will have a healthy baby in the

future. 199. Which of these children being admitted to the pediatric unit should be put on isolation?

a. A 3-year-old child who has a ruptured appendix and peritonitis b. A 3-year-old child with nephosis c. An 18-month-old child with bacterial meningitis d. An 18-month-old child with laryngotracheobronchitis

200. While the nurse is bathing with a newborn, a loud noise is heard in the nursery. The nurse should observe the baby for the possible demonstration of which of these reflexes in response to the noise?

a. Moro b. Tonic neck c. Babinski d. Rooting

101

201. A female resident in a nursing home makes all of the following comments. Which one should be recorded on her nursing record?

a. “I dreamt about my mother, and she cooked my favorite food.” b. “My roommate repeats herself a lot, but she is good natured.” c. “The doctors only come around once a week.” d. “I was tricked into giving up my home to come to this jail.”

202. The nurse is monitoring an immediate postoperative patient for bleeding. Which of these findings would indicate that the patient is in the early stage of hypovolemic shock?

a. The patient has a large urinary output b. The patient has a rapid pulse rate c. The patient has slow, labored breathing d. The patient has an elevated temperature

203. A woman who is at term and whose cervix is 5 cm dilated is receiving an epidural anesthesia for pain relief. Which of these nursing assessments would identify a side effect of epidural analgesia?

a. Monitoring the woman’s oral termperature b. Monitoring the woman’s apical pulse rate c. Checking the woman’s blood pressure d. Checking the woman’s nail beds for capillary refill

204. Which of these measures should the nurse include in the care plan of a patient who had a modified radical mastectomy two days previously?

a. Having the patient do self-care activities b. Encouraging the patient to strengthen her upper extremities c. Limiting range-of-motion of the patient’s affected side. d. Prohibiting the patient from abducting the affected arm

205. Which of these approaches by a parent would most likely encourage a child to be successfully toilet trained?

a. Verbally remind the child when wet and/or soiled b. Toilet the child in a unattractive room c. Dress the child in snug-fitting panties d. Limit toileting sessions to five to ten minutes

206. Which of these approaches should the nurse use when a patient who has weakness on the left side is assisted to ambulate with a cane?

a. Hold the cane in the right hand and advance it with the unaffected leg b. Hold the cane in the left hand and advance the cane with the affected leg c. Hold the cane in the left had and advance the cane with the unaffected leg d. Hold the cane in the right hand and advance the cane with the affected leg

207.What is the first intervention for a client having a myocardial infarction (MI)?

a. Administer morphine b.Administer oxygen c. Administer sublingual nitroglycerin d.Obtain an electrocardiogram (ECG)

102

208. A 78-year-old client is admitted with diagnosis of dehydration and change in mental

status. He’s being hydrated with intravenous (IV) fluids. When you take his vital signs, you note that he has fever of 103 degrees F, a cough producing yellow sputum, and pleuritic chest pain. You suspect he may have which of the following conditions?

a. acute respiratory distress (ARDS) b. MI c. Pneumonia d. Tuberculosis

209.When teaching a newly diagnosed diabetic client about diet and exercise, be sure to include which one of the following?

b. use of fiber laxatives and bulk-forming agents c. management of fluids , protein and electrolytes d. reduction of calorie intake before exercising e. caloric goals, food consistency, and physical activity

210.A client with diabetes has polyphagia, polydipsia, and oliguria; he also complains of

headache, malaise, and visual changes. Assessment shows signs of dehydration. Which of the following diagnosis could be made?

a. diabetes insipidus b.diabetic ketoacidosis c. hypoglycemia d.syndrome of inappropriate antidiuretic hormone (SIADH)

211. A patient begins having auditory hallucinations. When the nurse approaches, the

patient whispers, “Did you hear that terrible man? He is scary!” Which would be the best response for the nurse the nurse to make initially?

a. “What is he saying/” b. “I didn’t hear anything. What scary things is he saying?” c. “Who is he? Do you know him?” d. “I didn’t hear a man’s voice, but you look scared.” 212. A woman is in preterm labor and is bleeding. She requests a sip of water after being

in labor for 2 hours. The most appropriate nursing action would be: a. Checking the physician’s order b. Giving her small sips of water. c. Offering her ice chips. d. Telling her she cannot have fluids at this time. 213. A woman in labor has a history of undiagnosed vaginal bleeding. Which procedure

may be contraindicated on her arrival in the labor room? a. Initiating intravenous therapy b. Taking her blood pressure c. Examining her vaginal canal d. Monitoring FHR

103

214. A pt with a leg fracture is to be taught the three-point gait prior to discharge. This patient should be given which instruction by the nurse?

(a) “Advance your right crutch forward, swing the left foot forward, advance the left crutch, and then bring the right foot forward.”

(b) “Move your right crutch and left foot forward together, and then swing the right foot and left crutch in one movement,”

(c) “While partially bearing weight on your left leg, advance both crutches and then bring your right leg forward

(d) “Using one movement, advance your left foot and both crutches and then bring your right leg forward.”

215. Clients who use PCA analgesia usually: (a) Require less pain medicine (b) Have slower post operative recovery. (c) Require supplemental pain medicine. (d) Experience more respiratory depression 216. If the PCA client’s respiratory rate is depressed (rate of 10), the nurse should: (a) Administer nalaxone (Narcan) (b) Discontinue the PCA (c) Obtain pulse oximetry (d) Obtain other vital signs (BP, Pulse). 217.Which assigned client should the nurse take care of first? (a) A client who is incontinent of urine has wet the bed. (b) A client who had surgery yesterday asking for pain medication (c) A client who has stage III decubitis ulcer needs dressing change. (d) A client is diabetic says he feels faint. 218.A client has been diagnosed with advanced untreatable cancer. When the nurse

enters the room to set up the bath equipment, the client says, “I’m not an invalid, you know. I can take care of myself. Get out and leave me alone.” What is the best nursing response?

(a) “I know you are not an invalid. I was just trying to help. (b) “It sounds to me that you are angry about something. Did somebody do something

wrong?” (c) “You are pretty upset. Let’s talk about it.” (d) “I’ll just set up this equipment for you to bathe and come back later when you are not

so angry.” 219.While transferring a client with left-leg weakness from bed to wheelchair, the

priority nursing action is to: (a) Have the seat of the wheelchair at a right angle to the bed. (b) Lock the wheels of the bed and of the wheelchair (c) Allow the client to do as much as possible to increase sense of independent. (d) Tell the client to lock the hands around the nurse’s neck to provide a sense of

security.

104

Section SIX (1) A nurse enters a room of a patient who is suspected of experiencing pulmonary

embolism. What signs and symptoms would this patient show? (a) Patient is in a comatose state (b) Patient is coughing out blood (c) Pt has no breath sounds in affected area (d) Pt is restless and lethargic (2) A patient continues to have diarrhea despite being on medication therapy; what type

of medicine would be causing this diarrhea? (a) Vitamins (b) Antibiotic (c) Cold medicine (d) Antihypertensives (3) You received the lab results for a patient which reads: potassium 3.0, WBC 6000, Platelets 280,000. Which diet would you recommend for the patient? (a) Dried apricots (b) Green leafy vegetables (c) Dairy products (d) Organ meats (3) A diabetic patient was cool, pale and restless when the nurse checked on him. The

nurse was later charged with negligence. Which of the following actions by the nurse could be used to refute the charge of negligence?

(a) The nurse took vital signs and documented it on the flow sheet (b) The nurse provided oxygen at two liters by nasal cannula and informed the charge

nurse. (c) The nurse did a finger stick which read 58, so gave the patient orange juice and

turkey sandwich (d) The nurse gave the patient his insulin right on time before breakfast. (4) You are the charge nurse and you have to decide on room assignment, which of these

patients would you assign a private room (a) A patient with scabies (b) A patient with conjunctivitis (c) A patient with a positive Mantoux skin test (d) A patient with HIV infection

(5) A patient on strict input and output monitoring consumed 4oz of jello, 1 slice of

bread, 1 cup of cereal with 4 oz of milk, I cup of water, 8 lbs of steak, and 7 oz of broth. What is the total input for this patient?

(a) 470 cc (b) 690 cc

(c) 1200 cc (d) 1470 cc

105

(6) Which of these patients would be given first priority when planning a meal (a) A diabetic patient (b) A patient with tuberculosis

(c) A patient with renal failure (d) A patient with hypertension

(7) A patient is experiencing Kussmaul respiration, the underlying cause of this type of

respiration would be found with (a) a patient with an asthma attack (b) a patient with end stage renal disease (c) a patient with full blown AIDS (d) a patient with diabetic ketoacidosis

(8) A 12 year old with Juvenile Rheumatoid Arthritis is complaining of severe pain; what

would the nurse do to relieve the pain? (a) Apply heat to affected site (b) Apply cold compress to the joint frequently (c) Allow rest periods for the client (d) Do range of motion exercises as tolerated

(9) A neonate is to receive vitamin K injection to prevent bleeding; which injection site is

appropriate for the neonate? (a) Vastus lateralis muscle (b) Deltoid muscle (c) Ventral gluteal muscle (d) Dorsal lattimus muscle

(10) A patient with tuberculosis is being transported to the Radiology department for

chest X’ray. Which infection control measure is appropriate for the nurse to use? (a) The nurse must put on a mask while transporting the client (b) The client should wear a mask before leaving room (c) Patient should be given a tissue paper to cover his mouth if he feels like

coughing (d) The nurse should choose the route less traveled to avoid contact with other

persons.

(11) Which of these actions would be the safest for the nurse to do when sending a patient for Magnetic Resonance Imaging?

(a) Sedate the client (b) Remove the wooden prosthesis (c) Remove the client’s dentures (d) Turn off the client’s pacemaker

(12) A nurse is to administer epidural to a patient who is scheduled for vaginal

delivery. What is the appropriate time to administer the epidural? (a) At the beginning of labor (b) During the second stage of labor (c) When the mother starts pushing (d) During the third stage of labor

106

(13) There is an outbreak of hook worm infestation in a juvenile group home. Which patient teaching would be effective to reduce the problem

(a) Counsel clients to wear shoes (b) Counsel clients to avoid eating cold foods such as salads (c) Advise clients to take baths twice a day (d) Encourage client to take vitamin pills daily

(14) A patient with type 1 diabetes received NPH insulin at 8 am. When should the

nurse arrange to provide snacks to the patient (a) At mid afternoon around 4pm (b) Mid morning around 11 am (c) After lunch time at 12 mid-day (d) After dinner at 8pm

(15) A patient is suspected to be undergoing a hypoglycemic reaction after receiving

30 units of regular Insulin. What response by the patient would indicate such a condition?

(a) Pt unable to tell the nurse his name, time and place. (b) Pt has a fruity odor breath. (c) Pt has a rapid shallow breathing (d) Pt complaints of severe thirst

(16) The priority nursing intervention when a patient returns from

bronchotracheoscopy is to: (a) Check for patient ability to speak coherently (b) Provide a tracheostomy tray at the bedside (c) Check for the return of gag reflex before feeding (d) Provide oxygen at 2 liters per nasal cannula stat.

(17) A patient is admitted for DIC (Disseminated intravascular coagulation); which

findings would the nurse have to report to the physician immediately? (a) Platelet count of 20,000/mm3 (b) Hemoglobin of 16 (c) WBC of 11000 (d) Hematocrit of 45-52%

(18) A patient is being discharged from the hospital after treatment for tuberculosis.

Patient should be taught that he should continue taking his medicine for a period of (a) 1 year (b) 9 months

(c) 2 years (d) As soon as he feels better.

(19) A patient placed on a Holter monitor would indicate understanding of patient

teaching on the use of the monitor if he states: (a) “I would wear monitor for 23 hours of the day” (b) “I would take off monitor when taking care of my hygiene” (c) “I cannot do any strenuous exercise whilst on the monitor” (d) “I would take a record of all my activities”

107

(20) When collecting a urine specimen for a routine urinalysis, which of the nursing action is most important

(a) Label the container (b) Check the identification of the client (c) Do not use gloves when handling a urine specimen (d) Instruct the client to put the specimen on the counter at the nurse’s station for

pick up.

(21) The nurse is planning care for a preschooler postoperatively, which behavior will characterize pain response in this age group?

(a) Demanding explanations (b) Clinging to mother (c) Asking for special treats (d) Becoming mute and showing regressing behavior

(22) A patient has been placed on Disulfiram (Antabuse). What questions would you

ask before administering the drug (a) “When was your last meal?” (b) “Are you allergic to shell fish?” (c) “When was your last drink?” (d) “Have you been hearing voices?”

(23) A child is diagnosed with seizure disorder. Which statement made by the mother

would indicate understanding of care for the child during a seizure episode? (a) “I would put a padded tongue blade in his mouth during an episode to prevent

him biting his tongue” (b) “I would restrain him from hitting his head on the floor” (c) “I would move furniture away from him” (d) “I would move him unto a safety mattress”

(24) At a health seminar, which of these topics should the nurse address as a priority

(a) The importance of rectal examinations after 18 years to check for prostrate cancer

(b) The importance of women doing annual pap smears after 40 years of age (c) Breast self examination should be done weekly to detect for breast cancer (d) Testicular self-examination once a year

(25) One hour after delivery, the nurse notes that a post partum mother has vaginal bleeding. What action should the nurse take first?

(a) Notify the doctor (b) Count the patient’s pulse (c) Do a vaginal examination (d) Massage the uterus

108

(25) A patient blood work indicates hypocalcemia, which of these would be recommended in the diet by the nurse

(a) Eat a lot of Kiwi (b) Eat prunes (c) Drink Coca Cola (d) Drink pineapple juice (26) A patient diagnosed with Hepatitis C asked the nurse how he must have acquired

the disease. The appropriate questions to ask the patient to elicit an answer is: (a) “Did you receive a blood transfusion lately?” (b) “Did you eat any raw oysters?” (c) “Have you traveled to a foreign country recently?” (d) “Have you been having unprotected sex?”

(27) A patient is to undergo a liver biopsy. What position would the nurse place the

patient? (a) Pt would be placed in an upright position (b) Pt would be placed in a trendelburg position with feet slightly elevated (c) Place pt in a sitting position with head on bed side table (d) Pt would be placed on the right side with a pillow on his side.

(28) An employee is one hour late and on arrival smells of alcohol. As a charge nurse

what would be the appropriate action to take? (a) Ask the employee to go home (b) Do not give an assignment to the staff and call the supervisor for further

instructions (c) Warn the staff and document behavior (d) Assess the staff about orientation to person, time and place; then if oriented,

allow to work. (29) The nursing team develops a plan of care for client with CVA. Which is the most

important plan to the client’s rehabilitation? (a) Prevent contractures and joint deformities (b) Regulate bowel and bladder elimination (c) Client’s ability to communicate (d) Client assumes normal dietary intake.

(30) A patient with a WBC of 4000 was given teaching on his condition. What

statement indicates patient understands teaching? (a) Pt states “I would go out to get some fresh air frequently” (b) Patient states “I have to watch my cholesterol level” (c) Patient states “I‘II tell my family to call me instead of visiting” (d) Patient states “I have to get my allergy

(31) A patient with diagnosis of type 1 diabetes received lente insulin at 8am, when

would be an appropriate time to serve patient a snack? (a) 2 o’clock pm (b) 10 o’clock am

(c) 6 o’clock pm (d) 12 midnight

109

(32) What would be the recommended fluid intake for a client with emphysema? (a) 8 glasses of milk a day (b) 2 liters of water a day (c) Place on fluid restriction. (d) Provide copious amount of broth

(33) A 16 year old sexually active patient reported during history taking that she has

Chlamydia infection. What appropriate action should the nurse take at this time? (a) Report patient’s disease to the CDC in Atlanta because that is the law (b) Ask patient to bring parents for informed consent for treatment (c) Ask patient to provide names, address and phone numbers of sexual partners (d) Ask patient to provide urine sample for pregnancy test.

(34) A patient with type 1 diabetes has gone into a hypoglycemic reaction. What are

the signs and symptoms of hypoglycemia? (a) Diaphoresis, lethargy and confusion (b) Acetone breath, hunger and restlessness (c) Polydipsia, polyphagia, and polyuria. (d) Dizziness, vomiting, hypotension

(35) A child weighs 35 pounds. You are to give a drug with instruction “Give 5 ml per

kilo of body weight”. What is the total dosage of the drug you would give? (a) 7 cc (b) 80cc (c) 96cc (d) 102cc

(36) You are taking of a newborn baby whose mother used crack cocaine. What

nursing action would be most appropriate in meeting the child’s needs? (a) Tightly swaddle the baby. (b) Provide feeding with a Breck feeder (c) Supplement respiration with O2 at 2 liters (d) Increase sensory stimulation of the child.

(37) What would be an appropriate toy for 3 year-old children in a playroom? (a) A paper and crayon for drawing (b) A pulling wagon or cart (c) A punch bag (d) Soft molding clay

(38) A pregnant woman who is 38 weeks gestation was observed choking while eating.

What would be the most appropriate rescue method? (a) Quickly perform a Heimlich maneuver. (b) Call for help and start CPR (c) Place patient prone and provide 5 back blows (d) Perform a chest thrust

110

(39) A patient with Leukemia is hospitalized; all visitors to patient’s room are screened at the Nurses’ station. Which of these visitors would be barred from visiting patient?

(a) A visitor who has AIDS (b) A visitor with a rash on the cheeks (c) A patient with genital herpes (d) A patient with hemophilia

(40) A client was admitted to your ward and one hour later decided to refuse treatment

and wants to go home. What action should the nurse take at this time? (a) Call the hospital security to help you confine patient to unit. (b) Designate another nurse to supervise the patient on 1:1 basis (c) Read the patient his Miranda rights before he leaves. (d) Have the patient sign a form indicating his refusal for treatment against

medical advise (41) A patient fell from the roof of his house resulting in head injury. What would be

the best way to assess his neurological status? (a) Ask the patient to wiggle his toes (b) Ask the patient to state his name (c) Ask the patient to open eyes (d) Ask patient to move from side to side

(42) Which of these patients would you assign to a nursing assistant on your team?

(a) A patient who just returned from a hip replacement surgery? (b) A patient with renal failure (c) A patient with a stillbirth delivery (d) A patient with CVA who needs blood pressure checked before medication.

(43) Which of these signs would be an indication that labor is progressing normally?

(a) Contraction felt in the abdomen and pelvis (b) Braxton-Hicks contraction (c) When the amniotic sac breaks (d) Pain radiating from the waist to the back

(44) A 45-year-old man had a permanent pacemaker implanted one year ago. He

returns to the outpatient clinic because he thinks the pacemaker battery is malfunctioning. The nurse would expect the client to exhibit which of the following symptoms?

(a) Abdominal pain, nausea, and vomiting (b) Wheezing on exertion, cyanosis, and orthopnea (c) Peripheral edema, shortness of breath, and dizziness. (d) Chest pain radiating to the right arm, headache and diaphoresis.

111

(45) You are a nurse working in the emergency room where you encounter a 79 year-old female who has been admitted for a broken arm. The woman resides with her daughter because of decreased mobility secondary to arthritis. During your initial assessment, you notice multiple bruises on the woman’s back and burn on her hand. You should:

(a) Call the patient’s daughter into the room and confront her about abusing her mother.

(b) Call the police to report the abuse (c) Do nothing; older individuals are prone to accidents and injuries. (d) Question the patient regarding recent injuries, falls or trauma.

(46) After receiving report from the night nurse, which of the following patients

should the nurse see FIRST? (a) A 31 year old woman refusing Carafate before breakfast (b) A 20-year-old man with left-sided weakness asking for assistance to the

commode (c) A 52-year-old woman complaining of chills who is scheduled for a

cholecstectomy (d) A 65-year-old man with a nasogastric tube who had a bowel resection

yesterday (47) The physician orders Clindamycin 3mg/kg IV every 8 hours for a 3-year-old boy.

The nurse enters the patient’s room to administer the medication and discovers that the boy does not have an identification bracelet. What should the nurse do?

(a) Ask the parents at the child’s bedside to state their child’s name (b) Ask the child to say his first and last name (c) Have a co-worker identify the child before giving the medication (d) Hold the medication until an identification bracelet can be obtained.

(48) A nurse is going on rounds to check on her patients, found a child in Bryant’s

traction. Which of this observation would need the nurse’s immediate intervention? (a) The child buttocks is comfortably laying on the bed (b) The weight is hanging off the floor instead of it being on the bed. (c) The child leg is suspended at a 90 degree angle (d) The child’s head is resting on a pillow.

(49) The treatment plan for an Alzheimer patient in a nursing home identified one of

the problems as poor dietary intake. What suggestions would you recommend as a solution to this problem?

(a) Sit down with patient to discuss his favorite foods (b) Provide small frequent feedings consisting of finger foods (c) Recommend vitamins to increase appetite (d) Assist client to clean mouth frequently to improve appetite

(50) A postpartum direct Coomb’s test indicates that an Rh- mother is not sentisitized

to the RH+ baby blood. What action is appropriate before the mother is discharged? (a) No action necessary since the mother was not sensitized (b) Give vitamin K shot to the neonate to prevent bleeding

112

(c) Test the neonate for PKU before discharge (d) Give the mother RhoGam within 72 hours after delivery.

(51) Which of the following is a characteristic sign of systemic lupus erythematosus

(SLE)? (a) Butterfly rash (b) Watery eyes (c) Diplopia (d) Ptosis

(52) A client with Alzheimer’s disease has a nursing diagnosis of Risk for injury

related to memory loss, wandering and disorientation. Which nursing intervention should appear in the client’s care plan to prevent injury?

(a) Remove hazards from the environment (b) Provide many detailed instructions to the client (c) Keep the client sedated whenever possible (d) Obtain an order from physician to use restraints especially at night.

(53) A 20 year-old female with cystic fibrosis has a small bowel obstruction. She’s

admitted to the medical-surgical unit for treatment, which involves placement of an intestinal tube connected to intermittent suction. Which nursing intervention would be most effective for this client?

(a) Record intake and output accurately (b) Turn the client from side to side as prescribed (c) Give the client sips of water to facilitate passage of the tube through the bowel (d) Add antacids to the intestinal tube to reduce bowel reaction

(54) A client returns from an endoscopic procedure during which he was sedated.

Before offering the client food, which of the following action should the nurse take? (a) Monitor the client’s respiratory status. (b) Check the client’s gag reflex. (c) Place the client in a side-lying position. (d) Have the client drink a few sips of water.

(55) A client complains of severe pain 2 days after surgery. Which of the following

actions should the nurse perform first? (a) Offer reassurance to the client that he will feel less pain tomorrow. (b) Allow the client time to verbalize his feelings (c) Check the client’s vital signs (d) Administer an analgesic

(56) During a routine prenatal visit, the nurse teaches the client how to relieve

constipation. Which statement by the client indicates that she requires further teaching?

(a) “I’ll decrease my intake of green leafy vegetables” (b) “I’ll limit daily fluid intake to four 8-oz glasses” (c) “I’ll increase my intake of unrefined grains” (d) “I’ll take iron supplements regularly”

113

(57) You are evaluating the nutritional needs of a strict vegetarian, which essential nutrient would you expect the client to lack?

(a) Vitamin C (b) Vitamin B12 (c) Carbohydrates (d) Amino Acids

(58) Which of these actions would constitute false imprisonment?

(a) Failing to give an elderly client his eye glasses. (b) Placing a confused patient in a room with the door locked. (c) Putting restraints on a self injurious patient before calling the doctor for an

order. (d) Denying a patient in the intensive care unit a visit from family.

(59) Which of these foods would be inappropriate to give to a 2 year old?

(a) A peeled banana (b) Crackers with cheese (c) Rice cereal with milk (d) Fresh grapes

(60) In children suspected to have a diagnosis of diabetes, which of the following

complaints would be most likely to prompt parents to take their school age child for evaluation?

(a) Polyphagia (b) Dehydration (c) Bed wetting (d) Weight loss

(61) The nursing care plan for a client with decreased adrenal function should include

(a) Encouraging activity (b) Placing client in reverse isolation (c) Limiting visitors (d) Measures to prevent constipation

(62) A patient has been receiving digoxin (Lanoxin) 0.125 mg daily for a week. When

the nurse visits him at home, the patient tells the nurse about several problems that have been developing over the last few days. Which of these complaints is suggestive of digoxin toxicity?

(a) Constipation (b) Urinary frequency (c) Loss of appetite (d) Ankle edema

(63) The nurse is assigned to work with a patient diagnosed as having pernicious

anemia. In evaluating the diet for the patient, the nurse would know the patient understands dietary parameters when he chooses

(a) Meat, milk, cheese (b) Whole grains, cereals

114

(c) Fruits, green leafy vegetables (d) Organ meats, yellow vegetables

(64) Which nursing action is the most critical when caring for a patient who is

receiving continuous nasogastric tube feeding (a) Warming the feeding to room temperature (b) Maintain accurate records of intake and output (c) Flushing the tube with water every 4 hours (d) Keeping the patient in a semi-Fowler’s position

(65) The nurse is caring for a client with Rheumatoid Arthritis. Which nursing

diagnosis should receive priority in the plan of care? (a) Risk for injury (b) Self care deficit (c) Alteration in comfort (d) Alteration in mobility

(66) The nurse is caring for a client with COPD who becomes dyspneic. The nurse

should (a) Instruct the client to breathe into a paper bag (b) Place the client in a high fowler’s position (c) Assist the client with purse lip breathing (d) Administer oxygen at 6l/minute via nasal cannula.

(67) A client is admitted to the emergency room in a nonketotic hyperosmolar coma.

After ascertaining that her airway is patent and her heart rhythm is sinus, the nurse should assess:

(a) her neurological status (b) her urine output (c) her family’s coping mechanisms (d) her skin breakdown

(68) A client has returned from the recovery after a gastrectomy. The nurse anticipates

that she will have a(n): (a) nasogastric tube(NG) tube (b) Hemovac self-contained system (c) sternal suture line (d) cardiac arrhythmia

(69) A 22-year-old male client has been recently diagnosed with pyelonephritis. The

immediate nursing intervention include: (a) encouraging increased fluid intake (b) inserting a nasogastric tube (c) asking Joe where he feels the pain (d) encouraging Joe to verbalize his feelings

(70) Which of the following activities in a high-risk prenatal clinic could the

registered/licensed nurse delegate to a nursing assistant?

115

(a) Taking the blood pressure of a woman with preeclempsia (b) Teaching a pregnant woman how to distinguish signs of true labor from

Braxton-Hicks contractions (c) Obtaining weights of pregnant clients before they are examined by a provider. (d) Referring an obese pregnant woman to a dietitian for nutrition counseling.

(71) A client is admitted for COPD. Which of the following symptoms would require

the nurse’s immediate attention? (a) Nausea and vomiting (b) Restlessness and confusion (c) Low grade fever and cough (d) Irritating cough and liquefied sputum

(72) An 80-year-old client receiving Lasix has a serum potassium level of 2.3. Which

nursing action is most appropriate? (a) Check the EKG patterns (b) Observe for signs of decrease cardiac output (c) Check physicians order for digitalis (d) Assess for client’s vital signs and record them

(73) A client has arterial blood gases reading as follows: PaO2, 98 mmHg; Sao2, 98%,

pH 7.47, PaCo2, 40mmHg, HCO3, 30mEq/l. What condition would patient show? (a) Respiratory acidosis (b) Respiratory alkalosis (c) Metabolic acidosis (d) Metabolic alkalosis

(74) Following an open reduction of the tibia, the nurse notes bleeding on the client’s

cast. Which intervention would be most appropriate? (a) Call the physician and prepare to take the client back to the operating room (b) Assess the client’s hemoglobin to determine if the client is in shock (c) Outline the area with ink pen and check it every 10 min to see if the area has

increased (d) No action is required since post operative bleeding can be expected.

(75) To administer a saline enema to Ms Green, the nurse inserts the enema tubing 3

inches into the client’s rectum and elevate the saline container 4 inches above the client’s rectum. After the nurse opens the clamp, the saline solution does not infuse. What is the best action for the nurse to take?

(a) Raise the saline container 8 more inches above the rectum (b) Insert the tubing an additional 4 inches into the rectum (c) Instruct the client to take several slow deep breaths (d) Remove the tube and check the client for fecal impaction

116

(76) The nurse is caring for a client receiving total parenteral nutrition. Which serum lab test should the nurse expect to monitor daily?

(a) Hemoglobin (b) Cholsestrol

(c) Prothrombin time (d) Glucose

(77) What the position of choice during the immediate postoperative period for a client

who has undergone a radical mastectomy of the right breast? (a) Side lying on the affected side with the bed flat (b) Supine with arm on the operative side in a dependent position (c) Semi-Fowler’s position with the arm on the affected side elevated (d) Sim’s position with the arm on the operative side in a dependent position

(78) The night nurse is unable to contact the physician and she is concerned that her

critically ill patient’s condition is getting worse. She is legally obligated to (a) Document that the physician did not return her call (b) Provide whatever care is necessary to save the patient (c) Notify the risk manager of the problem (d) Continue to go up the chain of command until her patient receives medical

attention. (79) A young adult seeks treatment in an out-patient mental health center. The client

tells the nurse he is a government official being followed by spies. On further questioning, he reveals that his warnings must be heeded to prevent nuclear war. What is the most therapeutic approach by the nurse?

(a) Listen quietly without comment (b) Ask for further information on the spies (c) Confront the client on his delusion (d) Contact the government agency.

(80) You have been assigned to work in a reservation for the Sioux (American Indian)

tribe. What are some of the basic characteristics of this tribe would you observe. (a) Most of the youth indulge in unprotected sex (b) Native Americans direct their eye contact to the floor during conversation. (c) You should not shake a man’s hand (d) You should not speak, unless spoken to.

(81) When assessing a client admitted to the hospital for diabetic acidosis, which of the

following clinical manifestations would the nurse expect (a) A blood pH level of above 7.5 (b) Arterial blood pCO2 above 40 (c) Blood pH level below 7.3 (d) Arterial blood pCO2 below 10

(82) A client has been receiving dexamethasone (Decadron) for control of cerebral

edema. Which of the following assessment would indicate that treatment is effective? (a) A positive Babinski’s reflex (b) Increased response to motor stimuli (c) A widening pulse pressure

117

(83) A client has been taking levodopa( Larodopa) for Parkinson’s disease. To evaluate the effectiveness of the medication, the nurse would document which change in the chart? Client has:

(a) An increase of 2 lbs in weight. (b) Less resistance to a respiratory infection. (c) No tremors or shuffling gait (d) Been more ambulatory

(84) When teaching a client about an oral hypoglycemic medication, the nurse should

place primary emphasis on (a) Recognizing signs and symptoms of toxicity (b) Taking the medications at specific times (c) Increasing the dosage based on blood glucose (d) Distinguishing hypoglycemia from hyperglycemia.

(85) A nurse returns to the nurse’s station after making clients rounds and finds four

phone messages. Which of the following messages should the nurse return FIRST? (a) A client with hepatitis A who states “My arms are itching” (b) A client with a cast on the right leg who states “I have a funny feeling in my

right leg” (c) A client with osteomylitis of the spine who states, “I am so nauseous that I

can’t eat.” (d) A client with a client with arthritis who states, “I am having trouble sleeping

at night” (86) A 28 year-old male has been found wandering around in a confused pattern. The

male is sweaty and pale. Which of the following test is most likely to be performed first?

(a) Blood sugar check (b) CT scan (c) Blood cultures (d) Arterial blood gases

(87) A parent calls the pediatric clinic and is frantic about the bottle of cleaning fluid

her child drank 20 minutes ago. Which of the following is the most important instruction the nurse can give to the parent?

(a) This too shall pass (b) Take the child immediately to the ER (c) Contact the poison control center quickly (d) Give the child syrup of Ipecac

(88) A client admitted for treatment of alcohol dependence, displays the following

symptoms: slurred speech, ataxia, uncoordinated movements, and headaches. Which nursing action should be taken first?

(a) Observe the client for 8 hours to collect additional information (b) Perform a complete physical assessment (c) Collect a urine specimen for a drug screen (d) Encourage the client to talk about whatever is bothering him.

118

(89) An elderly man diagnosed with major depression is demonstrating decreased problem-solving ability, psychomotor retardation, and social isolation. In planning activities for this client during early phase of hospitalization, which nursing action would be appropriate?

(a) Prepare and give him a schedule of activities to follow and monitor his participation

(b) Encourage him to choose his own activities. (c) Allow him sometime to acclimatized to the milieu before scheduling activities (d) Allow him to rest quietly to restore his energy.

(90) An elderly client who has dementia related to cerebral arteriosclerosis says to the

nurse, “I’m going to the university today to be a guest lecturer on aerodynamics. Which response by the nurse would be therapeutic?

(a) “Do you know that you are in the hospital now?” (b) “Are you saying that you would like to be asked to give a lecture at the

university?” (c) “How about watching a movie on television instead?” (d) “It is more important that you don’t tire yourself out.”

(91) A Jehovah Witness client is 48 hours postpartum. She is scheduled to be placed

on some medications. Which of these medications you would not give? (a) Bethametazone (b) Regular insulin

(c) RhoGam (d) Morphine

(92) Prior to developing a nursing care plan for a hospitalized 3-year-old, which

activity would be a priority? (a) Assess the child’s rituals and routines at home. (b) Introduce the child to the other personnel (c) Explain to parents the importance of limiting visiting hours (d) Identify the child’s favorite foods.

(93) The nurse arrives for the day shift and receives her assignment around 7:30 a.m.

Which client is a priority and should be seen first? (a) A man with a diagnosis of rule-out an MI. He is on a monitor and having 4-6

premature beats per hour. (b) An elderly lady who is confused and has constant urinary dribbling (c) A pneumonia client with increasing confusion and a temperature of 104

degrees at 6:30 a.m. (d) A diabetic client who experienced a restless night and 7:00 a.m. blood sugar

was 170. (94) The unit is short-staffed because two nurses did not report to work. Of the

following patients, which should the nurse care for FIRST? (a) A pt just admitted with acute abdominal pain and possible Cholecystisis. (b) A pt with nephritic syndrome with increasing edema, hourly urine and vital

signs order. (c) A confused pt yelling because he is in soft restraints and cannot get out bed. (d) A head-injury pt with an IV who was just admitted to the unit.

119

(95) Which client would be the highest risk for injury? (a) A 3-month-old in an infant seat sitting on a coffee table. (b) A 2-year-old playing in a living room unattended by an adult. (c) A 2 1/2 year-old with a tracheostomy playing outside in the backyard. (d) A 7-year-old who goes to after school care in a 38-year-old home.

(96) As a charge nurse, you would demonstrate an understanding of appropriate

delegation when you assign one of the following clients to the UAP. (a) A psychotic client (b) A client receiving chemotherapy (c) A client in Buck’s traction. (d) A client receiving a blood transfusion

(97) Which sequence is correct when providing care for a client immediately prior to

surgery? (a) Administer preoperative medication, client signs operative permit, determine

vital signs. (b) Check operative permit for signature, advice clients to remain in bed,

administer preoperative medication. (c) Remove client’s dentures, administer preoperative medication, client empties

bladder. (d) Verify client has been NPO. Client empties bladder, family leaves room.

(98) A client is admitted with a diagnosis of Hepatitis B. In reviewing the initial laboratory results, the nurse would expect to find elevation in which of the following values?

(a) Blood urea nitrogen (b) Acid phosphate (c) Bilirubin (d) Sedimentation rate

(99) The nurse is assessing a client who has taken Haldol (Haloperidol) for several

months. Which of the following is a side effects of this medication and must be reported IMMEDIATELY to the Physician

(a) Muscle flaccidity (b) Dystonia reaction (c) Mood swings (d) Dry, harsh cough

(100) A client was re-admitted to the hospital following a recent skull fracture. Which

of the following symptoms require the nurse’s IMMEDIATE attention? (a) Lethargy (b) Agitation (c) Ataxia (d) Hearing loss

120

(101) A homecare nurse is planning activities for the day. Which of the following clients should the nurse see FIRST?

(a) A new mother is breastfeeding her two day-old infant who was born five days early.

(b) A man discharged yesterday following treatment with IV heparin for a deep-vein thrombosis

(c) An elderly woman discharged from the hospital three days ago with pneumonia

(d) An elderly man who used all his diuretic medication and is expectorating pink-tinged mucus.

(102) A nurse performs a vaginal assessment on a pregnant client in labor. On

assessment, the nurse notes the presence of the umbilical cord protruding from the vagina. The nurse immediately does which of the following?

(a) Places a gloved hand into the vagina and holds the presenting part of the umbilical cord.

(b) Transports the client to the delivery room (c) Gently pushes the cord into the vagina (d) Summons for help from other staff members

(103) A nurse working in an outpatient clinic receives a call from a 46-year-old

African-American client who is complaining of new onset painless vaginal bleeding and watery, blood-tinged vaginal discharge. Based on this information, the nurse determines that the most appropriate instruction to the client would be which of the following?

(a) “Dial 911 and come to the clinic in an ambulance.” (b) “Come into the clinic for a Papanicolou smear (c) “Keep your scheduled appointment in 2 months and call the doctor if

symptoms worsen” (d) “Wait one more menstrual cycle and call if the symptoms present.”

(104) A nurse is assigned to care for a client who is Asian (Chinese). The nurse enters

the room and following a greeting and introduction to the client, the nurse begins to discuss the plan of care for the day. The client turns away from the nurse. Which of the following nursing actions is most appropriate?

(a) Continue with the discussion (b) Ask the client if he or she can hear the nurse (c) Walk around to the client so that the nurse faces the client (d) Leave the room and return to continue with the explanation.

(105) A nurse is caring for an Asian-American client, creates a care plan that takes into

consideration the client’s view of illness. Which of the following most appropriately describes the Asian-America’s view of illness?

(a) Illness is caused by supernatural forces (b) Illness is a punishment (c) Illness is a disharmonious state that may be caused by demons and spirits (d) Illness is due to imbalance between Ying and Yang

121

(106) A 5-year-old child is brought to the clinic with iron deficiency anemia. What behavior reported by the mother would confirm this diagnosis?

(a) The child cries in his sleep (b) The child has nighttime enuresis (c) The child runs out to eat dirt (d) The child has frequent diarrhea.

(107) During your orientation as a new nurse, you are preparing a dead body for the

morgue. You observed the dead body has a thread tied around the neck and wrist. You are reminded that this is a religious practice of the

(a) Bhuddists (b) Hindus

(c) Greek Orthodox (d) Bahai Faith

(108) A diabetic client’s blood sugar is 306 this morning. After the nurse reports this

lab result and the client’s symptoms of excessive hunger and thirst, the nurse would expect the Physician to order

(a) Orange juice (b) Regular insulin (c) NPH insulin (d) Repeat blood sugar level

(109) A client is experiencing symptoms of tardive dyskenesia. The nurse would expect

to see (a) Rapid tongue movement (b) Uncontrolled hand tremors (c) Behavioral changes (d) Repetitive slapping movement

(110) A pt is admitted with a diagnosis of hypothyrodism. The nurse would expect the

Doctor to prescribe which medication for life? (a) Vitamin B12 (b) Synthroid

(c) Iron tablets (d) Prednisone

(111) A nurse reviews a pt’s lab results which indicated that the pt’s hemoglobin was

8.0. The nurse has scheduled the pt to walk in the hallway. Which action should the nurse take first?

(a) Have her dangle at the end of the side of the bed. (b) Provide oxygen before the ambulation (c) Do range-of-motion exercise before ambulation (d) Ask the pt to void.

(112) Your patient left your unit for a thyroidectomy surgery. What equipment should

keep at the bedside awaiting her return from surgery? (a) A sterile dressing kit (b) A suture removal kit (c) A petroleum gauze (d) A tracheostomy set

122

(113) A patient develops Chovstek signs after surgery. What is the essential electrolyte to monitor?

(a) Low serum calcium level (b) High level magnesium level

(c) High level calcium level (d) Low level potassium level

(114) A Roman Catholic client is terminally and is unable to talk. In order to meet the

spiritual needs of the client before death, the nurse would intervene by (a) Reading the bible to the patient (b) Invite the priest to provide the last sacrament. (c) Anoint the body with holy oil (d) Bathe the body before death.

(115) When providing feeding instructions to a mother who delivered a child with a

cleft lip, it is essential to include (a) Provide small frequent meals (b) Keep head upright when feeding (c) Refer to a speech therapist before starting feeding (d) Use elbow restraints

(116) When taking care of a child with septal defect, which observation would the nurse

report IMMEDIATELY? (a) Cyanosis (b) Patient is sleeping without breathing (c) Patient assumed a squatting position (d) Patient is crying frequently

(117) You are scheduling an immunization of a child for first MMR shot, when do you

ask the mother to bring the child to the clinic? (a) First week of birth (b) 4 weeks after birth

(c) In 12 to 15 months (d) In 4 years

(118) When completing the pre-op check list, the patient asks the nurse “What exactly

am I going to have done in surgery?” The appropriate response is: (a) To explain the procedure in non-medical terms to the patient (b) Ask the patient what the physician explained to him (c) Check the chart to see what the physician explained and report the

information to him (d) Explain that you do not know and he should ask the Physician when he sees

him before the surgery. (119) A client has received two units of whole blood today following an episode of GI

bleeding. Which of the following laboratory reports would the nurse monitor most closely?

(a) Bleeding time (b) Hemoglobin and hematocrit (c) White blood cells (d) Platelets

123

(120) The nurse is caring for a client several days following a cerebral vascular accident. Coumadin (Warfarin) has been prescribed. Today’s prothrombin level is 40 (normal range 10-14). Which of the following is a priority assessment?

(a) Neurological sign (b) Lung sounds (c) Homan’s signs (d) Gum bleeding

(121) Under which circumstance should the nurse not routinely administer the oral polio

vaccine (OPV) to an infant? (a) The father has a cardiac problem (b) The mother has a leg fracture (c) The grandmother has a flu (d) A sibling is on isoniazid.

(122) While caring for a toddler with croup, which of the following signs requires the

nurse’s IMMEDIATE attention. (a) Respiratory rate of 30 (b) Lethargy (c) Apical pulse of 54 in toddler (d) Coughing up copious secretions.

(123) A nurse has been instructed to place an IV line in a patient that has active TB and

HIV. The nurse should wear which of the following safety equipment? (a) Sterile gloves, mask, and goggles (b) Surgical cap, gloves, mask, and proper shoewear (c) Double gloves, gown, and mask (d) Goggles, mask, gloves and gown

(124) A nurse is caring for a patient who has recently been diagnosed with

fibromyalagia and COPD. Which of the following tasks should the nurse delegate to a nursing assistant?

(a) Transferring the patient to the shower (b) Ambulating the patient for the first time (c) Taking the patient’s breath sounds (d) Educating the patient on monitoring fatigue

(125) A patient that has TB can be taken off restrictions after which of the following

parameters have been met? (a) Negative culture results (b) After 30 days of isolation (c) Normal body temperature for 48 hours (d) Non-productive cough for 72 hours

124

(126) A nurse is instructing a person who had a left CVA and right lower extremity hemiparesis to use a quad cane. Which of the following is the most appropriate gait sequence?

(a) Place the cane in the patient’s left upper extremity, encourage cane, then right lower extremity, then left upper extremity gait sequence.

(b) Place the cane in the patient’s left upper extremity, encourage cane, then left lower extremity, then right upper extremity gait sequence

(c) Place the cane in the patient’s right upper extremity (127) A 64 year-old Alzheimer’s patient has exhibited excessive cognitive decline

resulting in harmful behaviors. The physician orders restraints to be placed on the patient. Which of the following is the appropriate procedure?

(a) Secure the restraints to the bed on all extremities (b) Notify the physician that restraints have been placed properly. (c) Communicate with the patient and family the need for restraints. (d) Position the head of the bed at a 45 degree angle.

(128) A 22 year-old patient in a mental health lock-down unit under suicide watch

appears happy about being discharged. Which of the following is probably happening?

(a) The patient is excited about being around family again. (b) The patient’s suicide plans has probably progressed (c) The patient’s plans for the future have been clarified. (d) The patient’s mood is improving.

(129) A 37 year-old female in room 600 has a diagnosis of acquired immune deficiency

syndrome (AIDS). Which of the following situations requires nurse intervention? (a) A certified nursing assistant states, “The patient in 600 is not wearing gloves

shaving her legs” (b) A nursing assistant at the nursing staton states, “The patient in room 600 has a

respiratory rate of 16” (c) A nursing assistant in the cafeteria states, “Dr. Jones told the patient in room

600 that she was going to die.” (d) A certified nursing assistant states, “Dr. Jones hasn’t made rounds this

morning.” (130) A 14 year-old boy has been admitted to a mental health unit for observation and

treatment. The boy becomes agitated and starts yelling at nursing staff members. What should the nurse first response be?

(a) Create an atmosphere of seclusion for the boy according to procedures (b) Remove other patients from the area via wheelchair for added speed. (c) Ask the patient, “What is making you mad?” (d) Ask the patient, “Why are you doing this, have you thought about what your

parents might say?”

125

(131) A young female teenager describes a brutal assault and rape to the nurse on duty. Which of the following actions should the nurse take first?

(a) Check with case manager on duty about possible police intervention (b) Provide an environment of concern and emotional stabilization (c) Clean the patient’s wounds with normal saline and gauze. (d) Recommend a good attorney to the patient.

(132) During the shift report, a client’s ventilator alarm is activated. Which action

would the nurse implement first? (a) Notify the respiratory therapist (b) Check the ventilator tubing for excess fluid (c) Deactivate the alarm and check the spirometer (d) Assess the client for adequate oxygenation.

(133) The nurse is reviewing a depressed client’s history from an earlier admission.

Documentation of Anhedonia is noted. The nurse understands that this symptom refers to

(a) Reports of difficulty falling and staying asleep (b) Expression of persistent suicidal thoughts (c) Lack of enjoyment in usual pleasures (d) Reduced senses of taste and smell.

(134) The physician orders a magnesium sulfate infusion for a woman with

preeclampsia. Which assessment is most important when administering this drug? (a) Monitoring the serum-magnesium level every 8 hours (b) Evaluating the apical heart rate every 4 hours. (c) Counting the respiratory rate every hour (d) Auscultating bowel sounds before meals

(135) Paracentesis is a minor surgical procedure done at the bedside; its purpose is to

remove ascitic fluid. After explaining the procedure to the patient, the next nursing action would be to:

(a) Position a patient in a chair or in a high fowler position (b) Instruct the patient to void (c) Take vital signs (d) Drape the abdomen with sterile towels.

(136) Which is a complication of thyroidectomy?

(a) Hypercalcimia (b) Respiratory obstruction (c) Elevated serum T4 (d) Paralytic ileus

(137) A client has been diagnosed to have acquired the Human Papilomma Virus(HPV).

What further laboratory test would you encourage the client to take? (a) Pulmonary function Test (b) HIV testing

(c) Pap smear (d) EKG.

126

(138) Which of these clients would be a likely candidate for breast cancer? (a) A client with multiple sexual partners (b) A client with early menarche (c) A multiparous client (d) A client with late age pregnancy

(139) Which of these clients’ occupation can lead to the development of deep vein

thrombosis? (a) A hair stylist (b) A professional boxer (c) A nurse (d) A bartender

(140) A client came to the clinic complaining of difficulty driving at night. He asked

your advice on what to do to improve his vision at night. The nurse would teach the client a non-pharmacological means of improving night vision by stating:

(a) “Eat a lot of carrots” (b) “Drink eight 8 oz of water a day” (c) “You need to be exposed to the sun” (d) “You have to reduce the fat intake”

(141) Which of these neonates condition would you have to report to the physician

immediately? (a) A neonate with jaundice 48 hours after birth. (b) A neonate with a positive Babinski reflex (c) A neonate with a heart rate of 124 beats per minute (d) A neonate with bilateral acrocyanonis in lower extremity.

(142) Which client is least likely at risk for cervical cancer?

(a) A 27 year-old woman who was sexually active at the age of i6 (b) A woman infected with the Human papillomavirus. (c) A woman who is a heavy drinker (d) A woman who uses oral contraceptives.

(143) A woman who is in 37 weeks gestation is being examined on the table

complained of shortness of breath. What action should the nurse take first? (a) Provide oxygen at 2 liters (b) Turn the patient on the left side (c) Encourage the client to void. (d) Place prone but in a high fowler position

(144) The nurse in the nursery noticed some sparks from the feeding pump cord igniting

an empty bassinette. What should the nurse do first? (a) Disconnect the feeding pump cord from the wall socket (b) Push on the call bell to summon help (c) Remove neonates to the hallway. (d) Smolder the fire with a blanket.

127

(145) What laboratory values would you monitor in a client with nephrogenic diabetes insipidus?

(a) Blood sugar levels (b) BUN (c) Urine creatinine levels (d) Urine specific gravity

(146) You responded to a code for a patient who is having massive myocardial

infarction. What position would you place the client in? (a) Semi-fowler’s (b) High fowlers (c) Tredelenburg (d) Lateral knee chest

(147) What is the immediate priority thing to check for when taking care of a client with

Parkinson’s disease? (a) Their dietary intake (b) Breathing pattern (c) Risk for falls (d) Decubitis ulcers

(148) An alcoholic patient is admitted in the hospital. He is restless and complaining

that the bed is shaking. He is making several attempts to get in and out of bed. What should the nurse do?

(a) Pull the side rails up (b) Offer him some cool drinks (c) Obtain an order for vest restraints (d) Place the patient’s mattress on the floor.

(149) A patient with gastroenteritis is vomiting a green bile substance. What lab values

would you monitor? (a) Biluribin (b) Uric acid (c) BUN (d) Hematocrit

(150) What discharge teaching should the nurse give to the parent of a child with cystis

fibrosis who is being discharged from the hospital? (a) Limit exposure to the sun (b) Monitor for skin breakdown. (c) Child would take some medicine for rest of life (d) Illness would disappear during adulthood.

(151) When a client has a generalized tonic-clonic seizure, the nursing priority is to

(a) call the code for cardiac arrest (b) hold the client’s extremities down (c) protect the client from immediate injury and allow free movement (d) place an oxygen mask on the client

128

(152) The nurse is caring for a client with a head injury. The nurse should be alert for signs of increased intracranial pressure which include

(a) A change in level of consciousness (b) Bilateral pupil size increase (c) Narrowing pulse pressure (d) Tachycardia

(153) A client returns from a cardiac catheterization with a bandage over the right groin.

The nursing assistant assigned to the client reports that the client’s right foot is cool to touch. The first nursing action of the nurse should be to

(a) Check the foot every 15 minutes (b) Call the provider and report the findings (c) Loosen the bandage (d) Check the client’s groin

(154) The nurse knows that when assessing a client suspected of having a myocardial

infraction, it is most important to ask which of the following? (a) “What medications are you currently taking?” (b) “Have you ever had similar symptoms in the past?” (c) “How long ago did these symptoms start?” (d) “Have you ever been under a lot of stress lately?”

(155) The nurse is teaching a parent whose teenage son has hepatitis A. The nurse

teaches the mother that the best way to avoid spread of infection is (a) Wearing a mask (b) Wearing gloves (c) Hand washing (d) Wearing a gown

(156) When a nurse is giving discharge teaching instructions to a client with

pyelonephritis, information on how to prevent reoccurrence should be given. This would include information

(a) Maintenance of perineal hygiene (b) Discontinuing antibiotic therapy when the client feels better (c) Limiting fluid intake to 1 liter a day (d) Monitoring urine output

(157) A client is 1 day postoperative after a total hip replacement. The client should be

placed in one of these positions for meals. (a) Supine (b) Semi-Fowler’s (c) Orthopneic (d) Trendelenburg

(158) Using the rule of nines, the nurse determines the percentage of the body burned

on a client with both arms burned to be (a) 9% (b) 36%

(c) 18% (d) 1%

129

(159) A child who has been burned is brought to the emergency room. What is the first nursing priority when dealing with burns in children?

(a) Inserting an IV (b) Inserting a Foley catheter

(c) Comforting the child (d) Maintaining a patent airway

(160) The nurse caring for a diabetic client knows that which of the following clinical

manifestation distinguishes a hypoglycemic reaction from ketoacidosis. (a) Blurred vision (b) Diaphoresis

(c) Nausea (d) Weakness

(161) Which of these precautions should staff include in the care of a client who is

receiving a sealed source of radiation therapy for uterine cancer? (a) Limiting a staff member’s time with client to less than 30 minutes per shift (b) Keeping a zinc-lined container and forceps in the client’s room. (c) Instructing visitors to maintain a distance of about 3 feet from the client (d) Reminding the client to flush the toilet several times after each use.

(162) Assessing the patient following abdominal surgery, the nurse observes pinkish

fluid and a loop of bowel through an opening in the incision. The first nursing action is to:

(a) Notify the physician (b) Notify the operating room for wound closure (c) Cover the protruding bowel with a moist, sterile, normal saline dressing (d) Apply butterfly tapes to the incision area.

(163) Assessing a patient who has developed atlectasis postoperatively, the nurse will

be most likely to find (a) A flushed face (b) Dyspnea and pain (c) Decreased temperature (d) Severe cough with no pain.

(164) A nurse is assigned to take two patient’s vital signs, complete a focus assessment

and provide hygiene care, administer meds, and complete a dressing change for a patient with an abdominal wound. Which task will have a priority with this assignment?

(a) Complete a focus assessment and provide hygiene care on the first patient. (b) Administer medications to the patient (c) Complete the dressing change (d) Take vital signs on the two patients

(165) The nurse accidentally has had a needle stick in her hand as she pulled an IM

needle from the muscle. The first action is to (a) Report the accident to the charge nurse (b) Scrub the area vigorously with soap and water for 5 minutes (c) Cleanse the area with povidone-iodine (Betadine) (d) Irrigate the wound with sterile water.

130

(166) The nurse is just exiting an isolation room. Considering infection control protocol, which action would the nurse take first?

(a) Bag equipment and double-bag it at the door. (b) Remove protective gear (c) Dispose of equipment appropriately inside the room (d) Wash hands

(167) What behavior may be observed when a preschool child is hospitalized?

(a) Temporal regression (b) Pretend to be sicker to get parents’ sympathy (c) Increase in temper tantrums (d) Refusal to follow nurse’s instructions

(168) A client is receiving an IV solution of 500cc of Normal Saline. If 700 units of

regular insulin is added to the solution, how many units is the client receiving per cc.? (a) 15 units (b) 1.4 units (c) 1.75 units (d) 0.5 units

(169) A female client came to the clinic reporting a possible gonorrhea infection. Which

assessment will assist the nurse in confirming the diagnosis? (a) Painful urination (b) Elevated bodily temperature (c) Purulent vaginal discharge (d) Hematuria

(170) A client with type I diabetes and taking insulin verbalizes her frustration by

saying, “Can’t I take a pill instead of these painful injections?” The nurse’s best response is

(a) “The oral insulin is not as effective as the injection forms.” (b) “You would get used to the pain after a while.” (c) “Insulin does not come in pill form.” (d) “The Z track method lessens the pain.”

(171) A patient is accidentally hit on the head with a baseball bat and sustains a head

injury. How would you assess his neurological status? (a) Ask him to open his eyes (b) Ask him his name (c) Assess his jugular vein (d) Assess his pupil size

(172) The nurse collects all the following data, which one should be reported

immediately? (a) A six year-old complains of nausea (b) 21 year-old with no bowel sounds 12 hours post surgery (c) A child refuses to eat (d) A child with Tetralogy of Fallot who is in a ‘Tet spell”.

131

(173) When a pregnant woman visits the clinic at her 30th week of pregnancy, the nurse makes all the following observations. Which one needs further investigation because it is not an expected finding at this stage of pregnancy?

(a) She has gained 3 lbs since her last visit to the clinic 4 weeks ago (b) The fetal heart tones are now heard in the upper right quadrant of the

abdomen; 4 weeks ago they were heard in the lower right quadrant of the abdomen

(c) She says she has noticed that her vaginal discharge is pink tinged (d) Her uterine fundus is fingerbreadths above the umbilicus.

(174) When the fetal head is well engaged, the doctor will rupture the bag of water in

order to: (a) Avoid using difficult and painful instruments (b) Ensure that a small amount of amniotic fluid would be left in the amniotic sac (c) Prevent prolapsed of the umbilical cord during the forceful expulsion of

amniotic fluid from the sac (d) Have the amniotic sac in a dependent position so that it can be reached easily.

(175) A client attends the clinic when she is tow weeks postpartum. The nurse obtains

the following data about her. Which finding definitely requires further investigation? (a) When she was admitted to the labor room, she weighed 190 lbs, and now she

weighs 177 lbs. (b) The uterine fundus is not palpable (c) She has small amount of lochia rubra (d) She has pink abdominal striae

(176) A client tells you that at times she wishes she were dead. Which of the following

would be your first intervention? (a) Call the doctor and check out for suicidal ideation. (b) Further assess the suicidal thoughts and plans by asking questions. (c) Assign a 1:1 to stay with the client (d) Have the client agree that she won’t commit suicide

(177) The nurse assesses a client who has been re-admitted to the Psychiatric-in-patient

unit for Schizophrenia. His symptoms have been managed for several months with Fluphenazine (Prolixin). Which of the following should be the FIRST assessment?

(a) Stressors in the home (b) Medication compliance (c) Exposure to hot temperature (d) Alcohol use

(178) You are setting up to do sterile wound dressing. How would you open up a sterile

package? (a) It does not matter so far as you do not touch the inside contents (b) Open the first flap towards you (c) Open the side flaps first (d) Open the last flap towards you.

132

(179) When taking care of a patient with Lou Gehrig disease, the initial nursing priority of care would focus on

(a) Altered nutrition (b) Altered though process

(c) Altered skin integrity (d) Airway breathing impaired

(180) You entered a room occupied by two adolescents; one has started a fire in trash

can, what would be your first action to take? (a) Run to the nursing station for help (b) Remove the two clients from the room (c) Activate the fire alarm system (d) Attempt to quench the fire with a blanket

(181) Which of these actions would be considered a breach of patient’s confidentiality?

(a) Placing patient’s diagnosis at the front of patient’s chart (b) Talking to patient’s wife on the telephone about his condition (c) Answering patient’s question about anti-viral medications in front of his

visitors (d) Faxing patient information accidentally to a wrong number after patient has

given consent. (182) Among the assessment of the newborn that you must do is to test for Babinski

reflex. How do you elicit the Babinski reflex? (a) Stroke the cheek (b) Put on a blood pressure cuff (c) Shine penlight in the eye (d) Stroke lateral side of foot from heel to toe

(183) A pregnant woman with 38 weeks gestation is observed choking; what would be

the appropriate nursing action to take? (a) Perform a Heimlich maneuver (b) Quickly do a mouth sweep (c) Perform a chest thrust (d) Give back thumps like you’ll do an infant

(184) A 4 month-old baby came for OPV vaccination; what further assessments will

you need? (a) Allergy to cow milk (b) Cried a lot after the last OPV (c) Baby sitter’s daughter has leukemia (d) Had redness at the injection site of the last OPV

(185) You’re doing a physical for a client whose family member has been diagnosed

with TB. Which of the following questions would be appropriate to ask? (a) Do you sweat a lot at night? (b) Do you sometimes feel as if you can’t exhale completely? (c) Do you sometimes cough up green-tinged sputum? (d) Do you have any reddish lesions on your body?

133

(186) A Holter monitor is a device used for monitoring the activities of the heart. What important instructions would you give to a patient going home on a Holter monitor?

(a) “Stay in bed while the monitor is in place.” (b) “Take off monitor one hour for hygiene purposes only.” (c) “Keep a diary to record major activities.” (d) “Avoid having sex.”

(187) A nurse wants to logroll a patient with a hip replacement surgery. What procedure

should be avoided? (a) Ask for assistance of 2 nurses (b) Abduct both legs with pillow (c) Put patient’s hand/arms across the chest (d) Removing pillow between legs after moving

(188) The practical nurse is teaching a male patient; what is the best time to do

testicular examination? (a) Post menstrual period (b) Early in the morning

(c) During shower (d) Whan he has an erection

(189) On the morning a client is scheduled for surgery, the nurse enters the room. The

nurse notices the client is crying. Which of these responses by the nurse would be most appropriate?

(a) “Good morning. Why are you crying?” (b) “I see that you need some private time. I’ll come back in five minutes.” (c) “It seems that you are crying. Can you tell me what you are feeling?” (d) “You don’t need to worry. The surgery will be over in a few hours.”

(190) A client who is Roman Catholic dies on your shift. What preparation should you

make about the body before it is taken to the morgue? (a) The body needs to be washed by a rabbi (b) A priest would offer prayers (c) The hospital chaplain can baptize the corpse (d) The family should be the only one touch the dead body.

(191) When preparing to give a client a bed bath, nursing action should include:

(a) Placing the bed in the lowest position (b) Exposing the top side of the body, washing and drying quickly, then doing the

same on the posterior side (c) Gathering all the articles necessary for the bed bath and placing them within

easy reach of the nurse (d) Using firm, scrubbing strokes to remove dirt and bacteria.

(192) The nurse is admitting a client suspected of having diabetes mellitus. If the client

does have diabetes mellitus, the nurse would expect to find: (a) Shallow, labored respirations (b) Increased blood pressure associated with slight periorbital edema. (c) Periods of altered pulse rate (d) Increased urinary output

134

(193) A newly diagnosed adult diabetic is demonstrating of the proper technique for

insulin injection. The client draws up the correct dose of insulin using the proper technique, but when ready to inject the needle, hesitates and says, “I’m not sure I can do this.” Which response by the nurse would be best initially?

(a) “I’ll show you again how to inject the needle.” (b) “I’ll inject the needle for you this time.” (c) “You’re doing fine so far. Give it a try.” (d) “Why are you so nervous? Do you need help?”

(194) A pregnant woman of 18 weeks gestation was observed to have a Chadwick sign.

What characteristics of this sign should the nurse observe? (a) A bluish and purple discoloration of the cervix (b) A softening of the cervix (c) A fetal movement observed (d) Auscultation of fetal heart tones.

(195) You are providing instructions on health maintenance for a client with essential

hypertension. What statement made by the client would indicate understanding of the illness?

(a) I will walk to the shop anytime I need a newspaper (b) I will limit the number of times I have sex. (c) I will check my pulse before taking my medication (d) I report to my doctor immediately I feel any symptoms of illness.

(196) You are taking care of a client who undergoes hemodialysis three days a week.

What is essential to assess before and after each dialysis session. (a) Signs and symptoms of hemolytic reaction (b) For a thrill in the shunt (c) Anemia (d) Cardiogenic shock

(197) The nurse enters a room and finds the adult client holding the neck and cannot

speak. What action would the nurse take next? (a) Call for a code (b) Bring the emergency cart (c) Check for airway (d) Perform a Heimlich maneuver.

(198) How would you administer an ear medication to an adult client?

(a) Pull the auricle back and up (b) Pull the auricle down and back (c) Push the auricle up and back (d) Pull the auricle up and back

135

(199) What would a patient who is to have paracentesis done say to make the nurse know that he understands the procedure?

(a) “I will be in a side lying position” (b) “I will be put in a sitting position” (c) “I would have to maintain a full bladder” (d) “I would have to assume a knee-chest position”

(200) A woman brought her infant child who has RSV. She stated that “I am afraid that

my older child would catch the disease also. What would be your appropriate response based on your understanding of RSV infection?

(a) “Only infants have RSV” (b) “The older child would be protected if he has had HIB immunization”. (c) “If the older child has RSV before, then he has acquired immunity” (d) “The older child would not be infected if he washes his hands after touching

the infant” (201) What is an antidote to Heparin sulfate?

(a) Potassium sulfate (b) Protamine sulfate

(c) Calcium sulfate (d) Iron sulfate

(202) What would a patient with diabetis insipidus state to make the nurse know that

she understands patient teaching on her condition? (a) “I should exercise every other week” (b) “I should be expecting to gain weight (c) “I should monitor my diet” (d) “I should monitor urine specific gravity”

(203) A patient returned from surgery and on arrival on the unit the nurse saw an

evisceration from the incision site. What would the nurse do first? (a) Keep the patient NPO (b) Call the Physician immediately (c) Prepare the patient for surgery (d) Cover wound with sterile dressing.

(204) A patient was observed hyperventilating; which of these describes Kussmaul

respiration? (a) A shallow fast breathing (b) Slow breathing with periods of apnea (c) Deep, sighing respiration (d) Respiration marked by wheezing

(205) A patient is on Epogen. What would the patient say to indicate an understanding

of the correct use of this medication? (a) “It helps to produce more red blood cells for me” (b) “It controls my blood sugar” (c) “I need it to help me sleep” (d) “It helps to relieve my stomach cramps”

136

(206) Which of the following has the highest source of potassium? (a) A slice of tomato (b) One large baked potato

(c) Two pounds of beef (d) Two sticks of carrots.

(207) A patient who is on oxygen therapy rubbed petroleum jelly on the mouth. Which

statement made by the nurse is correct? (a) “Petroleum and oxygen would cause static electricity” (b) “Petroleum works well with oxygen” (c) “Oxygen would stick to the petroleum” (d) “Oxygen is converted to water in the presence of the petroleum”

(208) A neonate is receiving Vitamin K after birth. Why does the neonate need vitamin

K? (a) To build up liver function (b) To be able to clot during a bleeding episode (c) To prevent jaundice (d) To prevent bleeding injuries.

(209) The nurse is giving discharge teaching to a post-partum woman about lochia.

What statement by the woman would indicate the teaching has been effective? (a) “Clots in the lochia is normal” (b) “I should see bright red blood on day seven” (c) “I can take hot sitz baths” (d) “A fleshy odor of lochia is normal’

(210) What statement made by a type1 diabetic would indicate that he needs more

teaching? (a) “I will eat in the local restaurant every day” (b) “I will eat more complex carbohydrates instead of simple carbohydrates. (c) “I will add artificial sweetener to my food instead of sugar” (d) “I will limit my daily meals to two times a day.”

(211) A woman who delivered by C-Section was receiving epidural IV infusion. When

is the appropriate time to discontinue the epidural? (a) When she has fully recovered. (b) One hour after delivery (c) When she says she has abdominal cramps (d) When she says she feels no post-partum pains

(212) The nurse has organized a discussion session for nursing assistants at an extended

care facility about cultural and religious rituals at the time of death. One of the participants has a misconception when the nursing assistant states that:

(a) Jews wash the deceased and dress the body in a simple shroud (b) Islamics bury the deceased body within 24 hours of death (c) Wailing is a socially acceptable sign of respect in the African-American

culture (d) Organ donation is strictly forbidden by the Baptist church.

137

(213) How would the nurse know that the disease is progressing for an Alzheimer patient (in stage II of the disease)?

(a) Cannot remember the meal she ate 1 hour ago (b) Cannot remember husband’s name (c) She states she is on Mars (d) She does not know what a toothbrush is used for.

(214) The nurse knows that discharge teaching is successful for a client with Hepatitis

A virus if the client indicates she will refrain from (a) Donating blood (b) Eating fried foods (c) Vacationing in a foreign country (d) Ordering a salad in a restaurant

(215) In planning a diet for a patient with Phenylketouria, the nurse knows that diet

therapy will be based on the knowledge that the nutrients to be controlled are: (a) Fats (b) Minerals

(c) Proteins (d) Carbohydrates

(216) A 2 year-old girl is brought to the well –baby clinic for check-up. The average

vital signs for a 2 year-old should be: (a) Heart rate-110, R-25, BP-96/68 (b) Heart rate-120, R-35, BP-70/50 (c) Heart rate-80, R-19, BP-110/70 (d) Heart rate-100, R-21, BP-105/70

(217) A client whose newborn has been diagnosed with Down syndrome asked the

nurse how retarded will my baby be. What will he be able to do? What would be the appropriate response by the nurse?

(a) “Only genetic counseling would determine the extent of his disability”. (b) “Most children with Down Syndrome eventually become self sufficient” (c) “It is not possible to determine the extent of the child’s disability at this time” (d) “Your child would need total care for the rest of his life”

(218) An 18 month-old child has been placed in Bryant traction. The traction has been

properly applied. What should the nurse observe? (a) Both legs are at 35 degree angle. (b) 10 pounds weight is applied (c) The ropes are slack (d) Both buttocks are off the bed.

(219) A 5 year-old was admitted to the hospital for a diagnostic studies, the nurse

expect a child with Tetralogy of Fallot to exhibit which of the following clinical manifestation?

(a) Skin with bluish tint, child favoring squatting position. (b) Skin with a natural pink color, clubbing of the finger and toes (c) Retarded physical growth, but usually normal in intelligence (d) Difficulty breathing, requiring oxygen therapy frequently.

138

(220) A nurse is preparing to give (NPH) insulin and notices that the medication is

cloudy, which of these actions is indicated? (a) Dilute the insulin with normal saline (b) Discard the insulin and get another vial. (c) Shake the vial of insulin vigorously (d) Rotate the vial of insulin before drawing it up into the syringe.

(221) The nurse was interviewing a 46 year-old war veteran who is troubled with

flashbacks of his war time experiences, having difficulty sleeping at night, reports problems on his job and experiencing problems in his marriage. The nurse is aware that this client is likely suffering from

(a) Chronic pain disorder (b) Delusional disorder (c) Post-traumatic stress disorder (d) Hypochondriasis (222) A patient is diagnosed with Raynaud’s disease. When providing instructions to

prevent reoccurrence of the discomfort associated with the disease, the nurse should advise the client to:

(a) Avoid exposure to the sun (b) Avoid exposure to cold and stress (c) Take Motrin instead of Tylenol (d) Minimize your alcohol intake

(223) You are a home health nurse who visited a patient who colostomy surgery 1 week

ago. What should you observe to indicate that the colostomy is healthy and healed well?

(a) The colostomy is red, moist and beefy (b) There colostomy is dark in color and painless (c) The colostomy has a small liquid drainage seeping out. (d) The colostomy has a fecal odor.

(224) What vaccinations would you give to a 2 month baby at the well-baby clinic?

(a) MMR, OPV, DTP (b) DTP, OPV, HIB (c) Hep B, DTP, HIB (d) Hep B, DTP, OPV, HIB

(225) A mother of a child who has a cast on the leg calls to find out if he can use a blow

dryer to dry the cast quickly. What would be the nurse’s response considering principles of cast care?

(a) “No use a heating lamp instead” (b) “No, let it air dry” (c) “Yes, the quicker it dries the better” (d) “Yes, use the palm of your hand to handle cast.”

139

(226) A patient scheduled to do EEG asked you what the test is for. Your response would be?

(a) It used to test for muscle strength (b) It is used to test for fetal heart tones (c) It test how your heart is working (d) It test for brain activities

(227) Which statement by a patient with Hiatal Hernia would indicate an understanding

of patient teaching to help minimizing the discomfort of the disease? (a) “I will drink a lot of water with my meals” (b) “I will sit upright for 30 minutes at least after eating” (c) “I would eat foods high in fiber” (d) “I would avoid taking alcohol”

(228) A patient went for a surgery to place an ileal conduit. What would the nurse be

monitoring on return? (a) Urinary output (b) Fecal output (c) Food intake (d) Sodium levels

(229) A pt is taking Chlopromazine (Thorazine); which statement made by the patient

would indicate effectiveness of drug therapy? (a) Pt states “The voices are not as loud as they used to be” (b) Pt states “My spirits are up, I do not feel like dying anymore” (c) Pt states “I do not feel anxious, I can go out to public spaces” (d) Pt states “I have a large surge of energy”

(230) Which of these would be considered side effects of Haldol (Haloperidal)?

(a) Constipation (b) Dystonia (c) Chills and fever (d) Bronchospasms

(231) A patient is receiving calcium sulfate IV; what side-effects should the nurse

expect the patient to have? (a) Pt may have constipation (b) Pt may have diarrhea (c) Pt would have increase in appetite (d) Pt may loose appetite

(232) A nurse enters a room of a pt with DVT and the pt complains of chest pain and

cough. What will the nurse do first? (a) Raise the bed to semi-fowler position (b) Administer pain meds PRN as ordered (c) Check the cough for blood stain. (d) Give oxygen at 2 liters as ordered.

140

(233) A pt was diagnosed with angina and heart failure and was prescribed Lasix, Digoxin and Nitroglycerin. What teaching will you give to the patient?

(a) Take the medication at one time in a day (b) Ensure you go out with medication list/paper. (c) Put nitroglycerin sublingual in water for easy absorption (d) Take the medications at least one hour apart.

(234) What is the sign that the medication is working for someone on Levonox?

(a) Pt is complaining of gum bleeding (b) Pt states his energy has gone up (c) Pt complains of no more abdominal pain (d) Pt states that the itching has resolved.

(235) A pt is prescribed Asprin (Salycilates). How will he use it?

(a) With meal (b) Without meal (c) Between meal (d) Use it with Tylenol.

(236) What food would the nurse recommend for a patient with Diverticulitis

(a) Tomatoes and Raspberries (b) Mashed potatoes and French fries (c) Prunes and peas (d) Candy yam and bread.

(237) In which religion must the family arrange for the burial of the patient?

(a) Islam (b) Hindu (c) Bhuddist (d) Judaism

(238) In which religion does somebody stay with corpse after death?

(a) Judaism (b) Bhuddism

(c) Hindu (d) Islam

(239) How would the nurse assess that a patient with Total Hip Replacement

understands discharge teaching? (a) ‘I will raise my toilet seat at home.” (b) “I will not sit down in a chair for more than 2 hours” (c) “I will do a range of motion exercise to prevent contractures (d) “I understand that I have to be on bed rest for several weeks.

(240) Which statement made by a diabetic client needs a follow-up?

(a) I have to cut my toenails straight across (b) I strain when I pass stool (c) I do not have to go outside for a walk frequently (d) I have to eat more complex carbohydrates.

141

(241) How would the nurse position a patient who is 2 day post-operative hip replacement surgery for feeding?

(a) Place in a wheel chair when eating at the dining room table. (b) Place in a recliner with a food table attached. (c) Place in a high-fowler’s position in bed. (d) Keep side- lying to prevent aspiration.

(242) When a nurse signs an informed consent, what does it mean?

(a) To confirm that the doctor has explained the procedure to the patient. (b) That the patient understands the procedure. (c) That the patient can withdraw his signature after signing the informed consent

form (d) That the nurse can be a witness in court in case of a law suit.

(243) The nurse has a doctor’s order to do an Alpha-Feto Protein test on a woman in 18

weeks gestation. How is the nurse going to conduct the test? (a) By amniocentesis (b) By blood draw (c) By pap smear (d) By urine test.

(244) How often would the nurse do range-of-motion (ROM) exercises for a person

with a CVA? (a) Three times a day (b) As frequently as possible in a day (c) Increasing the number of times gradually day by day (d) Changing the direction of the exercise each day.

(245) What food would the nurse recommend for clients with Osteoporosis?

(a) Foods rich in calcium (b) Foods low in calcium (c) Foods rich in potassium (d) Foods low in potassium

(246) How would the nurse know that a patient teaching given to a patient with

Osteoarthritis is understood? (a) The patient states: “I will do ROM to the affected extremity” (b) The patient states: “I will not bear weight on my joints” (c) The patient states: “I will pour hot water on it” (d) The patient states: “I will do all my work in the morning”

(247) What advice do you give to a patient with multiple sclerosis who has had an

exacerbation of the disease? (a) Encourage bed rest (b) Avoid stress (c) Eat low protein diet (d) Disease gets better in the morning, gets worse in the day.

142

(248) Which food would the nurse give to a patient with WBC count of 2500? (a) Sliced apple (b) Tossed salad

(c) Milk shake (d) Skim milk

(249) Who would be a good room mate for a client with multiple myeloma?

(a) A pt with hepatitis A (b) A pt with tuberculosis (c) A pt with Alzheimer’s disease. (d) A pt with AIDS

(250) How would the nurse recognize that a patient has Hepatitis A.?

(a) Clay colored stool (b) Dark colored urine (c) Complaint of flank pain (d) Complaint of rashes on chest.

(251) A pt complains of dribbling, scant and frequent urination. What examination

would you recommend for the patient? (a) Intravenous pyelogram (b) 24 hour urine collection (c) Rectal examination (d) In- and-out catheterization.

(252) Which of these would be developmental accomplishment of a two-year-old?

(a) Says “mama” with meaning (b) Understands why he is punished (c) Can ride a tricycle (d) Can turn a doorknob.

(253) A CNA reported to you that a pt with Huntington disease could not eat because

his hands were shaking so badly. Who would you report pt’s condition to for follow-up?

(a) An occupational therapist (b) A neurologist

(c) A physical therapist (d) A nutritionist

(254) A pt is being discharged after TURP. What statement would indicate that pt’s

discharge teaching has been successful? (a) I would be on complete bed rest for a week. (b) I would drink a large amount of fluids to flush out blood clots. (c) I would check my groin for bleeding (d) I would not have sex for one year.

(255) A pt is demonstrating to the nurse the use of insulin. Which action indicates the

patient understands teaching? (a) Pt wipes the top of vial with hydrogen peroxide (b) The pt shakes the vial to mix solution well (c) Pt states “I will take my insulin even when I am ill and have not eaten” (d) Pt states “When I exercise, I have to reduce the amount of insulin I take”

143

(256) The nurse finds someone in the floor of an elevator. What would the nurse do? (a) Check for unresponsiveness and start CPR (b) Start CPR immediately. (c) Check for unresponsiveness, call medical team and start CPR (d) Call medical team and start CPR.

(257) A young-adult client is scheduled for her first debridement of 2nd degree burn of

the left arm. Which action is most important? (a) Assemble all necessary supplies and medications. (b) Plan adequate time for dressing change and provide support. (c) Prepare family for pain during the procedure. (d) Limit visitors prior to the procedure to limit stress.

(258) A client who is gravida 2, para 1 has been admitted for induction of labor with

Pitocin. Which action would be most important (a) Mix Pitocin in D5W at 5mg/cc to flow by gravity. (b) Decrease the rate if fetal heart rate is below 150. (c) Piggyback into mainline IV and maintain flow by gravity. (d) Start an IV line with an infusion pump

(259) A nurse is performing a discharge teaching for a client with Multiple Sclerosis. It

is most important for the nurse to include which of the following instruction? (a) Ambulate as tolerated everyday (b) Avoid overexposure to heat or cold. (c) Perform stretching and strengthening exercise (d) Participate in social activity.

(260) The pt’s lab values from a CBC read thus: sodium (130), potassium (3.52), WBC

(5002), and platelets (160,000). Which of these lab values would you report immediately?

(a) The sodium (b) The potassium

(c) The calcium (d) The platelets

(261) Which of the following findings indicate that a salem sump tube (nasogastric) was

functioning properly? (a) Fluctuation of the fluid in the chamber. (b) Active bubbling in suction bottle. (c) The presence of hissing sound from tube lumen (d) A pressure of 25mm in esophageal balloon

(262) A client in labor is receiving Magnesium sulfate IV. Which assessment must the

nurse do? (a) Vaginal examination (b) Patellar reflex (c) Chovstek’s sign (d) Chadwick sign

144

(263) What instruments would be provided at the bedside of a patient on Magnesium sulfate IV?

(a) reflex hammer (b) tongue blade (c) pulse oximeter (d) heat blanket

(264) A pt was recently diagnosed with Glaucoma. What symptoms would the pt

report? (a) Peripheral vision loss (b) Blurred vision (c) Curtain drawn in field of vision (d) Photophobia.

(265) What teaching would you give to a patient on Acyclovir?

(a) Continue to use protection during sex while on this drug (b) Do not get pregnant while on this drug (c) Monitor your blood sugar levels when on this drug. (d) Avoid long exposure to the sun.

(266) The most appropriate candidate for a PCA use is:

(a) A toddler but with parents present (b) A nonresponsive client with head injury. (c) An elderly client with Alzheimer’s disease (d) A postoperative alert client.

(267) A patient is talking to you and stops all of- a- sudden. You shake the patient and

do not get a response. What should you do next? (a) Assess for orientation to name, place and time. (b) Check carotid pulse (c) Start chest compressions (d) Provide three quick breaths.

(268) You observed a child at a playground experiencing epitasis. What action by the

nurse is appropriate at this time? (a) Pinch the nose and lean the head back. (b) Let the child sit down and lean head forward. (c) Let child stand up and tilt head to the side. (d) Let child lie down quietly in a prone position.

(269) You were assessing a pt that is two day post-thyroidectomy. Which finding

would you follow up? (a) Pt complains of difficulty swallowing (b) Pt’s exhibits hoarseness of voice (c) Pt complains of muscle spasms (d) Pt complains of blurred vision.

145

(270) An adolescent came to the hospital to be circumcised. The patient will indicate that he understands discharge teaching if he reports;

(a) Yellowish discharge around the penis (b) States: “I will not have sex for 6 weeks” (c) My penis would be tender or sore for about a month. (d) Circumcision can lead to priaprism.

(271) Pt with herpes Zoster (shingles) understand teaching when he states: (a) “I will apply an emollient to lesions after bathing” (b) “I will not go to work until rash disappears” (c) “The rashes will go away even without treatment” (d) “I should request for pain medicine.”

(272) A pregnant woman with 37 weeks gestation is lying on the examining table and

suddenly complained of shortness of breath. What would the nurse do first? (a) Provide oxygen (b) Turn patient to the left side (c) Place in shock position (d) Place in Lithotomy position

(273) A pt on G-tube feeding has a prescribed medication Procardia XL (extended

release). How will the nurse administer this medication? (a) Give after feeding (b) Give half of feeding and give medication; then continue feeding (c) Crush the medication and give in 60cc of water. (d) Report to physician to give you the liquid form of the medicine.

(274) What vital sign is important to check before administering Atropine?

(a) Pulse (b) Blood Pressure

(c) Respiration (d) Temperature

(275) What are the side effects of Atropine?

(a) Dry mouth, tachycardia, (b) Excessive salivation, drowsiness (c) Drowsiness, hypotension (d) Urinary retention, renal failure.

(276) Where would you check a dark-skinned person for Jaundice?

(a) Below the sclera of the eye (b) Nail beds (c) Buccal mucuosa (d) Ear lobe

(277) Your client reports suffering from night blindness. Which food will you

recommend? (a) Oatmeal (b) Toasted wheat bread

(c) Egg yolk (d) Baked potatoes.

146

(278) A pt is scheduled for duodenoscopy. What preparations will you do for the pt towards the procedure?

(a) Give enema the night before the procedure (b) Take fecal sample before procedure (c) Provide ice chips before the procedure (d) Place on NPO status 6-8 hours before procedure

(279) How would the nurse determine that a pt taking Vitamin A has reached the toxic

levels? (a) Pt would report night blindness. (b) Pt would complain of dysphagia. (c) Pt would complain of dry and cracked lips. (d) Pt would complain of constipation.

(280) An elderly client is seen as frequently disoriented when he wakes up from sleep.

How would the nurse deal with this problem? (a) Place call bell within reach of the patient (b) Stay with the pt until oriented and fully awake. (c) Restrain pt. to prevent falls. (d) Put side rails up till oriented.

(281) A pt with 2nd and 3rd degree burns is receiving morphine. The nurse notes a

decrease in bowel sound and slight abdominal distension. Which action of the nurse is best?

(a) Recommend a decrease in the morphine. (b) Withhold the pain medication. (c) Administer by another route. (d) Explore alternative pain management technique.

(282) A pt is being discharged on Nitroglycerin. Which statement by pt indicates he

needs more pt teaching? (a) I‘ll take medication before I undertake any strenuous exercise. (b) I’ll take medicine until my pain stops. (c) I should expect a tingling sensation in my mouth when I take medication. (d) Headache is a common side effect of this medicine.

(283) Which of the following findings indicate that a salem sump tube (nasogastric) was

functioning? (a) The fluctuation of fluid levels in the chamber. (b) Active bubbling in suction bottle. (c) The presence of hissing sound from the lumen of tube (d) A pressure of 25mm in esophageal balloon.

(284) What advise would you give to a client who is taking iron pills to minimize the

side effects? (a) Take the medication with

milk (b) Take with full glass of water

(c) Take with an antacid (d) Take with small amount of

red wine.

147

(285) The nurse is preparing to do sterile wound dressing. Which of the following steps

is appropriate? (a) Pour the solution in a sterile container. (b) Pour 10cc. of the solution into the sink first. (c) Pour solution on sterile 4X4 gauze before applying. (d) Drop dressing kit on a sterile field.

(286) You have a doctor’s order to give 375mg of Ampicillin when it is supplied as

250mg/5ml. How many ml(s) would you give to the patient? (a) 10ml. (b) 15ml. (c) 7.5ml. (d) 5ml.

(287) Which one of these actions by the nurse would generate an incident report?

(a) The nurse gave the elderly pt a shower without checking the temperature of the water?

(b) The nurse cutting a diabetic pt’s toe nails and mistakenly cut the toe and it started bleeding.

(c) The nurse put a violent patient in a seclusion room. (d) The nurse forgets to give a pt his medication on time.

(288) In a skilled nursing home, you saw a male and a female with osteoporosis kissing

in a room. What would you do? (a) Stop them from kissing and send them to the day room. (b) Report incident to the nursing supervisor. (c) Contact family members about behavior. (d) Close the door to the room and provide privacy.

(289) What would you tell a patient with fecal impaction when removing the impaction?

(a) Tell pt to hold his breath (b) Tell pt to rest when he feels uncomfortable (c) Use a catheter (d) Take a suppository before the procedure.

(290) The nurse should expect a patient with diagnosis of borderline personality

disorder to show one of the following behavior or characteristics; (a) Hallucination (b) Idea of reference (c) Autism (d) Manipulation

(291) Which characteristic is usually associated with someone with major depression?

(a) Low self-esteem (b) Paranoia (c) Dementia (d) Delusions

148

(292) How would the nurse perform Crede’s Maneuver?

(a) By range-of motion exercise of the thigh (b) By manual compression of the bladder (c) By placing the pt in trendelenburg positon. (d) By rolling the pt from side to side

(293) A pt is scheduled for Oopherectomy. The pt asks you what the procedure is

about. The appropriate nurse’s reply would be: (a) “It is to remove a tumor in your abdomen” (b) “It is a surgery to repair a valve your heart” (c) “It is similar to laparoscopy” (d) “It involves the removal of the ovary”

(294) What will a pt say to indicate understanding of tubal ligation?

(a) “I will still continue to have my menses” (b) “I can no longer have menses” (c) “I can tell my partner there is no need to use condoms” (d) “I can no longer enjoy sex”

(295) Some nursing home patients have an appointment with an ophthalmologist.

Which of these residents would likely go to see the doctor? (a) A resident with diabetes (b) A resident lupus erythematosus (c) A resident with Alzheimer disease. (d) A resident with bunions on the feet.

(296) When a nurse witnesses an informed consent form, what does it imply?

(a) To confirm that the doctor has explained the procedure to the patient. (b) That the patient understands the procedure. (c) That the patient can withdraw his signature at any time. (d) That the procedure is not dangerous.

(297) What protective devices must the nurse wear when taking care of a pt with

Meningitis? (a) Mask (b) Mask and gloves (c) Mask, gloves and gown (d) Mask, gloves, gown and goggles.

(298) Which statement correctly describes a 7 year-old child understanding of the

concept of death? (a) Has no understanding at all (b) Understands that death is sleep and sleep is death. (c) Death is seen as a punishment for “badness” (d) Death is a cessation of life.

149

(299) At what age can a child hold a pen or pencil between the thumb and index finger to write on a piece of paper

(a) 18 months (b) 3 years (c) 2 years (d) 4 years.

Section SEVEN 12 ounces of beer = 5 ounces of table wine = 1.5 ounce shot of 80 proof whiskey.

1. Alcohol can pass directly into the bloodstream from the mouth or stomach.

True

2. Alcohol is a colorless, odorless, volatile pungent drug that acts as a_____:

depressant 3. The frontal lobe of the brain is _____ for driving a motor vehicle.

essential 4. Narcotics act on the central nervous system by producing a _____ effect.

soothing

lulling

dulling

all of the above 5. The most dangerous aspect of synergism is the _____ effect.

additive

6. The amount of ethyl alcohol in a 12 oz. beer, a 5 oz. Glass of wine or a 1.5 oz. Shot of 80 proof whiskey are:

the same

7. Drugs of addiction act upon a portion of the brain called the limbic system.

True

8. Prescription drugs can impair your ability to driver safely.

True

9. Your judgement is not affected by your emotions.

False

150

Module 3

1. Substance use/abuse becomes a form of self-soothing when you feel stress or anxious.

True

2. Driving requires no awareness or attention.

False 3. The total cost of a DUI could be as high as _____ dollars:

$8,000 4. Alcohol has many sensations and effects like ______ .

nausea

dizziness

loss of coordination

all of the above 5. Alcohol is used as a gateway drug.

True

6. Using alcohol to manage stress is a _____ coping skill.

negative

Module 4

1. Motor vehicle crashes cost Americans _____ of dollars every year.

billions 2. Impaired drivers are a _____ to other drivers on the road.

danger

3. If you are convicted of a DUI, this event will have an impact on your family.

True

4. As you consume alcoholic beverages, your ability to function is improved.

False

Module 5

1. Alcohol and other drugs alter mood and affect the mind by interfering with the central nervous system.

151

True

2. Alcohol _____ a driver's reaction time.

increases 3. Alcohol is an example of a _____ type of drug.

depressant 4. As you increase your use of alcohol and other drugs your ability to safely

operate a motor vehicle decreases.

True

5. It is possible to have fun when you go out without the use of alcohol and other drugs.

True

Module 7

1. Graduated licensing is designed to introduce beginning drivers to driving.

True

2. Florida's zero tolerance law is implemented administratively.

True

3. Driving in Florida is a _____ :

privilege 4. Seatbelts save lives.

True

5. When you see an emergency vehicle responding to an emergency, you are required to _____.

yield the right of way

6. When you drive through a school zone, you should be looking for children in or near the road.

True

7. Defensive driving simply means being prepared for the worst and always being alert.

True

152

Section EIGHT (1) Emergency restraints or seclusion may be implemented without a physician order

under which of the following conditions? (a) Under no condition (b) A written order will be obtained from the primary physician within 8 hours (c) If a voluntary client wants to leave against medical advice (d) When a minor child is out of control (2) In a group therapy session, one member is very demanding, especially interrupting

others, and taking most of the group’s time. The nurses best response would be: (a) “Would you briefly summarize your point because others need time also?” (b) “Your behavior is obnoxious and drains the group” (c) To ignore the behavior and allow him to vent (d) “I am so frustrated with your behavior” (3) A client whose husband has left her has a recurrence of anorexia nervosa. The nurse

caring for her realizes that the exacerbation of anorexia nervosa results from the client’s effort to:

(a) To manipulate the husband (b) Gain control of one part of her life (c) Commit suicide (d) Live up to her mother’s expectation (4) A client with schizophrenia tells the nurse he hears voices of his dead parents. To

help the client control the voices, the nurse should recommend that he: (a) Sit in a quiet, dark room and concentrate on the voices (b) Listen to a personal stereo through headphones and sing along with the music (c) Call a friend and discuss the voices and his feelings about them (d) Engage in strenuous exercise. (5) The client with paranoid type Schizophrenia becomes angry and tells the nurse to

leave him alone. The nurse should: (a) Tell him she’ll leave for now but will return soon (b) Ask him if it is okay if she sit quietly with him (c) Ask him why he wants to be alone (d) Tell him that she won’t let anything happen to him (6) The nurse is caring for a client with Schizophrenia who expresses auditory

hallucinations. The client seems to be listening to someone who is not visible. He gestures, shouts angrily and stops shouting in mid-sentence. Which nursing intervention is most appropriate?

(a) Encourage the client to go to his room where he would experience fewer voices. (b) Acknowledge that the client is hearing voices but make it clear that the nurse does

not hear those voices. (c) Approach the client and touch him to get his attention

153

(d) Ask the client to describe what the voices are saying (7) The nurse notices that a client with obsessive compulsive behavior (OCD) washes his

hands for long periods of time each day. How should the nurse respond to this compulsive behavior?

(a) By designating times during which the client can focus on behavior (b) By urging client to reduce the frequency of behavior as rapidly as possible (c) By calling attention to or attempting to prevent the behavior (d) By discouraging the client from verbalizing anxieties. (8) A 24 year-old is experiencing an acute Schizophrenic episode. He has vivid

hallucinations that are making him agitated. The nurse’s best response at this time would be to:

(a) Take the client’s vital signs (b) Explore the content of the hallucinations (c) Tell him his fears are unrealistic (d) Engage the client in reality-oriented activities (9) A client with borderline personality disorder has been playing one staff member

against another. In formulating a plan of care for this client, the nursing staff should include which intervention?

(a) Assigning the same staff members who work with the client (b) Rotating staff members who work with the client (c) Avoiding setting limits (d) Avoiding interaction with the client until splitting behavior stops. (10) Before eating a meal, a client with OCD must wash his hands for 4 minutes,

combs hair 444 times, switch the bathroom lights on and off 44 times. What is the most appropriate goal of care for this client?

(a) Omit one unacceptable behavior each day (b) Increase the client’s acceptance of therapeutic drug use. (c) Allow ample time for client to complete all rituals before each meal (d) Systematically decrease the number of repetitions of rituals and the amount of time

spent performing them (11) A client on Lithium for bipolar disorder comes to the clinic for follow up. What

statement by the patient would indicate Lithium serum toxic levels (a) “I see green halo around lights” (b) “My vision has become blurred of late” (c) “I have been throwing up for the last 3 days” (d) “I am experiencing shortness of breath at night” (12) Which of these statements by a paranoid schizophrenic patient describes delusion

of grandeur? (a) “The FBI agents are following me wherever I go” (b) “I am moving to Hollywood, I am going to be the greatest movie star ever known.” (c) “People are jealous of me, they want me dead” (d) “My family hate me, but I don’t need them”

154

(13) A rape victim says to the nurse, “I feel so humiliated, I don’t want anyone to

know what happened to me.” What is the most therapeutic response? (a) “This is a normal feeling after what has happened to you” (b) “You should not be so hard on yourself, it was not your fault” (c) “Are you saying that you are fearful about what others think? Lets talk about that

feeling” (d) “Are you afraid people will blame you for what has happened?” (14) A paranoid Schizophrenia client is hospitalized for treatment. While everyone is

congregated in the dining room, the client shouts, “Quick! Get out! The electricity is going to kill us all” Which is the most appropriate response?

(a) “Please speak quietly, if you cannot, please leave the dining room” (b) “There is no electricity in here. All electricity is contained in the outlets.” (c) “I do not feel or see anything. Tell me what you are feeling” (d) “Electricity does not necessarily kill someone. Are you afraid?” (15) A client tells the nurse that people from Mars are going to invade the earth.

Which response by the nurse would be therapeutic? (a) “That must be frightening to you. Can you tell me how you feel about it?” (b) “There are no people living in Mars!” (c) “What do you mean when you say they are going to invade the earth?” (d) “I know you believe the earth is going to be invaded, but I do not believe that” (16) While looking out the window, the client with Schizophrenia remarks, “That

school across the street has creatures in it that are waiting for me”. Which of the following terms best describes what the creature represents?

(a) Anxiety attack (b) Projection (c) Hallucination (d) Delusion (17) The nurse is caring for a client diagnosed with bulimia. The most appropriate

initial goal for a client diagnosed with bulimia is to: (a) Avoid shopping for large amounts of food (b) Control eating impulses (c) Identify anxiety causing situations (d) Eat only three meals per day (18) Which client statement indicates the client is on the ‘road to recovery” from

alcoholism? (a) “As long as I stay away from my wife, I won’t have to drink.” (b) “I have an alcohol problem and I know things won’t be easy.” (c) “I’m OK now. I haven’t had anything to drink for 2 weeks” (d) “I’ll be OK if I just have one or two drinks”

155

(19) How may family therapy be beneficial for the family of a child with personality disorder?

(a) Helps the family understand the situation as the child’s problem—not the family (b) Creates a family therapy group to facilitate growth of all members (c) Helps the parents see that if they would work out their marital problems, the child

would be OK—he really isn’t sick. (d) Assist the family adjusts to the child’s illness as she probably won’t get better. (20) The nurse learns that an obsessive-compulsive client brushes his tongue several

times a day, and has developed several ulcerations on it. His nursing care plan should have at its highest initial priority

(a) The client would eliminate his brushing and mouth care rituals (b) The client would verbalize the underlying cause of his behavior (c) The client will seek out the nurse when he is feeling anxious (d) The client will re-establish healthy tissue in his mouth and tongue (21) In caring for abused children, the nurse understands that sexual abuse of children (a) Is significantly less common than physical abuse and neglect (b) Is usually perpetrated by strangers (c) Is often repeated from generation to generation (d) Occurs primarily in poor and disenfranchised segments of society. (22) A client in a psychiatric unit is very suspicious of the nursing staff and of the

other clients. Her nurse would like to establish a therapeutic relationship with her. Which nursing action would promote this?

(a) Avoid pressuring the client by waiting for her to initiate interactions with the nurse. (b) Approach the client frequently during the day for brief interaction (c) Set aside a specific time each day to spend with the client (d) Approach the client in a friendly manner offering to disclose some personal

information so she will feel she knows the nurse. (23) A patient has been in the hospital for 4 weeks. He is dying of terminal cancer.

During this time, he has never mentioned his condition or the fact that he is dying. The nurse knowledge of the dying process leads to the conclusion that the patient is

(a) In the grieving process and does not wish to talk (b) Still in the denial process (c) Depending on his family for support and talks to them (d) Afraid that by talking about dying, it will become a reality. (24) A 24-year-old client is admitted to the ED with a broken wrist, swollen and

bruised eye, and a fractured jaw. The client agreed to tell the nurse what actually happened if she promised not to tell anyone. The client admitted that her husband abused her in the past, approximately 3 to 4 times, but only when he was drunk. The most appropriate nursing intervention would be:

(a) Treat her physical injuries, suggest she talk to a counselor, but keep the client’s secret to maintain trust.

(b) Ask her to bring her husband so that she can get a referral for therapy (c) Report the abuse to the police

156

(d) Have her talk to another victim of domestic violence (25) A 75 year-old female client is admitted to the hospital for pneumonia. The nurse

observes that her clothes are old and dirty and have holes. She also has decubit on her sacral area. The best question to assess for the possibility of abuse would be:

(a) “How long have you lived with your son?” (b) “How much money do you have in your bank account?” (c) “Describe a typical day at home-what you have to eat, what you do during the day, and what time you bathe?” (d) “Who buys your clothes for you?” (26) A client in a psychiatric unit is very suspicious of the nursing staff and of the

other clients. Her nurse would like to establish a therapeutic relationship with her. Which nursing action would promote doing this?

(a) Avoid pressuring the client by waiting for her to initiate interaction with the nurse (b) Approach the client frequently during the day for brief interactions (c) Set aside a specific time each day to spend with the client (d) Approach the client in a friendly manner offering to disclose some personal

information so she will feel she knows the nurse better. (27) A pt looks into the mirror and cries out, “I look like a bird. My face is no longer

me.” Which would be a best response by the nurse? (a) “Which bird?” (b) “That must be a distressing experience; your face doesn’t look different to me.” (c) “Maybe it was the light at that particular time. Would you like to use another

mirror?” (d) “What makes you think that your face looks like a bird?” (28) Restraints may be used to immobilize a client’s extremity. The nurse understands

that restraints are not used (a) Prevent a client from pulling out IV or other type of therapy (b) Reduce the risk to all elderly clients from falling out of bed or off a chair (c) Prevent the removal of life support equipment (d) Prevent injury to health care personnel by combative clients (29) A delusional client refuses to eat because of a belief that the food is poisoned.

One of the most appropriate ways for the nurse to initially intervene is to (a) Taste the food in the client’s presence (b) Simply state that the food is not poisoned (c) Suggest that food is brought from home (d) Tell the client that tube feeding would started if eating does not begin (30) A client with a diagnosis of Schizophrenia refuses to eat meals. The nursing

action that would be beneficial to this client is to (a) Allow the client to eat whenever desired (b) Repeatedly allow the client to eat the food (c) Explain the importance of eating to the client (d) Sit with the client while the meals are being eaten

157

(31) A young handsome man with a diagnosis of antisocial personality disorder is

being discharged from the hospital next week. He asks the nurse for her phone number so he can call her for a date. The nurse’s best response would be

(a) “We are not permitted to date clients” (b) “No! You are a client and I am a nurse” (c) “I like you, but our relationship is professional” (d) “It is against my professional ethics to date clients” (32) The activity that would be least therapeutic for a severely depressed client would

be (a) Specific, simple instructions to be followed (b) Simple, easily completed short term project (c) Monotonous, repetitive projects and activities (d) Allowing the client to plan their own activities. (33) Which of the following independent nursing interventions would not be

appropriate to implement with clients who are actually psychotic? (a) Listening emphatically to identify precipitants of their behavior (b) Conveying calmness by providing structure within the one-to-one interaction. (c) Recognizing and dealing with their own feelings so as to not escalate the client’s

anxiety level. (d) Encouraging the client to become involved with others in a psychotherapeutic group

so as to lessen anxious feelings. (34) A nurse sees the wife of a client with end-stage AIDS crying outside the patient’s

room. Which of the following is the most appropriate action for the nurse to take? (a) Leave her alone, she needs time to grieve by herself. (b) Gently touch her arm, offer some tissues, and encourage her to verbalize her feelings. (c) Tell her it’s all right for her to cry, after all, her husband is about to die. (d) Tell her you appreciate how upset she is, but ask her to leave the hallway because her

crying is upsetting the other clients. (35) A 19 year-old pt is brought to the emergency room because she slashed her wrists.

What’s the nurse’s first concern? (a) Stabilization of physical condition. (b) Determination of antecedent, causal factors relevant to the wrist slashing. (c) Reduction of anxiety (d) Obtaining a detailed nursing history. (36) A nursing care plan for a hospitalized hyperactive client in a manic episode needs

to include: (a) Involvement in a group activity and encouragement to talk. (b) Attention to adequate food and fluid intake. (c) Protection against suicide. (d) Permissive acceptance of bizarre behavior.

158

(37) A patient refuses to eat meals in the hospital, stating that the food is poisoned. The nurse is aware that the patient is expressing an example of:

(a) Hallucination (b) Illusion (c) Delusion (d) Negativism

Section NINE (1) Which of these adaptations is considered a positive sign of pregnancy? (a) Maternal nausea and vomiting (b) A positive pregnancy test (c) Ultrasound recognition of fetus (d) Braxton-Hicks contraction (2) A 35-year-od woman, at term is admitted with a diagnosis of pregnancy- induced hypertension. The priority nursing assessment for the client should be (a) Deep tendon reflex (b) Gravida and para

(c) Vital signs (d) Protein in urine

(3) The nurse instructs the antepartal woman that preeclampsia in pregnancy is defined as the (a) occurrence of hypertension in the second half of pregnancy (b) progression of pre-existing hypertension to include maternal seizures (c) development of hypertension in pregnancy accompanied by proteinuria (d) increase in pre-existing hypertension accompanied by edema. (4) A woman in her 38th week of pregnancy, diagnosed with a complete placenta previa, asks if she can still have a vaginal delivery. The nurse responds, (a) “Yes, if the fetal head descends enough to put pressure on the placenta” (b) “Probably’ if bleeding can be kept under control” (c) “Maybe, you’ll have to discuss this with your provider” (d) “Eighty percent of women with placenta previa require operative delivery” (5) A multiparous client tells the nurse that she experienced afterbirth pains while breastfeeding her infant. The nurse should (a) instruct the client to lie prone with a small pillow under her abdomen. (b) suggest that the client offer the infant a bottle at the next feeding (c) encourage the mother to empty her bladder every 2 to 3 hours (e) administer the p.r.n. dose of simethicone (Mylicon) (6) Which equipment is found in the delivery room with a pre-eclamptic client? (a) Television with the volume turned up high (b) Bright overhead light to observe pupillary response (c) Emergency cart in case of cardiac arrest (d) Plastic airway

159

(7) What is the physiologic effect of magnesium sulfate in the pre-eclamptic client? (a) Lowers the heart rate (b) Promotes hypofibrinogenemia (c) Raises blood pressure by causing vasodilation (d) Controls convulsions by acting on the central nervous system and the myoneural

junction (8) The nurse is frequently asked about the probability that a premature baby will survive and be normal. What information is most accurate when counseling the obstetric client in preterm labor?

(a) A baby born prematurely will not have as high an intellectual capacity as if he had been born full term

(b) The earliest time at which an infant can be born with the prospect of survival is seven months gestation

(c) With each successive week after the fetus is viable, the chances of life become progressively greater.

(d) A baby, born before term is better able to perform some vital functions than those born at term.

(9) Which is considered a presumptive sign of pregnancy?

(a) Chadwick’s sign (b) Positive pregnancy test (c) Hegar’s sign (d) Urinary frequency

(10) In observing women in the latent phase of the first stage of labor, the nurse would expect to see which behavior?

(a) Tendency to hyperventilate (b) Euphoria, excitement, and talkativeness (c) Fairly quiet and introverted behavior. (d) Irritability and crying.

(11) Which physiological change occurs during the first semester?

(a) The uterus rounds out to a globular form; the walls become softer and thicker (b) Relaxation of the sacroiliac joints and the symphysis pubis creates a “wadding

gait” (c) Upward displacement of the diaphragm causing shortness of breath. (d) Milk production is complete

(12) Which physiologic changes occur during the second trimester of pregnancy?

(a) Continuation of “morning sickness” (b) Blood volume increases to peak (c) Linea nigra (d) Lightening

160

(13) The multigravida client tells the practical nurse that she has had short labor with her previous delivery. She expresses concern about getting to the hospital before the baby is born because their home is 20 miles from the hospital. What anticipatory guidance is appropriate?

(a) Appearance of show (b) Dialing 911 for assistance (c) Rupture of membranes (d) Timing of contractions

(14) The practical nurse monitors the client during an oxytocin challenge test. During her assessment the practical nurse observes that the uterine contractions are lasting 90-95 seconds and the fetal heart rate slows during the contraction and returns to normal 30 seconds later. What is the practical nurse’s first action?

(a) Notify the charge nurse (b) Check for sustained, sharp pain (c) Change the mother’s position (d) Administer oxygen to the mother

(15) What anticipatory guidance is appropriate for the obstetric client prior to an ultrasound?

(a) Even though it is very accurate test, it also has serious side effects (b) It measures the diameter of the fetal skull which facilitates estimating fetal size in

utero (c) Ultrasonography is an inaccurate index of fetal maturity (d) As an invasive procedure, ultrasonography may precipitate labor

(16) What is the nurse’s role during the administration of the oxytocin challenge test?

(a) Administer analgesics for pain (b) Assist with amniotomy (c) Interpret the findings from the test (d) Perform a biophysical profile

(17) What is the most frequently used method of observing uterine contractions during labor?

(a) Ask the mother when she has a contraction and its severity (b) Place a hand lightly on the mother’s abdomen (c) Check the external monitor printout for strength of contractions (d) Perform cervical examination every two hours

(18) In addition to careful observation and monitoring during the labor induction, what is the next most important nursing action?

(a) Open the windows to provide adequate ventilation (b) Provide reassurance and emotional support (c) Elevate foot of the bed (d) Prevent unnecessary persons from entering the room

161

(19) The obstetric client’s fundus is boggy and lochial flow heavy, which intervention does the nurse perform first?

(a) Elevate the foot of the bed (b) Manually compress the placental site (c) Massage the uterus and express clots (d) Check Victoria for signs of clot

(20) Which prenatal characteristic is associated with an increase risk for pre-eclampsia?

(a) Deficient antepartum care (b) Age 30 (c) 3 previous pregnancies (d) Inadequate dietary proteins

(21) The nurse is evaluating the prenatal patient. The patient asks when she can expect to feel the baby move. The nurse replies: (a) “You can expect to begin to notice movement during the first trimester” (b) “By the end of the sixth month you may feel some kicking” (c) “It is unusual to feel the baby move until two weeks before your due date” (d) “Most women can feel fetal movement approximately 16-20 weeks gestation” (22) Ms Sadie is seven and half months pregnant. She has a number of physical discomforts to report to the nurse. Which of these symptoms should be reported to the physician? (a) Shortness of breath when climbing the stairs (b) Heartburn (c) Constipation (d) Headaches and “spots” before her eyes (23) After the baby has been born, the nurse tells the mother that the staff will watch her carefully for the first 2-4 hours. Which of the following findings should be immediately reported to the RN in charge or the physician? (a) Increased bleeding and boggy uterus, despite massage (b) Systolic blood pressure of 100 (c) Shaking chills (d) Uterus displaced to the right (24) As the antepartum patient, Gina, was going to the bathroom, she suddenly noticed a looped cord coming out of the vagina. She quickly rang the emergency call bell, and the first nurse to arrive explained that her baby had a prolapsed cord, and quickly did the following: (a) Placed her in a trendelenburg or knee chest position to relieve pressure on the cord (b) Rupture her membrane to help speed the labor (c) Called ultrasound to get a stat biophysical profile (d) Performed a vaginal exam to determine cervical dilatation.

162

(25) The nurse is teaching the first-time mother the signs of approaching labor to report to the physician. The nurse correctly tells the patient to immediately contact the physician if she experiences. a. Vaginal bleeding b. Loss of the mucus plug c. Irregular contractions for over an hour d. Ripening o f the cervix. (26) A primipara who is due to deliver next week, calls and tells the nurse she has been having contractions every four minutes for an hour. Before asking any further questions, the nurse confirms that the client knows how to correctly time contractions which is (a) from the end of one contraction to the beginning of the next (b) from the beginning of one contraction to the end of the next (c) from the end of one contraction to the end of next. (d) From the beginning of one contraction to the beginning of the next (27) The nurse checks a patient in labor and delivery who is having mild irregular contractions and a steady painless trickle of blood from the vagina. In planning her care, the nurse is aware that it is important that this patient does not get (a) put on a fetal monitor (b) a nitrazine test

(c) put on bed rest (d) a vaginal exam

(28) A 29-year-old old mother of two, is in active with her third baby, and the nurse checks the fetal monitoring. The baby’s heart rate is going down with each contraction. It starts to get lower after of the contraction and slowly returns to baseline. Appropriate action for the nurse to take would be all of the following EXCEPT (a) positioning the patient in the lateral position (b) administering oxygen at seven to eight liters/minute (c) increasing the rate of pitocin to hasten the delivery. (d) notifying physician of fetal status while the charge nurse remains with the patient. (29) On the second day postpartum, the nurse asks the new mother to describe her vaginal bleeding. The nurse should expect her to say that it is (a) red and moderate (b) red with clots

(c) scant and brown (d) thin and white

(30) To prevent cracked nipples while breast feeding, the mother should be taught to (a) apply lanolin prior to feeding (b) nurse at least 20 minutes on each breast the first day. (c) use plastic bra liners (d) wash her nipples with water only (31) The nurse assessing a woman in labor informs you that the baby’s head is at +2. You understand this to mean: (a) The baby head has increased in size two times. (b) The baby’s head is two stations above the ischial spine. (c) The baby’s head is two stations below the ischial spine. (d) The baby’s head has dilated the cervix two times.

163

Section TEN 1. Lois is caring for Eileen, who had a median episiotomy prior to delivering a 7# 8 oz baby girl. In order to detect early signs of postpartum complications she would look for all of the following except:

a. discharge from the episiotomy b. edema of the perineum c. discomfort while in a sitting position d. separated edges of the episiotomy repair

2. Two days ago, a woman gave birth to a full-term infant. Last night, she awakened several times to urinate, and she noted that her gown and bedding were wet from profuse diaphoresis. What is one mechanism for the diaphoresis and diuresis this woman is experiencing during the early postpartum period? a. Increased temperature caused by postpartum infection b. Increased basal metabolic rate after giving birth c. Loss of increased blood volume associated with pregnancy d. Increased venous pressure in the lower extremities 3. 20-7 A woman gave birth to a 7-lb 3-oz baby boy 2 hours ago. The nurse determines that the woman's bladder is distended, because her fundus is now 3 cm above the umbilicus and to the right of midline. During the immediate postpartum period, what is the most serious consequence likely to occur from bladder distention? a. A urinary tract infection b. Excessive uterine bleeding

c. A ruptured bladder d. Bladder wall atony

4. Yesterday, a woman gave birth to a full-term baby girl. Which of the following laboratory results indicates a deviation from normal at this time? a. Hematocrit = 34% b. White blood cell count = 15,000 c. Prolonged PT and PTT d. +1 proteinuria 5 Breast engorgement is caused by which of the following? a. Overproduction of colostrum b. Accumulation of milk in the lactiferous ducts c. Hyperplasia of mammary tissue d. Congestion of veins and lymphatics 6. 20-1 A woman gave birth to a baby boy 12 hours ago. Where would the nurse expect to locate the fundus of this woman's uterus? a. One centimeter above the umbilicus b. Two centimeters below the umbilicus c. Midway between the umbilicus and the symphysis pubis d. Nonpalpable abdominally

164

7 What are the most common causes of subinvolution? a. Postpartum hemorrhage and macrosomia b. Multiple gestation and postpartum hemorrhage c. Uterine tetany and overproduction of oxytocin d. Retained placental fragments and infection 8. Which of the following patients is most likely to experience strong afterpains? a. A woman who exhibited oligohydramnios b. A woman who is a gravida 4, para 4 0 0 4 c. A woman who is bottle-feeding her baby d. A woman whose baby weighed 5 lb 3 oz 9. A steady flow of bright red blood from the vagina in the presence of a firm fundus suggests

a. uterine atony b. lacerations of the vagina or cervix c. perineal hematoma d. infection of the uterus

10. Which finding 12 hours following birth would require further assessment?

Lochia rubra, moderate Fundus at the level of the umbilicus Three voidings, total 150 ml T 98.9, pulse 72, respiratory rate. 18, BP 122/68

11. Which of the following treatments should be omitted from the client’s plan of care if she has a fourth-degree perineal laceration?

rectal suppositories sitz baths stool softeners anesthetic sprays

12. Judy has just received general anesthesia . the most important nursing action at this time is to

add pitocin to the intravenous solution administer a gastric acidifying agent place a wedge under the right hip to displace the uterus call the doctor to begin the operation

13. Which of the following techniques would the nurse use to determine the presentation of the fetus?

Naegle’s rule Ortolani’s maneuver Leopold’s maneuver MacDonald’s procedure

165

14. Marsha comes into the hospital in labor. She did not attend childbirth classes. She is very anxious and her husband is at home. The best nursing action to help with labor support at this time would be to:

teach self-hypnosis teach touch massage and breathing exercises administer analgesics call her husband to have him come in

15. Which of the following may happen if the uterus becomes overstimulated by oxytocin during the induction of labor?

weak contractions prolonged to more than 70 seconds tetanic contractions prolonged to more than 90 seconds increased pain with bright red vaginal bleeding increased restlessness and anxiety

16. Which of the following is the nurse’s initial action when umbilical cord prolapse occurs?

Begin monitoring maternal vital signs and FHR Place the client in a knee-chest position Notify the physician and prepare the patient for delivery Apply a sterile warm saline dressing to the exposed cord

17. When rupture of membranes occurs, which of the following provides evidence of the nurse understands the clients immediate needs?

a. the chorion and amnion rupture 4 hours before the onset of labor. b. Rupture of membranes removes the fetus’ most effective defense against infection c. Nursing care is based on fetal viability and gestational age d. Rupture of membranes is associated with malpresentation and possibly

incompetent cervix 18. During which of the following stages of labor would the nurse assess “crowning”?

a. First stage b. Second stage

c. Third stage d. Fourth stage

19. A normal fetal heart rate is:

80-120 110-160

120-220 120-190

20. The labor of a pregnant woman with preeclampsia is going to be induced. Before initiating the Pitocin infusion, the nurse reviews the woman's latest laboratory test findings, which reveal a platelet count of 90,000, an elevated AST level, and a falling hematocrit. The nurse notifies the physician, because these lab results are indicative of which of the following?

a. Eclampsia b. Disseminated intravascular coagulation c. HELLP syndrome d. Idiopathic thrombocytopenia

166

21. Mary Jones has been taking oral contraceptives for the past 5 years. She now desires pregnancy. Your advice to her is to:

Stop taking oral contraceptives with this period and attempt conception immediately

Stop taking oral contraceptives an use a back up method for 1 month before trying to conceive

Stop taking oral contraceptives and use a back up method for 3 months before trying to conceive

Stop taking oral contraceptives now and try to conceive 22. Mary wants to use natural family planning. Knowing that it is based on knowing when your fertile period is, you woul know that Mary understands your teaching when she states that her fertile period is when:

a. she experiences breast tenderness b. she has a clear thin elastic mucous c. she has decreased libido d. she has a weight gain of 2-3 pounds

23. Mary and John are experiencing infertility. Mary comes to you concerned that one problem may be that John smokes a pack of cigarettes everyday and asks you for information. What is the nurse's most appropriate response? a. "Your sperm count seems to be okay in the first semen analysis." b. "Only marijuana cigarettes affect sperm count." c. "Smoking can give you lung cancer, even though it has no effect on sperm." d. "Smoking can change the quality of your sperm." 24. Susan has just delivered a 7 pound 4 ounce baby girl. She is breastfeeding. She is interested in receiving Depo-Provera as a form of birth control. Then nurse responds appropriately when she tells her:

“You should have your shot before you leave the hospital.” “You can’t take Depo-Provera while breastfeeding.” “You must wait 6 weeks before beginning any birth control” “Breastfeeding will provide enough anovulatory support”

25. Susan is interested in tubal ligation and asks for more information. The nurse is correct when she tells the patient:

a. It causes a chemical reaction creating hostile mucous b. It blocks ova from entering the fallopian tube c. It creates a block making it impossible for sperm to read the uterus d. It causes ovulation to cease

26. Susan has just delivered a baby yesterday and is deciding what type of birth control to practice. She is breastfeeding and inquiring about depo-provera.

You don’t ovulate when you are breastfeeding Depo-provera is contraindicated in breastfeeding

You need to wait 6 weeks before getting the shot You can have the shot before you leave the hospital

167

Section ELEVEN 1. The nurse is assessing a 24 hour old newborn and notices a yellowish discharge from both eyes. The most likely cause of this would be: a. milia b. obstructed lacrimal duct

c. chemical conjunctivitis d. ophthalmia neonatorum

2. Due to his respiratory distress, 31-week premature newborn, Steven, is placed on ventilatory support. It is necessary to implement measures to avoid: a. RLF b. TTN

c. BPD d. NEC

3. The nurse notes that baby Jason’s breast tissues consist of a flat areola with no bud. This finding is consistent with: a. AGA b. post-term gestational age

c. decreased maternal hormone d. preterm gestational age

4. Baby Taylor gagged on mucus and became cyanotic. What should be the first action of the nurse? a. Call the health care provider. b. Aspirate the mouth with a bulb syringe. c. Lower the baby’s head and slap him on the back d. Give oxygen by positive pressure. 5. While caring for the post-term infant, the nurse recognizes that the elevated hematocrit level most likely results from a. hypoxia in utero b. underproduction of red blood cells c. increased breakdown of red blood cells d. the normal expected shift from fetal hemoglobin to normal hemoglobin 6. Thermoregulation can be a concern especially in the post-term infant who also has a a. bilirubin level of 8 b. WBC count of 10,000 cells/mm3

c. RBC count of 5 million d. blood glucose of 25 mg/dl

1. A primiparous woman is watching her newborn sleep. She wants him to wake up and respond to her. The mother asks the nurse how much he will sleep every day. The nurse's response to her is: A. "He will only wake up to be fed, and you should not bother him between feedings." B. "The newborn sleeps about 17 hours a day with periods of wakefulness gradually increasing." C. "He will probably follow your same sleep and wake patterns, and you can expect him to be awake soon." D. "He is being stubborn by not waking up when you want him to. You should try to keep him awake during the daytime so he will sleep through the night."

168

2. Parents of a newborn ask the nurse how much the baby can see. The parents specifically want to know what type of visual stimuli they should provide for their newborn. The nurse responds to the parents by telling them: A. "Babies can see very little until about 3 months of age." B. "Infants at 2 weeks of age can distinguish patterns and seem to like medium-colored or black and white geometric shapes and patterns." C. "The infant's eyes must be protected, and infants enjoy looking at brightly colored stripes." D. "It is important to shield the newborn's eyes. You should ask your physician what the infant should look at." 3. When evaluating the reflexes of a newborn, the nurse notes that upon hearing a loud noise the newborn symmetrically abducts and extends his arms, fans his fingers out and forms a C with thumb and forefinger, and has a slight tremor. The nurse would document this finding as a positive: A. tonic neck reflex B. glabellar (Myerson's) reflex

C. Babinski's reflex D. Moro's reflex

4. The nurse is using the new Ballard scale to determine the gestational age of a newborn. Which assessment finding is consistent with a gestational age of 40 weeks? A. flexed posture B. abundant lanugo C. smooth pink skin with visible veins D. faint red marks on the soles of the feet 5. Before a newborn is discharged, the nurse places a warm diaper on the newborn's foot so that a heel stick can be performed to obtain blood for the PKU test. The newborn's mother wants to know why this test needs to be performed. The nurse tells the mother that the test will determine if her newborn will need: A. lifelong dietary management B. blood transfusions C. medications to prevent infection D. iron-enriched formula 6. A newborn is jaundiced and is receiving phototherapy. An appropriate nursing intervention when caring for an infant with hyperbilirubinemia and receiving phototherapy would be to: A. apply an oil-based lotion to the newborn's skin to prevent dying and cracking B. limit the newborn's intake of milk to prevent nausea, vomiting, and diarrhea C. place eye shields over the newborn's open eyes D. change the newborn's position every 2 hours 7. Early this morning, a baby boy was circumcised using the plastibell method. The nurse tells the mother that she and the baby can be discharged after: A. bleeding stops completely B. yellow exudate forms over the glans

C. the plastibell rim falls off D. the infant voids

169

8. A mother expresses fear about changing her infant's diaper after he is circumcised. What does the woman need to be taught to take care of the baby when she gets home? A. cleanse the penis with prepackaged diaper wipes every 3 to 4 hours B. put constant pressure on the penis, if bleeding is noted, and check the site again in 2 hours C. cleanse the penis gently with water and put Vaseline (petroleum) around the glans after each diaper change D. wash off the yellow exudate that forms on the glans at least once every day to prevent infection 9. A nurse is responsible for teaching new parents about the hygienic care of their newborn. The nurse should tell the parents to: A. avoid washing the head for at least 1 week to prevent heat loss B. begin tub baths when the cord is dried and clamp is removed C. cleanse the ears and nose with cotton-tipped swabs, such as Q-tips D. create a draft-free environment of at least 75� F when bathing their baby 10. When preparing to administer a Hepatitis B vaccine to a newborn, the nurse should: A. obtain a syringe with a 25-gauge, 5/8-inch needle B. confirm that the newborn's mother has been infected with the hepatitis B virus C. assess the dorsogluteal muscle as the preferred site for injection D. confirm that the newborn is at least 24 hours old 11. When assessing the integument of a 24-hours-old newborn, the nurse notes a pink papular rash with vesicles superimposed on the thorax, back, abdomen. The nurse should: A. notify the physician immediately B. move the newborn to an isolation nursery C. document the finding as erythema toxicum D. take the newborn's temperature and obtain a culture of one of the vesicles 12. Which of the following findings on physical assessment of a neonate would indicate the need for further observation and examination? a. Epstein pearls b. Acrocyanosis of hands and feet

c. Babinski reflex d. Low set ears

13 Which of the following findings would the nurse consider normal in assessing the anterior fontanel of a neonate? a. Closed b. Sunken

c. Bulges when infant is asleep d. Pulsates when infant is asleep

14. Using the Ballard scale to estimate the gestational age of the newborn, the nurse finds thick and leathery skin with deep cracking and significant peeling. This is considered a sign of: a. prematurity b. meconium aspiration syndrome

c. postmaturity d. LGA

170

15. A post mature infant will have which of the following characteristics? a. large deposits of subcutaneous fat b. nails extending over fingertips c. pale skin d. fused eyelids 1. When caring for a client with ascites the nurse should understand that the portal

vein: a. Brings blood away from the liver b. Enters the superior vena cava from the cranium c. Brings venous flood from the intestinal wall to the liver d. Is located superficially on the anteromedial surface of the thigh

2. When assessing a client with portal hypertension, the nurse should be alert for

indications of: a. Liver abscess b. Intestinal obstruction c. Perforation of the duodenum d. Hemorrhage from esophageal varices

3. The nurse is aware that the symptoms of portal hypertension in clients with

cirrhosis are chiefly the result of: a. Infection of the liver parenchyma b. Fatty degeneration of Kupffer cells c. Obstruction of the portal circulation d. Obstruction of the cystic and hepatic ducts

4. The nurse understand that the ascites seen in cirrhosis results in part from:

a. The escape of lymph into the abdominal cavity directly from the inflamed liver sinusoid

b. Increase plasma colloid osmotic pressure due to excessive liver growth and metabolism

c. The decreased levels of ADH and aldosterone due to increasing metabolic activity in the liver

d. Compression of the portal veins, with resultant increased back pressure in the portal venous system

5. The physician orders a paracentesis for a client with ascites. Before the procedure,

the nurse should instruct the client to: a. Empty the bladder b. Eat foods low in fat c. Remain NPO for 24 hours d. Assume the supine position

6. The nurse administers Neomycin to a client with hepatic cirrhosis to prevent the

formation of: a. Bile b. Urea

171

c. Ammonia d. Hemoglobin

7. The basic pathophysiologic problem in cirrhosis of the liver causing esophageal varices is: a. Ascites and edema b. Portal hypertension c. Loss of regeneration d. Dilated veins and varicosities

8. When assessing a client with liver insufficiency the nurse will expect:

a. Anuria b. Fetor hepaticus c. Blepharospasm d. Globus hystericus

9. A client is diagnosed as having malabsorption syndrome. Striking clinical

improvement should be noted after administration of: a. Folic acid b. Vitamin B12 c. Corticotrophin d. A gluten-free diet

10. When planning dietary teaching for a client with malabsorption syndrome the

nurse should include the need to avoid: a. Rice or corn b. Milk or cheese c. Fruit or fruit juices d. Wheat, rye, or oats

11. A client is diagnosed as having acute appendicitis. This condition is associated

with: a. Poor diet habits b. Infection of the bowel c. Hypertension and resultant edema d. Compromised circulation to the appendix

12. An 18-year-old is admitted with an acute onset of right lower quadrant pain.

Appendicitis is suspected. To determine the etiology of the pain, the client should be assessed for: a. Urinary retention b. Gastric hyperacidity c. Rebound tenderness d. Increased lower bowel motility

13. A client has an appendectomy and develops peritonitis. The nurse should assess

the client for an elevated temperature and: a. Hyperacidity b. Extreme hunger

172

c. Urinary retention d. Local muscular rigidity

14. The position that is indicated for a client after surgery for a perforated appendix

with localized peritonitis is the: a. Sims’ position b. Semi-Fowler’s position c. Trendelenburg position d. Dorsal recumbent position

15. Four days after abdominal surgery a client has not passed any flatus and there are

no bowel sounds. Paralytic ileus is suspected. In this condition there is an interference caused by: a. Decreased blood supply b. Impaired neural functioning c. Perforation of the bowel wall d. Obstruction of the bowel lumen

16. A client is to have an enema to reduce flatus. The rectal catheter should be

inserted: a. 2 inches b. 4 inches c. 6 inches d. 8 inches

17. A 93-year-old client with a history of diverticulitis is admitted with severe

abdominal pain, anorexia, nausea, vomiting for 24 hours, a markedly elevated temperature, and increased WBC’s. The primary reason for performing surgery is most likely that: a. Surgery is usually indicated for clients with a diagnosis of diverticulitis b. The symptoms exhibited by the client on admission were life threatening c. In some instances diverticulitis is difficult to differentiate from carcinoma

except surgically d. The client’s age indicated that immediate correction of the potentially fatal

condition was needed 18. Vitamins are administered parenterally for clients with an inflamed intestine

because: a. More rapid action results b. They are ineffective orally c. They decrease colon irritability d. Intestinal absorption may be inadequate

19. After many years of coping with colitis, a client makes the decision to have a

colectomy as adviced by the physician. A significant factor in this decision may have been the knowledge that: a. Surgical treatment cures ulcerative colitis b. It would be temporary until the colon heals

173

c. Ulcerative colitis can progress to Chron’s disease d. Without surgery the client would be unable to eat table foods

20. The symptoms that the nurse should expect when assessing a client with colitis

are: a. Leucocytosis, anorexia, weight loss b. Anemia, hemoptysis, weight loss, abdominal cramps c. Fever, anemia, nausea and vomiting, leucopenia, diarrhea d. Diarrhea, anorexia, weight loss, abdominal cramps, anemia

21. The most serious complication associated with chronic inflammation of the bowel

is: a. Ileus b. Bleeding c. Perforation d. Obstruction

22. When caring for the client with an ileostomy the nurse would:

a. Encourage the client to eat foods high in residue b. Expect the stoma to start draining on the third postoperative day c. Anticipate that emotional stress can increase intestinal peristalsis d. Explain that the drainage can be controlled with daily irrigation

23. When receiving an enema, the client should be placed in the:

a. Sims’ position b. Back-lying position c. Knee-chest position d. Mid-fowler’s position

24. The nurse administer neomycin sulfate to a client before colon surgery to:

a. Destroy intestinal bacteria b. Increase the production of vitamin K c. Decreased the incidence of any secondary infection d. Decreased the possibility of postoperative urinary tract infection

25. The nurse should protect the client’s skin surrounding a colostomy opening by

using: a. Alcohol b. Mineral oil c. Skin barriers d. Tincture of benzoin

26. Postoperatively, if a client’s colostomy stoma is viable the nurse should expect

the color to be: a. Gray b. Brick red c. Pale pink d. Dark purple

174

27. The nurse is aware that the manifestation that is found more in ulcerative colitis

than in Chron’s disease is: a. Inclusion of transmural involvement of the small bowel wall b. Correlation with increased malignancy because of malabsorption syndrome c. Involvement beginning proximally with intermittent plaques found along the

colon d. Involvement starting distally with rectal bleeding and spreading continuously

up the colon 28. A client with ulcerative colitis has had frequent severe exacerbations over the last

several years. The client has had intense pain and severe diarrhea and has recently become cachectic. The therapeutic course that the nurse should expect the physician to explore with his client is: a. Intensive psychotherapy b. Continued medical therapy c. Surgical therapy (colectomy) d. Diet therapy (low residue, high protein)

29. A client who has had a colostomy should follow a diet that is:

a. Rich in protein b. Low in fiber content c. High in carbohydrate d. As close to normal as possible

30. The nurse suspects an elderly client has become impacted when the client states:

a. “I have a lot of gas pain” b. “I don’t have much of an appetite” c. “I feel like I have to go and just can’t” d. “I haven’t had a bowel movement for 2 days”

Section TWELVE

1. The nurse is caring for a 57 year old client with non-insulin dependent diabetes mellitus. The client is currently on orinase. The nurse knows that orinase is believed to lower the client’s blood glucose level by:

A. potentiating B. lowering the renal threshold of glucose C. stimulating pancreatic cells to release insulin D. combining with glucose to render it inert Rationale: oral hypoglycemics such as orinase, lower the blood glucose by stimulating pancreatic cells to release insulin. (C.)

175

2. Mrs. Anderson is a 55 year old client with adult onset diabetes mellitus (DM type II). Mrs. Anderson wants to know the best method to remove corns from her toes. The nurse should advise her to:

A. apply a high-quality corn plaster B. consult her physician about removing the corns C. apply iodine to the corns before peeling them off D. soak her feet in borax solution to peel off the corns Rationale: the diabetic client should consult the physician when she has a corn of her foot. Removing a corn is serious for a client who is diabetic due to the possibility of trauma related to poor circulation. (B.)

3. The nurse is caring for a client who has a tentative diagnosis of renal calculi. Which

one of the following associated signs and symptoms is inconsistent with a diagnosis of renal calculi.

A. pain radiating to the external genitalia B. uncontrolled diarrhea C. microhematuria D. nausea & vomiting Rationale: Uncontrolled diarrhea is inconsistent with renal calculi. All other choices are manifestation of renal calculi. (B.)

4. Mr. S is a client who has been found to have renal calculi. He is scheduled for an

intravenous pyelogram (IVP). The nurse teaches him what to expect when the dye is injected. The nurse knows he understands if he says the when the dye is injected he may experience:

A. a metallic taste B. flushing of the face

C. cold chills D. chest pains

Rationale: When the dye is injected the client may experience a flushing of the face, a salty taste in there mouth, and a feeling of warmth. (B.)

5. The nurse is caring for Mr. K. He is a 67 year old who has congestive heart failure.

He has edema and is lethargic. When caring for the client, the nurse should know that the primary pathophysiological reason for this condition to exist is inability of the heart muscles to:

A. contract effectively B. maintain a regular rhythm C. experience full refractory periods D. initiate electrical currents properly Rationale: The primary pathophysiological reason for congestive heart failure is the inability of the heart muscles to contract properly. (A.)

176

6. While caring for a client with congestive heart failure (CHF), the nurse knows that

the furosemide (Lasix) is working effectively based on the fact that the client’s data 3 days later reveals:

A. he eats better B. he weighs less C. he is less thirsty D. he has clearer urine Rationale: After three days of being on furosemide the client should show an improvement with his congestive heart failure. The primary reason for the diuretic is to excrete the sodium and water and therefore lessen the workload of the heart. (B.)

7. An 82 year old woman is admitted to the hospital with a diagnosis of pneumonia. She

has a temperature of 102 F and a productive cough and shortness of breath. Considering the client's clinical symptoms, what site should the nurse use to obtain her temperature.

A. oral B. groin fold C. rectal D. axillary

Rationale: The best way to take a temperature of a client who has a cough and shortness of breath would be rectal. (C.)

8. A client with pneumonia is coughing up purulent sputum. Which one of the following

nursing measures is most likely to help liquefy these viscous secretions? A. postural drainage B. breathing humidified air C. clapping and percussion over the affected lung D. coughing and deep breathing exercises. Rationale: Humidified air helps to liquefy the secretions, and therefore, it will help to make the drainage easier to expectorate. (B.)

9. A 78 year old client with pneumonia is extremely diaphoretic and has a fever.

Considering the client's advanced age, she is at particular risk for developing: A. hyponatremia B. hypokalemia C. hypercalcemia D. hyperphosphatemia

Rationale: The diaphoretic state that the client is in will predispose her to hyponatremia. (A.)

177

10. T is a 2 year old who is admitted to the hospital with acute asthma. A blood sample is obtained to measure T's arterial blood gases. The nurse should expect:

A. an elevated pH B. a raised oxygen level C. a decrease bicarbonate level D. a increase carbon dioxide level

Rationale: Gas exchange is limited because of narrowing and swelling of the bronchi. As a result, the Pco2 will increase. (D.)

11. The nurse is in the room of the monitored client who goes into ventricular fibrillation

(VF). The nurse calls for help, knowing that which of the following items will be needed immediately?

A. pacemaker insertion tray B. ventilator C. defibrillator D. lidocaine (xylocaine) Rationale: A physician or a nurse certified in advanced cardiac life support (ACLS) must immediately defibrillate the client to convert the heart into normal rhythm. (C.)

12. The client with heart failure has a magnesium level of 1.5 mg/dL. The nurse should:

A. encourage increased intake of phosphate antacids B. monitor the client for dysrhythmias C. administer ordered magnesium in normal saline D. encourage the client to eat foods such as ground beef, eggs, and chicken

breasts

Rationale: The client should be monitored for dysrhythmias because the client is predisposed to ventricular dysrhythmias. (B.)

13. A client has had a sputum specimen sent to the laboratory for a Gram's stain and for

culture and sensitivity testing. The laboratory calls back the initial results within 1 hour. The nurse evaluates the laboratory findings. The nurse would prepare to telephone the physician about which of the following reports?

A. gram’s stain B. culture C. sensitivity D. culture & sensitivity Rationale: The Gram stain gives information about the organism: whether it is gram negative or gram positive. Then, the physician can determine which type of antibiotic to use to treat the condition. (A.)

178

14. The nurse is caring for a child with leukemia. The nurse notes that the platelet count

is 20,000/mm3. Which of the following would not be a component of the plan of care?

A. assess stools for blood B. clean oral cavity with a toothette C. administer acetaminophen (Tylenol) suppositories for fever D. provide quite play activities

Rationale: platelet count of 20,000/mm3 places the child at risk for bleeding; the use of suppositories may risk rectal bleeding. (C.)

15. The nurse has conducted client teaching to prevent return of calcium oxalate stones.

The nurse evaluates that the client has understood the instructions if the client selects which of the following vegetables on the dietary menu?

A. beets B. green beans

C. peas D. spinach

Rationale: Vegetables that are high in oxalic acid include beets, carrots, green beans, and spinach. (C.)

16. A nursing priority with a client with a diagnosis of aruptio placenta is to minimize

alterations in fetal tissue perfusion. The nurse recognizes the goal has been met when which of the following is assessed?

A. decreased fetal heart rate variability B. presence of late decelerations C. presence of accelerations D. evidence of fetal bradycardia

Rationale: Accelerations are an indication of fetal well-being and oxygenated fetus. (C.)

17. The client reports to the clinic for follow-up after a 1 month treatment with acebutolol

(Sectral). The clinic nurse evaluate that a therapeutic effect of the medication has occurred when the nurse notes:

A. a blood pressure of 130/84 B. an apical pulse of 88 C. palpable peripheral pulses D. maintenance of desired weight Rationale: Acebutolol (Sectral) is a beta blocker used to manage mild to moderate cardiac dysrhythmias. The expected therapeutic response is controlled blood pressure. (A.)

179

18. R is a three year old who is admitted to the pediatric unit with a diagnosis of nephrosis. The most important nursing intervention for R is:

A. encourage fluids B. regulating his diet C. maintaining bed rest D. preventing infection

Rationale: Infection is a constant threat because of poor nutrition. (D.)

19. Lisa is hospitalized with a severe asthma attack. The acid-base imbalance

complicating this condition is:

A. respiratory alkalosis B. respiratory acidosis C. metabolic alkalosis D. metabolic acidosis

Rationale: Respiratory acidosis is caused by the impaired respirations and increased formation of carbonic acid. (B.)

20. Clients who have been diagnosed with tuberculosis will need treatment with which of

the following: A. old tuberculin B. BCG vaccine C. INH & Rifampin D. purified protein derivative of tuberculin

Rationale: INH and Rifampin are the most effective drugs used to treat tuberculosis. (C.)

21. A client presents to the Emergency Department with an open compound fracture and multiple lacerations. A high level priority nursing diagnosis is:

A. altered fluid volume B. impaired physical mobility C. knowledge deficit D. risk for infection

Rationale: Although options A & B may be correct, the risk for infection is extremely high because of the open fracture and lacerations. Trying to keep the client from developing an infection should begin on admit. (D.)

180

22. Mrs. A. is a 61 year old female diagnosed with chronic glaucoma. She is to be treated

with pilocarpine (Isopto Carpine). Which of the following is essential for the nurse to include when teaching Mrs. A. about pilocarpine (Isopto Carpine)?

A. take the medication whenever symptoms occur B. visual changes such as blurring and myopia are normal C. increase fluids to 3000 cc/day D. immediately report any tachycardia or difficulty breathing

Rationale: Essential teaching includes explaining that pilocarpine (Isopto Carpine) is a cholinergic agent, symptoms such as tachycardia and difficulty breathing may indicate hypersensitivity or the need for dose adjustment. (D.)

23. R.C. is experiencing lower leg cramps while in the hospital for hypertension. R.C.

informs the nurse of the symptoms. The nurse notices that the patient’s serum potassium level is 2.9 mEq/dL. The nurse receives an order to administer potassium supplements. The nurse plans to perform which of the following non-pharmacologic intervention to assist in maintaining a normal serum potassium level of 3.5 – 5.3 mEq/dL?

A. encourage the patient to lose weight at a rate of one to two lb. per week B. allow the patient to verbalize concerns about the diagnosis of hypertension C. teach the patient about the importance of eating bananas and drinking orange

juice. D. Encourage the patient to engage in regular exercise.

Rationale: Foods such as orange juice and bananas are high in potassium and will assist in maintaining a normal potassium level. (C.)

24. The nurse has just performed a neurological assessment on a client. The Glasgow

Coma Score was three. The nurse analyzes the assessment data and formulates a care plan. Which one of the nursing diagnoses indicates and understanding of the assessment findings?

A. risk for infection B. risk for injury C. self care deficit D. disturbed thought process

Rationale: The Glasgow coma scale is a reliable predictor of coma outcome but it is not reliable for predicting outcome for patients with middle range scores. Scores range from three to 15, with scores of eight or higher having a good prognosis for recovery. Scores lower than four are associated with high mortality and poor prognosis for cognitive recovery. (C.)

181

25. A male client comes to the medical nursing unit after several hours in the emergency

department. He was treated for chemical burns to the hands and face. During the initial assessment, he asks the nurse to be honest with him and tell him how bad his face looks. Which nursing diagnosis is most appropriate based on this data?

A. Altered tissue perfusion related to traumatic event B. Fluid volume deficit related to traumatic event C. Pain related to traumatic event D. Body image disturbance related to a traumatic event

Rationale: Disfigurement can result in a body image disturbance. The data in this stem supports the diagnosis. The other diagnoses may also apply, but they are based on physiological data. (D.)

26. An ambiguous order was written by a physician. The nurse believed the order was

incorrect. What should the nurse do? (D.) A. clarify the order with the physician caring for another patient on the unit. B. Ask the charge nurse if the order is correct. C. Contact the nursing supervisor and advise her of the concern. D. Call the physician who prescribed the medication.

Rationale:

27. The director of nursing (DON) has provided education for shift charge nurses to

become more democratic in their leadership style. In assessing individual charge nurses for subsequent behavior change, the DON would be looking for which of the following behaviors?

A. Cooperation among staff members with the charge nurse offering suggestions and providing constructive criticism.

B. Group members acting independently without input from the charge nurse. C. The charge nurse giving orders and providing direction to the group. When

new situations arise, the staff nurses obtain input from the charge nurse before intervening.

D. The charge nurse provides support to the staff nurse, while the staff nurse gives the UAP direction on how to change an occupied bed.

Rationale: In democratic leadership, leaders encourage group discussions and decision- making. The democratic leader acts as a catalyst or facilitator in guiding the group towards meeting its goals. Option 2 is an example of laissez-faire leadership. Option 3 demonstrates autocratic leadership styles, while option 4 reflects situational leadership. (A.)

182

28. Many patients with hypoparathyroidism are at high risk for injury. The nurse should

take which one of the following nursing actions to insure the safety of the patient? A. Encouraging exercise to strengthen muscles. B. Maintaining a stimulating environment. C. Checking on the client every two hours. D. Instituting seizure precautions. Rationale: Patients experiencing hypoparathyroidism are at risk for seizures, respiratory distress and tetany as a result of hypocalcemia. While frequent checking on the patient is useful, it is not the best response. (D.)

29. Mrs. G., 35 years old, is admitted to the hospital for complaints of nausea, vomiting

and abdominal pain which becomes more intense approximately 1 to 2 hours after eating. To correctly assess for the presence and character of bowel sounds, the nurse should:

A. palpate the abdomen for tenderness prior to auscultating for bowel sounds. B. use the bell of the stethoscope to auscultate for presence of bowel sounds. C. listen for bowel sounds for 3 to 5 minutes before charting “absent bowel

sounds.” D. assess for a positive fluid wave before listening for bowel sounds. Rationale: Normal bowel sounds are 5 to 35 clicks or gurgles every minute. Generally, sounds are about the same throughout the abdomen. (C.)

30. When assisting a client in bladder retraining following prostate surgery, which

independent nursing measure would be beneficial? A. Administer additional dose of antispasmodic. B. Insert a straight catheter after two hours if unable to void. C. Encourage fluids of 1 – 1.5 liters per day after the catheter is removed. D. Irrigate the bladder with normal saline periodically.

Rationale: To facilitate voiding and filter the urinary system, the client should be encouraged to drink at least 1-1.5 liter per day. (C.)

183

31. Mrs. Y is to be discharged after treatment for contact dermatitis which resulted in

ulceration of both hands from dishwashing. What precautions should the nurse include in discharge instructions for Mrs. Y.?

A. “You should try numerous detergents to determine which ones that you are sensitive to.”

B. “Since contact dermatitis results from an internal reaction to stimuli, there is no need for you to take any precautions.”

C. “You should wear rubber gloves while washing dishes, until a determination can be made of the causative factor after patch testing is done.”

D. “Continued exposure to these substances will not create any problems providing that you take antihistamines and steroids to counteract any reactions.”

Rational: Contact dermatitis is an inflammatory response of the skin to chemical or physical allergens. Use of a barrier to avoid contact is suggested until a series of patch test are done which would help to identify the causative agent. (C.)

32. Mrs. F. is a 69 year old postoperative client who has a tracheostomy and who was

placed on a ventilator support system. The nurse is assessing this client and establishes a nursing diagnosis to be Ineffective Airway Clearence. Which nursing goal would be MOST appropriate for this client? The client will:

A. identify resources to assist in promoting a safe environment. B. demonstrate problem-solving skills. C. demonstrate absence/reduction of congestion with breath sounds. D. recognize the need for/seek assistance to prevent accidents/injuries.

Rationale: Nursing goals or desired outcomes for a client with a nursing diagnosis of Ineffective Airway Clearance would include: maintaining airway patency; being able to readily expectorate/clear secretions; demonstrating absence/reduction of congestion with breath sounds clear, respirations noiseless; improved oxygenation exchange; verbalize understanding of cause(s) and therapeutic regimen, demonstrating behaviors to improve or maintain airway clearance and identifying potential complications and initiating appropriate actions. (C.)

184

33. P.S., at age 32, is diagnosed with endometriosis and given a prescription for a

gonadotropin-releasing hormone agonist. Noticing her puzzled expression, the nurse asks her what she is concerned about. P.S. says she needed “something for pain and the doctor gave this hormone prescription”. Considering a plan of care, what intervention should the nurse take first?

A. Have the M.D. write a prescription for pain medication. B. Explain the action of the prescribed drug. C. Evaluate P.S.’ knowledge about treatment for her condition. D. Refer her to a support group for women with endometriosis.

Rationale: While all of the intervention could be appropriate, P.S. is giving cues that she does not understand the relationship between the prescription and her pain. The best plan is to make a tentative nursing diagnosis of “knowledge deficit related to lack of information about treatment of endometriosis,” and then act to resolve the problem. (C.)

34. The nurse strokes the sole of the infant’s foot and observes for dorsiflexion of the

great toe. The nurse than holds the infant upright and allows the bottom of the infant’s foot to touch the examining surface and observes for knee and hip flexion. What two reflexes has the nurse assessed?

A. plantar reflex, stepping reflex B. trunk incurvation reflex, babinski reflex C. babinski reflex, stepping reflex D. palmar reflex, plantar reflex

Rationale: The Babinski reflex and the stepping reflex are described in the question in that order. Both reflexes should disappear by age three months. (C.)

35. The parents of a four month old child ask the nurse when would be the best time for them to go away for a weekend and leave the child with the grandmother. The BEST answer for the nurse to give them is:

A. “It is best to wait until the child is two years old.” B. “Now would be the best time.” C. “It’s better not to leave the child with another person until the child starts

school.” D. “You can leave the infant with his grandmother anytime.”

Rationale: Infants experience separation anxiety beginning around eight months. (B.)

185

36. Mrs. K. is brought to the hospital unresponsive due to diabetic ketoacidosis. Insulin is

ordered stat. Which type of insulin is appropriately administered in this situation? A. regular B. NPH

C. lente D. ultralente

Rationale: Regular insulin is a rapid acting insulin which begins acting within 15 minutes to one hour when administered subcutaneously. Regular insulin is a clear solution without any modifying agents and therefore is the only insulin that can be administered intravenously in an emergency situation. Intravenous administration is the route of choice when a client requires rapid reversal of hyperglycemia and ketoacidosis. (A.)

37. Which of the following adverse effects should the nurse be monitoring in the client

who is taking isocarboxaid (Marplan)?

A. respiratory depression B. hypertensive crisis

C. laryngospasm D. seizures

Rationale: Hypertensive crisis is a serious reaction that can occur when a client is taking isocarboxaid (Marplan). The symptoms of this are elevated blood pressure, severe headache, dizziness, stiff neck, and visual disturbances. (B.)

38. The nurse will know that Mr. J. has misunderstood teaching done about warfarin

sodium (Coumadin) therapy if he states: A. “I will not take aspirin while taking Coumadin.” B. “I will use mineral oil if I become constipated.” C. “I will be sure that my daily vitamin does not contain vitamin K.” D. “I will not floss my teeth while on Coumadin.”

Rationale: Hypoprothrombinemic effectof warfarin sodium (Coumadin) may be enhanced by mineral oil since it decreases vitamin K levels by decreasing absorption of vitamin K in the colon. (B.)

39. The nurse is monitoring a patient on Amphotericin B. The nurse should be alert for which of the following adverse effects?

A. chills, anorexia, angiodemia, or uticaria B. headache, diarrhea, and rash C. constipation, abdominal distention and nausea D. fatigue, insomnia and nervousness

Rationale: Amphotericin B is for use in life threatening fungal infections and is highly toxic. Adverse effects can be minimized by slow administration over two to four hours. (A.)

186

40. During the interview with the client, the nurse says, “We have talked about some of the problems of parenting and some of the difficulties you had when you were growing up. We can continue to discuss the feelings related to these areas.” The nurse is using which of the following facilitative (therapeutic) communication skills?

A. questioning B. restarting C. summarizing D. confronting

Rationale: Summarizing is the process of highlighting ideas that are expressed in an interaction. Summarizing help both the client and the nurse to focus on important points and keep the relationship goal-directed. Summarizing is also useful in synthesizing and connecting themes and patterns that arise in the relationship. (C.)

41. The most common cause of chronic renal failure is:

A. cystic kidney disease B. hypertension C. glomerulonephritis D. diabetic nephropathy

Rationale: Conditions causing chronic renal failure typically involve diffuse, bilateral disease of the kidneys leading to progressive destruction and scarring of the nephron. Diabetic nephropathy causes glomerulosclerosis and thickening of the glomerular basement membrane. (D.)

42. A client with bipolar disorder is losing weight. Which nursing intervention should

take priority in the plan of care? A. Establish an eating place that is free from stimulation. B. Determine the client’s calorie requirements. C. Create morning and evening rituals around eating. D. Obtain an order for a sedative 2 hours before meals.

Rationale: The nursing priority in this scenario is to set an appropriate goal for weight stabilization; this cannot occur until the nurse determines the client’s calorie requirements. Establishing a place to eat that is free from distraction would also benefit this client, but it does not take priority over determining calorie requirements. Creating a ritual or medicating the client before meals would not improve the client’s nutritional status or meet the goal of weight stabilization. (B.)

187

43. The nurse is evaluation the progress of an adolescent who suffered second and third-

degree burns over 30% of his body 4 days ago. Which of the following would indicate that he is improving?

A. He no longer feels pain. B. All of his blisters have opened. C. Revascularization occurs. D. Blanching of the tissue stops.

Rationale: Within in 4 days of a burn injury, blood vessels should reappear at the surface of the skin to begin to renourish the tissues. Because nerve ending are also burned away in third-degree burns, absence of pain would not indicate improvement. Keeping blisters intact would help protect against potential infection; open blisters are not a sign of improvement. Blanching of the tissues is a sign of revascularization. (C.)

44. The nurse is caring for a hospitalized client with severe pregnancy-induced

hypertension (PIH). Which of the following would be an appropriate nursing intervention for this client?

A. Encourage visitors to spend time with her. B. Keep her in the supine position. C. Keep her room darkened and limit nurse visits. D. Keep her on modified bed rest in a lateral position.

Rationale: Keeping the client in a lateral position would improve circulation to the placenta; modified bed rest is preferred because strict bed rest can lead to thrombophlebitis. A client with severe PIH should be protected from outside stimuli and too many visitors. Because the nurse must closely monitor the client, nurse visits would not be limited. (D.)

45. Which of the following IV solutions would be appropriate for the nurse to administer

to a client who has hypernatermia compounded by hypovolemia?

A. D5W B. 0.9% saline C. D5NS D. 0.45% saline

Rationale: A client who is both hypernatremic and hypovolemic needs isotonic fluids to restore circulating volume. A solution of 0.9% saline (NS) would be indicated to correct the initial volume deficit. Following normalization of circulating volume, a hypotonic fluid (0.45% NaCl or D5W) could be used to correct the hypernatremic state. Options 1 and 3 would not be appropriate at this time. Option 4 is incorrect because this is a hypertonic solution. (B.)

188

47. Which of the following diets would lead to impaired wound healing?

A. Deficient in vitamin K B. Deficient in niacin C. Deficient in vitamin C and zinc D. Deficient in iodine

Rationale: Deficiencies in vitamin C and zinc can lead to impaired wound healing since both are involved in collagen synthesis. The other options do not directly correlate with impaired wound healing from a structural standpoint. (C.)

48. J. has been on the telephone for ten minutes. He has become increasingly loud and

terminates the conversation by slamming the phone. He walks into the lounge and kicks a chair. The BEST nursing intervention for J. would be to:

A. respect J.’s personal space and offer him one-to-one in a quiet area. B. let J. know that continued agitation will result in restraints. C. remove other patients and let J. “blow off steam” in the lounge by himself. D. immediately gather several staff to physically take J. to seclusion.

Rationale: Give J. the opportunity to discuss the events leading to the acting out. Since J. is experiencing anger, respecting his personal space provides a boundary and gives him the opportunity to regain control. (A.)

49. You are working with a patient whose nursing diagnosis is ineffective individual

coping related to increased alcohol consumption secondary to stress at work. One of your expected outcomes is that the patient will:

A. decrease his alcohol use during stress. B. sleep without difficulties when under stress. C. decrease the stress in his life. D. attend AA meetings when stressed.

Rationale: An expected outcome for ineffective coping is to increase the coping skills. Attending an AA meeting rather than using alcohol is an effective coping mechanism. (D.)

50. Mr. T. has been admitted to your unit for Obsessive Compulsive Disorder. He

constantly washes his hands. In planning his nursing care which of the following is an important short term goal. The client will:

A. refrain from washing his hands. B. discuss conflicts underlying his behavior. C. be protected from skin integrity breakdown. D. verbalize understanding of his disorder.

189

Rationale: Mr. T.’s constant handwashing can cause skin breakdown and pain. An appropriate short term goal is to provide skin care in between handwashings and to contract with the patient for hand self care and reduction in the time used for the ritual. The goal is to help the client begin to gain control over the behavior. Impulsive behavior patterns are characterized by an irrational impulse to act. Mr. T.’s constant handwashing is an irrational stereotypical act aimed at reducing anxiety about his obsession with preventing self contaimination. When clients are hospitalized, their rituals often interfere with other client’s use of facilities. Hurrying or interrupting a ritual is counterproductive. They will usually stat the ritual again from the beginning. (C.)

51. The nuse is assessing a 17 year old girl, and her mother stays in the room. The BEST

way for the nurse to handle the situation is to:

A. ignore the mother’s presence. B. ask the daughter if she wants her mother to stay. C. ask the mother to leave, assuring her that you will talk to her later. D. ask the mother questions during the assessment.

Rationale: The nurse should assure the privacy of the 17 year old. The daughter may not feel comfortable talking about her physical concerns in front of her mother. (C.)

52. A client is having difficulty falling asleep and consequently has inadequate amount of

sleep. Which of the following is the most accurate assessment?

A. early insomnia B. intermittent insomnia C. somnambulism D. bruxism

Rationale: Difficulty falling asleep is early or initial insomnia, while intermittent or middle is when a person wakes up in the middle of the night and has difficulty getting back to sleep. Somnambulism is sleepwalking that occurs in the Stage III and IV NREM sleep. Bruxism is the clenching and grinding of the teeth that occurs during Stage II of the NREM sleep. (A.)

53. The nurse who suspects that a patient has disseminated intravascular coagulation

(DIC) should assess all of the following except:

A. I.V. and venipuncture sites B. Gastric output C. Intake and output D. Ability to ambulate with assistance

Rationale: A patient with suspected DIC must be protected from injury and complete bed rest is indicated. If the patient is combative, he should be placed in a

190

bed with padded side rails and sedatives should be administered if necessary. All potential bleeding sites, including I.V. sites, GI tract, and genitourinary tract must be monitored carefully for bleeding. In patients with acute DIC, intake and output must be monitored to assess fluid status and renal function. (D.)

54. All of the following are complications associated with septic shock except:

A. Renal failure B. GI ulcers

C. Septic arthritis D. Abnormal hepatic function

Rationale: Although it can lead to fatal septicemia, septic arthritis is not a complication of sepsis. This condition is caused by bacterial invasion of a joint. If untreated, septic arthritis eventually destroys the bone and cartilage. Renal failure, GI ulcers, abnormal hepatic function, heart failure, and disseminated intravascular coagulation are potential complications of sepsis. (C.)

55. Burn injuries are most commonly caused by:

A. Chemicals B. Moist heat or steam

C. Dry heat or fire D. Electricity

Rationale: Fire and sudden explosions of short duration (such as those caused by gasoline fumes or an electrical flash) are the most common causes of burn injury. Flash injury can cause inhalation injury because the superheated air can damage the larynx. (C.)

56. The patient does not respond to conventional treatment and the physician is concerned about lithium toxicity. The should continue to assess for all of the following signs and symptoms of lithium toxicity except:

A. Diarrhea B. Polyuria

C. Oliguria D. Muscle weakness

Rationale: Lithium toxicity is associated with lithium-induced diabetes insipidus syndrome which causes polyuria and polydipsia. Other signs and symptoms of lithium toxicity include diarrhea, muscle weakness, nausea, vomiting, drowsiness, and ataxia. (C.)

57. What is the most important immediate goal of therapy for a patient with a burn injury?

A. Maintaining fluid, electrolyte, and acid-base balance B. Planning for rehabilitation and discharge C. Providing emotional support to the patient and family D. Preserving full range of motion of all affected areas.

Rationale: Although all of the goals are important, the most immediate and life-sustaining goal is to maintain fluid, electrolyte, and acid-base balance. This helps prevent potentially life-threatening complications, including burn shock, disseminated intravascular coagulation, respiratory failure, cardiac failure, and acute tubular necrosis.

191

58. The patient requires hemodialysis. Which statement best explains the purpose of this treatment?

A. It helps improve kidney function B. It removes extra water retained by the body because of the lithium C. It removes lithium from the bloodstream D. It removes lithium because the drug is poorly soluble in water

Rationale: Hemodialysis is indicated for drug overdose in which the patient has severe poisoning from a dialyzable drug and remains unstable despite conservative treatment. It is used to remove water-soluble drugs, such as lithium, that are not highly bound to plasma protein and to correct severe acid-base and electrolyte abnormalities. Hemodialysis temporarily assists the kidneys or acts as an external kidney; however, it does not improve kidney function. Lithium toxicity does not cause the body to retain extra water. (C.)

59. Loss of circulation plasma volume occurs in burn injuries for all of the following reasons except:

A. Increased capillary permeability leads to escape of water, electrolytes, albumin, and protein into the interstitial and intracellular compartments B. Increased basal metabolic rate leads to increased water loss via the respiratory system C. Diuresis secondary to shifts in intravascular fluid further decreases intravascular volume D. Insensible loss via the burn wound ranges from 90 to 350 ml/hour, further decreasing circulating plasma volume.

Rationale: Diuresis does not occur in patients with burn wounds. Burn injuries trigger the release of antidiuretic hormone (ADH) in response to the hypovolemia. ADH decreases the production of urine by increasing water reabsorption by the renal tubules. Fluid volume is lost through increased capillary permeability which begins when the injury occurs and extends to unburned tissue if the injury involves more than 30% of the body surface area. Insensible fluid loss through burn wounds and an increased basal metabolic rate contribute to the decreased circulating plasma volume. (C.)

60. A 32-year-old male with chronic renal failure is admitted to the intensive care unit with bilateral infiltrates of the lungs. His first arterial blood gas readings reveal pH, 7.24; PaCO2, 40 mm Hg; PaO2, 64 mm Hg; and HCO3, 13 mEq/liter. His respiratory rate is 22 breaths/minute. What is the patient's acid-base status?

A. Uncompensated respiratory alkalosis with hypoxia B. Uncompensated respiratory acidosis with hypoxia C. Uncompensated respiratory acidosis with mixed metabolic alkalosis D. Uncompensated metabolic acidosis with hypoxia

192

Rationale: The patient is acidotic with a pH of 7.24. The normal PaCO2 and low HCO3 indicate a metabolic problem not compensated by the increased respiratory rate or acidosis. The patient may require mechanical ventilation because of the low PaO2 level and known pulmonary infiltrates. (D.)

61. A patient with sepsis is being treated with a cooling blanket for a fever of 103 degrees F (39.4 degrees C). How often and by which route should the patient's temperature be checked?

A. Every hour via a rectal temperature B. Every 2 hours via a rectal temperature C. Every hour via a core temperature D. Every 4 hours via a core temperature

Rationale: Unless a continuous monitoring system is available, the best method of evaluating the patient's fever is by taking a core temperature reading. Regardless of how the temperature is measured, a reading is necessary at least every hour to prevent hypothermia, which can lead to chills, shivering, and increased myocardial consumption at a time when the patient needs all of the available cardiac reserve. (C.)

Physiology Reference Section 1. Task 1.1 - Level 1 Q1 Define the following: 1.Anatomy 2.Microscopic anatomy 3.Cytology 4.Histology 5.Gross anatomy 6.Physiology 1. Anatomy is the study of the structure of the body and body parts and their relationships to one another. 2.Microscopic anatomy is concerned with structures that ca n only be seen under magnification . 3. Cytology analyses the internal structure of individual cells and the chemical substances they are composed of. 4. Histology takes a broader perspective and examines tissues, groups of specialised cells and cell products that work together to perform a particular function. 5. Gross anatomy considers features visible to the naked eye that can be studied without a microscope. 6.Physiology is the study of how the body and its parts work or junction. -examines both the physical and chemical processes of the body. Q 2. Explain what is meant by anatomical position. The subject is standing erect, feet almost together, facing forward, the arms are at the sides, and the palms of the hands are facing forward.

193

Q 3.Match each item in column A with one in column B A.Anatomical terms B. Common names Patella Front of knee Carpus Carpal one of the 8 bones of the wrist Popliteus Popliteal behind the knee joint Gluteus Gluteal buttock Cranium Cranial skull Cephalon Cephalic head Brachium Brachial arm Dorsum Dorsal top surface of foot Pollex thumb Axilla armpit Oral mouth Plants Plants sole of the foot Q.4 Explain with diagrams the three body planes Frontal (coronal) plane --extending from side to side that separates the body /organs into anterior and posterior sections (Coronal usually refers to sections passing through the skull.) Midsagittal (median) plane--a vertical plane that passes through the midline of the body dividing it into equal sections ie. left and right halves. Transverse (horizontal )plane ---a horizontal plane separates the body into upper (superior) and lower (inferior) sections.A cut in this plane is called a transverse section, a horizontal section, or a cross section.

194

Q5. What is contained in the dorsal body cavity? The dorsal body cavity is located around the brain and spinal cord consisting of: -The cranial cavity. The cranial cavity is formed by the cranial or skull bones and encloses and protects the brain. -The spinal cavity, The spinal cavity surrounds the spinal cord. The spinal cord is protected by the vertebrae which surround the spinal cavity. What is contained in the ventral body cavity? --The ventral body cavity is the larger of the two body cavities and consists of the upper thoracic and lower abdominopelvic cavity separated from each other by the diaphragm Organs in the ventral cavity are called viscera. --The thoracic cavity has two divisions. The pericardial cavity surrounding and protecting the heart and the pleural cavities each surrounding and protecting a lung. --The abdominopelvic cavity is separated into the upper abdominal cavity surrounding the organs of the stomach , liver, intestines, spleen, gallbladder, pancreas, kidneys and uteters, The lower pelvic cavity surrounds the internal reproductive organs, bladder and rectum. Q6. Name the four quadrants of the abdomen and draw them below. There is a precise division of the abdominopelvic region. Medical practitioners use a basic method of dividing the region into four known as the abdomino-pelvic quadrants. The imaginary lines intersect at the navel and divide the region into right and left upper quadrant and right and left lower quadrant. Q 7 List the 6 levels of structural organisation. -Chemical Level -Cellular Level -Tissue Level -Organ Level -System Level -Organismic Level

195

Q 8 Give an example to Illustrate the following terms. Superior Toward the head end or upper part of a structure or the body; above. Inferior Away from the head end or toward the lower part of a structure or the body; below Medial Toward or at the midline of the body; on the inner side of Lateral Away from the midline of the body; on the outer side of Proximal Close to the origin of the body part or the point of attachment of a limb to the body trunk Dorsal (posterior) Toward or at the backside of the body ;behind. Superior Medial Proximal The nose is superior The heart is medial to The elbow is proximal to to the chin the arm. the wrist( its closer to the shoulder) Inferior Lateral Dorsal The navel is inferior to The arms are lateral The heart is dorsal or the breastbone. to the chest. posterior to the breastbone. Q. 9 Name the nine body regions and give an example of the organs in each. 1. Right Hypochondriac Region: Right Lung Lower: Liver 2. Epigastric: Pancreas or Heart Pancreas 3. Left Hypochondriac Region: Left Lung Stomach or Spleen 4. Right Lumber Region: Liver Right Kidney 5. Umbilical Region: Large Intestine (Transverse Colon) Intestines 6. Left Lumbar Region: Stomach. or Spleen Left Kidney 7. Right Iliac Region. Ascending Colon Right Ovary. 8. Hypogastric Region :Bladder Uterus / Bladder/Rectum . 9. Left Iliac Region. : Descending Colon Left Ovary. Q.10 Explain each of the levels of structural organisation and how they are related to each other. The human body consists of levels of structural organisation that are related to each other and increase in complexity .Each level incorporates the structures and functions of the lower level .A person who is in good health is in a state of homeostasis where the body maintains relative internal stability despite external change. The Chemical level: Is the lowest level. Atoms the smallest units of matter combine to form molecules -this includes chemical substances essential for maintaining life. The chemical s join together to form the next level. The Cellular level.-Cells are the smallest living units of structure and function . Each type of cell has different structure according to its function. The Tissue Level - is made up of groups of cells with similar structure and function., The four main groups of tissue are epithelial tissue which covers or lines body surfaces and forms the outer layer of skin, connective tissues that connect and support parts of the body , muscle tissues that assist in movement of parts of the body and nerve tissue that transmit impulses to regulate body functions. The Organ Level consists of different types of tissue structured to produce and organ capable of complex and specific functions. The System level is a group of organs that together contribute to a specific function such as the digestive system. The Organismic level consists of all the organ systems functioning together to maintain a healthy organism.

196

Task 1.2 Q1 Briefly describe the following: 1.Cell 2..Plasma membrane 3.Nucleus Cells are mainly made up of four main elements; carbon , oxygen, hydrogen, and nitrogen. There are other trace elements such as iron sodium and potassium which are important for certain cell functions. Plasma (cell ) membrane- the outer covering which selectively controls the entry and release of substances from the cell. Nucleus- is the “control centre” of the cell. The nucleus contains chromosomes made form DNA and protein. Q2. Briefly describe the functions of a cell. -Respiration-cells require oxygen for combustion of food. -Ingestion and assimilation of the substances required for their own structure. -Growth and repair-through the use of substances from food. -Excretion-into surrounding tissue fluid of waste products. -Irritability and activity in response to physical and chemical stimuli -Metabolism- may be anabolic where energy is used to build new substances or catabolic where substances are broken down to release energy. -Reproduction- production of new cells. Q3 What is meant by “transfer of substances” in the cell. The cell needs to take nutrients and excrete waste products. This is done across the cell membrane which is selectively permeable. Methods of transfer may be passive such as diffusion, osmosis and filtration or active - moving against the natural flow. Q4.Briefly explain the meaning of mitosis, why is it important? How does it differ from meiosis? Mitosis is when one parent cell divides to for two identical daughter cells. This means the parent cell must double the genetic information. There are four stages- prophase; metaphase; anaphase; and telophase. Mitosis is essential for growth repair and replacement of damaged cells. Meiosis is when one cell divides twice and forms four cells with half the number of chromosomes. Q5. Research and describe the structure and function of 3 cells from different parts of the body. Compare and contrast the 3 cells. Similarities Differences 1. 2. 3. Task 1.3- Q.1. Name the 4 major tissue types with examples. 1.Epithelial tissue - covers or lines the body organs and is the main tissue of the skin 2.Muscle tissue- has the property of contracting to allow movement. Cardiac muscle Heart 3.Connective tissue- connects , supports and fill out the body. Soft connective tissue , loosely held together with semi-liquid material between the cells such as adipose (fat) Fibrous connective tissue is densely packed with fibres such as ligament and tendons. Hard connective tissue is firm such as cartilage or hard as in bone. Liquid connective tissue is blood and lymph. 4.Nervous tissue Only in the nervous system.

197

Q.2 .What is a membrane? Name the main types and functions of membranes. Membranes are layers of tissue which cover, line or divide. Cutaneous membrane is you skin Mucous membrane mainly found lining tubes. Serous membrane surround ing organs in the chest and stomach Synovial membranes covering joint surfaces and tendons Meninges covering the brain and spinal cord. Q.3. Describe the structure of an organ. An organ is a unit consisting of the two or more types of tissue with a specific structure and function.. Q.4. How are organs grouped together? Organs are grouped together in systems, not only because they are interconnected, but also because the organs contain the same type or types of tissue Q.5.Rresearch and explain the importance of the following: 1. Simple squamous epithelia in the alveoli of the lungs Squamous cells are flattened like fish scales Thin single layer resting on a basement membrane. The cells fit closely together, much like floor tiles. Forms membranes where filtration or exchanges of substances by rapid diffusion occur. It is the air sacs of the lungs , where oxygen and carbon dioxide are exchanged and it forms the walls of capillaries where nutrients and gased pass between the tissue cell and the blood in the capillaries, 2. Ciliated epithelia in the trachea- Pseudostratified (soo”do-stra`ti-fid) Lines most of the respiratory tract .The mucus produced by the gobler cells in this epithelium trap dust and other debris , and the cilia propel the mucus upward and away from the lings. 3. Fibrous connective tissue in the ligaments. Has collagen fibers as its main matrix element . Crowded between the collagen fibers are rows of fibroblasts(fiber-forming cells) that manufacture the fibers. Dense connective tissue forms strong, ropelike structures such as tendons and ligaments. Tendons attach skeletal muscles to bones; Ligaments connect bones to bones at joints . Ligaments are more stretchy and contain more elastic fibers than tendons. Q.6. Describe the main functions of skeletal muscles. What are the effects on skeletal muscles of (a) Exercise, (b) Ageing. These muscles, which can be controlled voluntarily (or consciously) from the flesh of the body. When the skeletal muscles contract, they pull on bones or skin. The result of their action is gross body movements or changes in our facial expressions (a) Exercise strengthens the muscles helps to function better. (b) Ageing muscles become not as strong the fibres loosen. Section 2 Task 2.1 Q.1. Describe the basic structure of the integumentary system. Integument means covering. Integumentary system consists of the skin, hair, nails and glands, I is the most visible organ system in the body and one of the largest in terms of surface area, The skin is a complex structure and is essential for survival. The skin or cutaneous membrane consists of two main layers of tissue - outer epidermis and underling dermis. Under the dermis is loose connective tissue called the subcutaneous tissue. The subcutaneous layer contains many fat cells and helps protect and insulate the inner organs. Q.2. What causes a particular skin colour? Depends on the amount and kind of melanin in the epidermis. Melanin is a yellow /brown/black pigment produced by melanocytes , responsible for colour of the skin and for protecting the body from damaging effects of ultraviolet radiation.

198

Q.3. Describe: (a) The skin appendages. (b) The cutaneous glands. (a) Hair and hair follicles-- Hair is formed in follicles which project into the dermis. The part of the hair inside the follicle is called the root the part that projects above the skin surface consisting of dead keratinized cells, is the shaft. (b) Sebaceous glands produce an oily substance to prevent dryness on the skin and hair. Sweat glands. Eccrine glands are numerous and found all over the body, particularly on the forehead, upper lip, palms of the hand and soles of the feet. This gland has a coiled tube in the dermis opening to a pore on the skin surface. It is effective in heat loss but can lead to excess water loss and dehydration. Q.4. What functions does the skin serve.? Protection-----Temperature regulation---Excretion of salts, water and organic wastes --Storage of nutrients. Q.5. (a) What might observation of different conditions of the skin tell you about general health? Wrinkles -decrease in fibres -in ageing. Dryness of skin and hair--the Sebaceous gland not producing enough oily substance -Sebum particularly in the elderly also with fragile skin. Excessive sweating in the areas of the forehead, upper lip, palms of the hands and soles of the feet.-- Over heating. Stress and emotional situations --excess sweating in the axillary and genital areas. (b) What changes to the skin might you observe in old age? Wrinkles--Drier and fragile skin.--Less response to pain , touch, pressure, and temperature. as the Nerve endings aged. Q.6. How is the integumentary system related to other body systems? Homeostatis --where the body maintains relative internal stability despite external change.The skin protects all the organs for the other systems. Task 2.2 Q. 1. Name the bones of the axial and appendicular skeleton. Axial Skeleton Appendicular Skeleton Skull = Cranium & Facial Limbs Upper Vertebral column 33 vertebral bones Clavicle 7 cervical Scapula 12 thoracic Humerus 5 fused Sacrum sacral Ulna and radius 4 fused coccyx coccygeal Carpals Metacarpals Phalanges Pectoral --shoulder Girdle Bony Thorax = sternum(breast bone) 12 pairs of ribs 12 thoracic vertebrae Pelvic Girdles Connects the trunk and lower extremities Lower Limbs=Femur Patella Tibia Fibula Tarsals Mertatarsals Phalanges

199

Q.2. What are the functions of the bones? To provide -Support - Protection - Movement - Storage - Blood cell formation Q. 3.Describe the functions of : -The Skull -The vertebral column -The rib cage -The scapula -The pelvic bone The Skull-Bony framework of the head The vertebral column-protects the spinal cord and act as a firm and flexible support for the body. The rib cage- Spaces between the ribs are filled with intercostal muscle. Contraction pulls the ribs upwards and outwards for inspiration. The diameter and volume of the chest increases and causes air to be drawn into the lungs, while relaxation of the intercostal muscles allows the rib to move downwards and inwards for expiration. The Scapula forms the posterior part of the shoulder girdle. Commonly called wings because they flare when we move our arms posteriorly. The Pelvic Bone- Pelvic Girdle - Connects the trunk and lower extremities.Bearing weight. Protectects the reproductive organs , urinary bladder, and part of the large intestine . Q.4.Describe the structure and function of a typical vertebra. How are vertebra in the different regions adapted? The vertebrae protect the spinal cord and act as a firm and flexible support for the body.Vertebrae all have similar structures and common features. The transverse processes and spinous process are for muscle attachment and the articular processes allow the vertebra to form joints with adjacent vertebrae. The vertebrae of different regions have specialised features. Q.6.How are joints classified? Give an example of each classification. A joint or artriculation is the union of two or more bones. There are 3 main classes: Fibrous or synarthroses-immovable or fixed joint eg. joints of skull bones. Cartilaginous or amphiarthroses -slightly moveable eg: intervertebral. Synovial or diarthroses- freely moveable. These have several features. Plane or gliding joint eg the carpals Ball and socket joints eg hips Hinge joint eg elbow Condyloid joint eg wrist Pivot joint eg head Saddle joint eg thumb Q.7. With a partner paint to and name each bone and find a hinge joint, a ball and socket joint,a pivot joint, a gliding joint and a saddle joint. Q.8. How is the skeletal system related to each of the other body systems Skeletal system provides some bony protection, provides support.

200

Task 2.3 Q1. (a) Name the three types of muscle. -Skeletal muscle or voluntary muscle -Smooth muscle or involuntary muscle -Cardiac muscle (b) Which muscles are voluntary and which involuntary? -voluntary skeletal muscles -involuntary Smooth muscle and cardiac. Q2. What is the meaning of antagonistic muscles? Muscles that act in opposition to an agonist or prime mover.Muscles that oppose or reverse a movement are antagonists.When a prime mover is active , its antagonist is stretched and relaxed. Q3. What are the 4 basic properties of muscle ? -Excitability ( they respond to stimulation) -contractility (can shorten to exert tension) -extensibility (contract to different degrees according to the stimulation;) -elasticity. Q4. What is the meaning of the following? -Isotonic - muscle contraction with movement occurring -Isometric - tension increasing -Muscle tone - result of systematic stimulation from the nervous system Q5. Name and describe the types of muscle actions possible in the body. -Flexion - to decrease the angle of a joint bringing bones closer together. -Extension -to increase the angle of a joint parting bones further. -Adduction - to move a limb closer to the midline. -Abduction - to move away from the midline. -Circumduction - to move in a circular motion. -Rotation - to move a bone around its longitudinal axis. -Pronation - to turn the palm down. -Supination - to turn the palm up. -Dorsiflexion - to elevate the foot. -Plantar flexion - to lower the foot (point the toes ) Q6.How is the muscular system related to the other systems in the body? All systems work together and influence each other and the muscular system through protection providing nutrients , promotes normal function, stimulation,support and maintains. Task 2.4 Q1 What is the meaning of the following -Central nervous system includes the brain and spinal cord. -Peripheral nervous system consists nerves given off from the brain and the spinal cord. -Autonomic nervous system the involuntary system. Q2.What are the general functions of the nervous system? Is the body`s most important and complex network,vital for the development of language, thought and memory, sensory perception , regulation of body function and control of movement. Q3 State the functions of different parts of the brain and draw a diagram below. Medulla reaches from the spinal cord to the pons and regulates the function of heart rate , breathing and blood pressure(vital signs) and the reflexes of coughing , sneezing, swallowing and vomiting. Pons is at the upper part of the medulla and contains respiratory centres that work with those in the medulla. Midbrain from the pons to the hypothalamus , contains centres fro visual reflexes, auditory reflexes and balance.

201

Cerebellum is concerned with movement and regulates coordination of voluntary movements and posture and equilibrium. Hypothalamus produces hormones and helps regulate body temperature and food intake; integrates the functioning of the autonomic nervous system and promotes visceral responses to emotional situations. Thalamus concerned with sensation. It integrates impulses as to body parts before relaying them to the cerebrum; suppresses unimportant sensations to permit concentration on important sensations to permit concentration on important items. Cerebrum is the largest portion of the brain. The outer portion is separated into two-the cerebral hemispheres.In general the function situated in on side of the cerebrum controls the opposite side of the body. Frontal lobes - motor area responsible for voluntary movement. Parietal lobes - sensory area that feels interprets the cutaneous senses and conscious muscle sense; taste area extends into temporal lobe, for sense of taste. Temporal lobes - the auditory areas for hearing ; olfactory areas for sense of smell and speech areas for thought before speech Occipital lobes - visual areas Association areas - abstract thinking , reasoning, learning, memory and personality. Basal ganglia - gray matter within the cerebral hemispheres; regulate subconscious accessory movements and muscle tone. Corpus callosum is a band of nerve fibres connecting the cerebral hemispheres so each hemisphere knows what is happening in the other. Q4 Describe the functions of- (a) The parasysmpathetic nervous system. nerve fibres carrying impulses to control heart beat, digestion, elimination, respiration and glandular activity. (b) The sympathetic nervous system. supplies impulses to control involuntary muscles in times of stress or damage. It prepares the body to meet physical demands Q5 What is meant by the `special senses’? Include taste, smell, sight and hearing and also the tactile functions of the skin. Q6.Describe the functioning of the eye. the sclera is the outer fibrous layer which supports the eye and is transparent in the front to form the cornea the choroid is the middle vascular layer which nourishes the eye and forms the iris with the central opening or pupil. the retina is the inner nervous layer containing the rods and cones. The eye receives the stimuli of rays of light on the retina and by means of the optic nerve transmits them to the visual centres of the brain for interpretation. Q7Describe functions of the outer, middle and inner ear. outer ear consists of the visible part of the ear and the external canal that helps to direct sound waves to the middle ear. middle ear .At the end of the auditory canal is the eardrum (tympanic ) membrane which vibrates and the sound waves are carried by three bones called auditory ossicles (stapes, incus and malleus) to the opening of the inner ear. The pressure is kept equal on both sides of the eardrum by the eustachian tube. Inner ear -The ossicles push on the membrane of the inner ear and cause fluid in the inner ear to move, this stimulates the dendrites in the cochlea with sound sensations. The inner ear also has three semicircular canals concerned with balance. Q8 Describe what happen if the following areas of the brain suffered damage. -The cerebellum.- The balance and equilibrium would not be normal . So could cause people to fall over not be able to keep their balance . Re strict normal movement -Frontal lobe - Loss of coordination and improper series of movement. Could affect speech. -Temporal lobe.- Hearing could be effected

202

Q9.What would be the result of severe damage to the second lumbar vertebra and the associated nerves? Damage to the spinal cord and the nerves in that area. Inability to extend leg and flex hip. Possibility of paralasis to the lower limbs.Unable to control bowels or bladder Q10.Trace the pathway of a light ray from outside the eye to the retina. What happens to the light ray in a person that is short sighted and how can it be corrected? The cornea ( a transparent “window” ) through which light enters the eye. . The middle coat of the eyeball is the choroid a blood - rich nutritive tunic contains a dark pigment . The pigment prevents light from scattering inside the eye. Anteriorly, the choroid is modified to form two smooth muscle structures, the ciliary body , to which the lens is attached , and the iris.The pigmented iris has a rounded opening, the pupil, through which light passes. Circularly and radially arranged smooth muscle fibres form the iris. This regulates the amount of light entering the eye so that one can see as clearly as possible in the available light. In close vision and bright light , the circular muscles contract , and the pupil constricts. In a distant vision and dim light, the radial fibres contract to enlarge (dilate ) the pupil , which allows more light to enter the eye .The innnermost sensory tunic of the eye is the delicate white retina .The retina contains millions of receptor cells, the rods and cones. Rods and cones are called photoreceptors, because they respond to light. Electrical signals pass from the photoreceptors via a two neuron chain- bipolar cells and then ganglion cells- before leaving the retina as nerve impulses that are transmitted to the optic cortex What happens to light rays in a person with short sightedness, and what to do to correct this. It occurs when the parallel light rays from distant objects fail to reach the retina and instead are focused in front of it. Therefore, distant objects appear blurry to myopic people. Correction requires concave corrective lenses that diverge the light rays before they enter the eye, so that they converge farther back. Q11 What is the relationship of the nervous system to other body systems? Necessary for activates and regulates the other systems. Task 2. 5 Q1 Name the main endocrine glands. .pituitary gland .thyroid gland .pineal gland .parathyroids .gonads .pancreas Q2 Describe the functions of 1. The pituitary gland 2. The pancreas 3. Thyroid and parathyroid 4. Thymus Pituitary gland -- Pituitary hormones control overall growth, urine production and the contractions of involuntary muscles. They also influence the activity of other glands and the pituitary is often known as the “master gland” Pancreas--produces the hormones insulin and glucagon which regulate the glucose level in the blood stream.Glucagon makes blood sugar level rise and insulin is needed for the cells to use glucose, thus lowering blood sugar level. Thyroid gland and Parathyroid --Thyroid gland helps regulate the metabolism of all body cells and is necessary for normal growth and development. Parathyroids produce a hormone which controls the body`s use of calcium and phosphorus. Thymus gland is believed to be important in defence . It atrophies in old age. Q3 Describe the relationship of the endocrine system to other systems in the body. Lymphatic System -Lymph provides a route for transport of hormones Digestive System- provides nutrients to endocrine organs Urinary System -Kidneys activate vitamin D (considered a hormone) Muscular system -protects some endocrine glands

203

Nervous System - Hypothalamus controls anterior pituitary function and produces two hormones. Respiratory System- Epinephrine influences ventilation (dilates Bronchioles) Cardiovascular System -Several hormones influence blood volume , blood pressure, and heart contractility. Reproductive System-Gonadal hormones feed back to influence endocrine system function Integumentary System-Skin produces cholecalciferol (provitamin D) Skeletal System-Protects some endocrine organs , especially those in brain ,chest, and pelvis. Task 2. 6 Q1 What organs constitute the cardiovascular system? Heart , blood vessels, lymphatic vessels, lymph nodes, spleen , blood. Q2 (a)The composition of blood. We have about 4-6 litres of blood in our bodies consist of a liquid called plasma and cells. Fibrinogen -plasma protein which helps the blood clot. Gamma Globulin -protect against some infection s and albumin to help maintain osmotic balance. Red blood cells (erythrocytes )carry oxygen to the cells and carbon dioxide away White blood cells (leucocytes) protect the body by surrounding and ingesting germs and other micro-organisms Platelets (thrombycytes ) important in blood clotting. (b) The general functions of blood. --Blood act as a transport system for essential chemical substances, oxygen and nutrients and to carry away carbon dioxide and waste products --Red cells convey oxygen to tissues and remove some carbon dioxide. --White cells provide many protective substances and protect the body against bacteria --Plasma distributes proteins from tissue functions, nourishes cells and conveys waste matter various excretory organs. --The internal secretions, hormones and enzymes are moved between organs. Q3. Describe /name: (a) The location of the heart. Lying in the thorax , between the lungs and behind the sternum, toward the left side of the body. (b) The chambers of the heart. Heart is divided into two ventricles , each has two chambers , the upper left and right atrium and lower left and right ventricle. (c) The function of the valves of the heart. The heart is equipped with four valves that allow blood to flow in only one direction through the heart chambers Q4 What is? (a) Heart rate. Closure of valves between the atria and ventricles and the closure of the aortic and pulmonary valves (b) Cardiac output is the amount of blood pumped by each side of the heart in one minute (c) Blood Pressure. Force blood exerts against the walls of the blood vessels in the cardiac cycle. It is measured in two different amounts - contraction when pressure is greatest -systolic blood pressure, and between contractions when pressure is lowest - diastolic blood pressure. Q5 State the part of the body supplied by the following arteries. .Bronchial supply the lungs .Femoral serve the thigh .Hepatic supplies the liver .Brachial supplies the arm .Inferior mesenteric supply the second half of the large intestine .Internal carotid serves the brain. .Subclavian clavicle .Intercostal supply the muscles of the thorax wall.

204

Q6.Why do doctors regularly check blood pressure in their patients? Cardiac output (the amount of blood pumped out of the left ventricles ) Peripheral resistance is the amount of friction encountered by the blood as it flows through the blood vessels. It can be increased by the constriction or narrowing , of blood vessels, especially arterioles. Increased blood volume or blood viscosity (thickness) also raises the peripheral resistance. Many factors can alter blood pressure - age , weight, time of day , exercise, body position, emotional state, and various drugs, to name a few. Vasoconstriction ,narrowing of the blood vessels which increases the blood pressure. The medulla of the brain is activated to cause vasoconstriction in many different circumstances. The kidneys play a major role in regulating the arterial blood pressure. Diet can effect blood pressure. If a diet low in salt , saturated fats, and chol- esterol can help prevent hypertension. Q7. Circulatory shock is when cardiac output decreases so tissues are deprived of oxygen. Research different types and causes of shock and record your findings When the skin is burned and its cells are destroyed, two life-threatening problems result. First the body loses its precious supply of fluids containing proteins and electrolytes as these seep from the burned surfaces. Dehydration and electrolyte imbalance follow and can lead to a shutdown of the kidneys and circulatory shock ( inadequate circulation of blood caused by low blood volume) Acute hypotension is a warning of circulatory shock. The body can compensate with blood lose up to a certain limit .Loss of 15 to 30 percent lead to pallor and weakness Losses of over 30 percent cause severe shock . Whole blood transfusion routinely given to replace substantial blood loss. Q8 Describe the relationship of the cardiovascular system to other systems in the body. The cardiovascular system delivers oxygen and nutrients and carries away wastes to all the other systems. Task 2.7 Q1 (a) Name the organs of the lymphatic system. --Lymphatic vessels, lymph nodes and lymph organs --tonsils, thymus, Peyer`s patches, (b) What is the function of the lymphatic system? It is concerned with conveying excess fluid, foreign bodies and other materials from the tissues and cells. It returns fluid and protein from tissues to circulation. Deals with waste materials and works closely with blood, particularly the white blood cells (Lymphocytes) which prevent infection entering the system and produces antibodies to protect against subsequent infection. Q2 How is the lymphatic system related to the circulatory system. The lymphatic system picks up leaked fluid and plasma proteins and returns them to the cardiovascular system. Q3. What is the location and function of: - The spleen --- One of the main filters of the blood. Old and worn out blood cells and any abnormal cells and abnormal particles floating in the blood are removed by the spleen. Gets rid of bacteria from the body and makes antibodies. -Tonsils and adenoids --Around the entrance of the pharynx .Tonsils help remove bacteria entering the throat also they produce antibodies to deal with local infections. --Adenoids are lymph glands at the back of the nose. Adenoids filter bacteria breathed in through the nose . -Peyers patches --found in the small intestine. They prevent bacteria present in the intestine from passing through the intestine wall. Q4. Why is the thymus relatively bigger in a younger person? --It establishes the body immune system in children. It shrinks in adulthood. Q5.Describe the relationship of the lymphatic system to other systems in the body. The Lymphatic vessels pick up leaked fluids and proteins in all the other systems.

205

Task 2.8 Q1. What are the organs of the respiratory system? The nose--The Pharynx (throat) --The larynx (voice box) --The trachea (windpipe)--The bronchi --The lungs and alveoli--The sinuses--Diaphragm and intercostal muscles Q2. What is the general function of the respiratory system? Function of the respiratory system is to take oxygen from the air into the blood and carry it to the tissues. Q3.Describe the structure and functions of : (a) The nasal cavities and pharynx. The interior of the nose consists of the nasal cavity.--The function is to filter, warm, humidify air; detect smells. The pharynx is a muscular passageway about 13cm long. Known as the throat , the pharynx is the passageway for food and air.---The function of the pharynx is the chamber shared with digestive tract; conducts air to larynx. (b) The trachea and bronchioles The trachea or the windpipe, air travel down from the larynx down the length of the trachea.--Function for the Trachea --Filters air, traps particles in mucus; cartilages keep airway open. Q4. Describe the action of : (a) The diaphragm. In inspiration , contraction of the muscle flattens the dome of the diaphragm causing air to be drawn into the lungs which expand to fill the enlarged thoracic cavity. Expiration , the muscle fibres of the diaphragm relax and air is forced out (b) The respiratory muscles. The diaphragm and intercostal muscles produce volume changes necessary for breathing. Q5 Explain the difference between external respiration and internal respiration. External respiration --Gas exchange ( oxygen loading and carbon dioxide unloading ) between the pulmonary blood and alveoli must take place. Internal respiration-- At systemic capillaries, gas exchanges must be made between the blood and tissue cells. Q6. Describe the relationship of the respiratory systems in the body. Respiratory system provides oxygen; disposes of carbon dioxide , to assist with the function of all the other systems. Task 2.9 Q1 Name the organs of the digestive system and accessory digestive organs. The organs of the alimentary canal --mouth , pharynx, oesophagus, stomach, small intestine and large intestine . The accessory organs --pancreas, liver, gallbladder, salivary glands and teeth. Q2 What is the general function of the digestive system? The function of the digestive system, is the process of food broken down into substances to be absorbed into the body to be used for energy, growth and repair. Q3.How is foodstuff in the digestive tract mixed and moved through the tract.? Food is chewed and mixed with saliva in the mouth Swallowed propelled by peristalsis down the pharynx --oesophagus ---stomach Stomach breaks food down into small pieces , mixes with chemicals commences breakdown - protein to amino acids, carbohydrates to simple sugars like glucose, fats to fatty acids and glycerol. The food which has been chemically broken down is held in the stomach by the sphincter muscle . The food leaves the stomach in a liquid form called chyme and passes into the intestine The small intestine

206

consists of three parts--duodenum, jejunum and the ileum .Food is further broken down here by the addition of pancreatic enzymes and bile from the liver. Food passes on to the large intestine or colon , water is removed changing waste to a more solid form . Peristalsis moves the waste to the rectum , where it is passed through the anus. Q4. What is the importance of fibre in our diets? Fiber in the diet increases the strength of common contractions and softens the stool, allowing the colon to act as a well-oiled machine. When the diet lacks bulk, the colon narrows and its circular muscles contract more powerfully ,which increases the strength on its walls Q5 Comment on the saying “We are what we eat.” Some of the food we eat is broken down to form our living flesh. A certain fraction of the nutrients are used to build cellular molecules and structures and to replace worn-out parts. Food is also metabolised to form the fuel we need for chemical energy for cells to do activities. The five food groups -Fruits -Vegetables -Grain products -Milk products - Meats and meat alternates. Have major nutrient value to our bodies so “We are what we eat ” Q6 Describe the relationship of the digestive system to other systems in the body. Digestive system provides nutrients for energy fuel , growth and repair. For all the other systems to function. Task 2.10 Q1 Name the organs of the urinary system. -Kidneys -Ureters -Bladder - Urethra Q2 Describe the function of the kidneys. -Maintaining the acid-base balance (ph) of the blood. -Secreting rennin which helps maintain correct blood pressure. -Secreting erythropoietin to help maintain the oxygen carrying capacity of the blood. -Activation of vitamin D to increase the absorption of calcium and phosphorus in the small intestine Q3 Describe the function of ureters, bladder and urethra. The ureters prevent back flow-The urinary bladder is a reservoir of collection of urine. The urethra carries urine from the bladder to the exterior. Q4 Describe the characteristics of normal urine. Amount-- can vary -1-2 litres /24 hours Colour--Straw coloured and clear. pH--Average 6 --this is affected by diet. Nitrogenous waste--Urea, Creatinine, uric acid Specific gravity --measures the dissolved material in the urine--1.010- 1.025 Q5 Describe the process of urine formation . Three processes -filtration , reabsorption, and secretion . Filtration:Water and solutes smaller than proteins are forced through the capillary walls and pores of the glomerular capsule into the renal tubule. Reabsorption: Water , glucose, amino acids, and needed ions are transported out of the filtrate into the tubule cells and then enter the capillary blood. Secretion: H+, K+. creatinine, and drugs are removed from the peritubular blood and secreted by the tubule cells into the filtrate. (H+ hydrogen, K+ potassium ) Q6 What problems may the elderly experience with the urinary system? The elderly tend not to drink enough fluids so the electrolyte balance is affected which affect the kidneys continuous process of the blood. Blood volume an be altered and so blood pressure alter. Bladder infections can occur from not enough fluids to flush out the bladder also the urine too concentrated, the bladder not being emptied properly. Incontinence of urine can occur from the weakened sphincter.

207

Q7 Describe the relationship of the urinary system to other systems in the body. Kidneys dispose of nitrogenous wastes; maintain fluid ; electrolyte and acid-based balance of blood .This is the relationship with all other systems. Task 2.11 Q1 (a) List the reproductive organs of the male. Testes, Accessory organs-Epididymis, Ductus deferens, seminal vessels, prostate, urethra External genitalia -penis ,Scrotum. (b) The female. Ovaries , Uterine tubes, Uterus, Vagina. External genitalia-Clitoris, Labia, Mammary glands Q2. Explain the function of (a) Seminal vesicles . Produce 60percent of the fluid volume of semen. (b) Prostate gland.Secretion is a milky fluid that plays a role in activating sperm (c) Testosterone. Most important hormonal product of the testes.The rising blood level of testosterone in the young male stimulates his reproductive organs to develop to their adult size , underlies the sex drive, and causes the secondary male sex characteristics to appear. (d) Ovaries : Primary female reproductive organ ,produce both and exocrine product (eggs, or ova) and endocrine product (oestrogens and progesterone.) (e) Fallopian tubes They receive the ovulated oocyte and provide a site where fertiliztion can occur. (f) Uterus. hollow organ that functions to receive , retain and nourish a fertilized egg. (g) Vagina. Provides a passageway for the delivery of an infant and for the menstrual flow to leave the body. Receives the penis during sexual intercourse. Q.3 Describe the relationship of the reproductive system to other body systems. Endocrine System- Placental hormones help to maintain pregnancy Lymphatic System /Immunity- Developing embryo/fetus escapes immune surveillance Digestive System - Digestive organs crowded by developing fetus heartburn ,constipation common during pregnancy. Urinary System -.Hypertrophy of the prostate gland inhibits urination; compression of bladder during pregnancy leads to urinary frequency and urgency Muscular System - Abdominal muscles active during childbirth; muscles of the pelvic floor support reproductive organs and aid erection of penis/clitoris Nervous System - Sex hormones masculinize or feminize the brain an influence sex drive Hypothalamus regulates timing of puberty; neural reflexes regulate sexual response. Respiratory System - Pregnancy impairs descent diaphragm causing difficult breathing provides vital oxygen and disposes of carbondioxide during pregnancy. Cardiovascular System.- pregnancy increases workload of the cardiovascular system. Integumentary System (skin) -hormone activate oil glands for lubrication skin and hair facial skin pigmentation Skeletal System - Androgens masculinize the skeleton and increase bone density ; estrogen feminizes skeleton and maintains bone mass in females. Bony pelvis encloses come reproductive organs. Task 3.1 Q1.What is the meaning of homeostasis? Homeostasis is referred to as - regulation of our body and maintaining a stable internal balance. Q2. Describe how homeostasis is maintained by : (a) Negative feedback. Variation outside normal limits triggers an autonomic response which opposes the stimulus and corrects the situation. Because the effector is activated by the control center to oppose or shut off the stimulus. (b) Positive feedback. Stimulus produces a response to reinforce the original stimulus. Q3. What happens if homeostasis is not maintained.? Use an example to explain. Homeostasis is vital to good health and poor health and disease is generally as a result of homeostatic imbalance.

208

eg. Body temperature is maintained , when temperature drops this stimulates the control centers to trigger off the effectors to warm up the body to reach the required body temperature.When the required temperature has been reached we have a happy homeostasis. If homeostasis is not functioning the body temperature could go too low and not be warmed. Or it could be too high so body over heating. Q4. Describe how body temperature is maintained. I used this as my example above. Q5. What is meant by immunity ?Resulting highly specific resistance to disease . Q6 What happens to the body during exercise ? The rate of oxygen usage increases and blood flow rate increases, this help to strengthen heart muscle and improve circulation. This is VASODILATION -VENOUS RETURN increases due to increased breathing rate. -CARDIAC OUTPUT - rises in line with increased demand to about double resting level. Increased blood flow to the active skeletal muscles and blood flows faster between the muscles, lungs and heart. -improves respiratory efficiency, circulation in the capillaries surrounding the alveoli increasing resulting in deeper breathing and better gas exchanges. Why is exercise important ? For all of the above reasons, also to prevent osteoporosis developing . To help muscle strength and joint flexibility muscle tone etc. Q7. What is the importance of good nutrition ? Involves the body using food for growth and repair and to maintain good health. Major nutrients are used to produce energy to drive the body Q8. How does the body maintain homeostasis of body fluids ? Thirst mechanism need to take in liquid , maintain the volume of water at a constant level .Intake of water needs to equal output as water is being lost from the body. Kidney activity -regulates the volume and composition of body fluids .The balance of electrolytes is mainly by the kidneys. pH of body fluids-need to be within a range of 7.4. Q9. Describe the regulation of the female reproductive cycle . Days 1-5 Menstrual period-- Breakdown of the endometrium. Bleeding Days 6-13 New follicle secreting oestrogen- Period of repair and preliminary thickening of endometrium Day 14 Ovulation. Day 15-28 --Corpus luteum secreting progesterone--Final preparation of endometrium to receive fertilised ovum. Q10 (a) Give an example of an external change and explain how the body responds to maintain homeostasis. Excessive exposure to the sun does damage to the skin .The elastic fibres clump leading to coarseness of the skin and premature wrinkling. Depresses the immune system (cold sores) .Alteraters the DNA cells and can lead to skin cancer. (b) Give an example of an internal change and explain how the body responds to maintain homeostasis. Trauma from an accident -head injury or spinal injury.--Concussion only slight damage Blow to the head may cause death from intracranial haemorrhage or cerebral oedema. Brain and spinal cord are only replaced by scar tissue.Scar tissue lacks flexibility and cannot perform normal function of tissue.

209

Q11. Select a homeostatic imbalance (disease ) from three of the systems and explain what happens in these diseases. 1.Muscular System.-Myasthenia gravies--drooping of the upper eyelids, difficulty in swallowing and talking and generalized muscle weakness and fatigability. Shortage of acetylcholine receptors at the neuromuscular junction.The muscles cells are not stimulated properly and get progressively weaker. Death usually occurs as a result of the inablility of the respiratory muscles to function.--Respiratory failure. 2. The Digestive System.Peritonitis -when the peritoneum is infected The peritoneal membranes tend to stick together around the infection site. This helps to seal off and localize many intraperitoneal infections( at least Initially) providing time for macrophages in the lymphatic tissue to mount an attack. 3. The Urinary System. -ptosis - The kidneys may drop to a lower position . Ptosis creates problems if the ureters , which drain urine from the kidneys, become kinked. When this happens , urine that can no longer pass through the uteters backs up and exerts pressure on the kidney tissue. Task 3.2 Q1. Research different types of bone fractures and how they mend. The repair of bone fractures involves four major event s 1, A haematoma is formed (blood -filled swelling ) 2.The break is splinted by a fibrocartilage callus ( mass of repair tissue) 3. The bony callus is formed (spongy bone) 4. Over the next few months, the bony callus is remodelled in response to the mechanical stresses placed on it, so that is forms a strong permanent “patch “ at the fracture site. Q2. Research burns and describe the effects of different degrees of burn and how the body works to restore homeostasis. First degree burns--only the epidermis is damaged. Area becomes red and swollen.Some temporary discomfort. Generally heal in two to three days .Sunburn eg of first - degree burn. Second-degree burns-injury to the epidermis and the upper region of the dermis.Skin is red and painful and blisters appear.Since sufficient numbers of epithelial cells are still present, regrowth (regeneration ) of the epithelium can occur. Third- degree burns destroy the entire thickness of the skin , these burns are also called full thickness burns.Appears blanched or blackened , nerve endings in the area are destroyed , the burned area is not painful.Regeneration is not possible, and skin grafting must be done to cover the underlying exposed tissues Q3. What mechanism controls the stress response? The catecholamines of the adrenal medulla prepare the body to cope with a brief or short-term stressful situation and cause a so called alarm stage of the stress response. The Clucocorticoids operate primarily during the resistance stage of the stress response. If they are successful in protectin g the body, the problem will eventually be resolved without lasting damage to the body.

DRUG REFERENCE Drug Albuterol – Ventolin 21 PO/Inhaled Classification ANS agt, Beta-adreneric agonist, Bronchodilator How Resp smooth muscle relaxant Uses Bronchospasm - acute and chronic asthma Contraindication Oral syrup in child < 2 yrs Adverse Effects fine tremors in fingers Administration Store away from direct sunlight Nursing Children 2-6 y/o prone to CNS stimulation (hyperactivity, excitement,

nervousness, insomnia), tachycardia, GI symptoms. STAT report to MD Drug-

210

induced insomnia-call MD re: take last dose hours before sleep Patient Education Correct use of drug and inhaler. Periodically check technique. Pt. also receiving

beclomethasone (Vanceril) inhalation treatment, admin albuterol 20-30 mins prior to allow deeper penetration of beclomethasone into lungs, unless otherwise directed by MD. Do not increase # or frequency of inhalations without MD ok Improvement should occur in 60-90 mins. Notify MD if drug fails to provide relief-worsening of pulmonary function. Reevaluation of condition and therapy may be indicated. Can cause dizziness or vertigo - take precautions No OTC meds without MD OK. May contain sympathomimetics that intensify drug action.

Drug Montelukast – Singulair 941 PO Classification Bronchodilator; resp stimulant, leukotriene receptor antagonist Uses Prophylaxis and chronic treatment of asthma. Contraindications Severe asthma attacks; bronchoconstriction, status asthmaticus Side Effects headache Administration Given in the evening for maximum effectiveness. Assessment Monitor effectiveness when used with phenobarbital. Patient Education Do not use for acute asthmatic attack. Inform MD if short-acting inhaled

bronchodilators are needed more often than usual while on medication. Drug Fluticasone Propionate, Salmeterol Advair Diskus Classification Corticosteriod/Beta-adrenergic Receptor Agonist Uses Asthma - long-term Contraindications Hepatic dysfunction; Status asthmaticus; Hypersensitivity; Side Effects aggravate DM and ketoacidosis; hypokalemia; HYPER: urticaria, angioedema,

rash, bronchospasm Administration Do not use to treat acute symptoms. Watch for Increasing Use - marker of

deteriorating Asthma. Do Not Exceed Recommended Dosage. Drug can produce paradoxical bronchospasm, which may be life threatening: laryngeal spasm, irritation, or swelling, stridor choking

Monitor Liver function Patient Education Bronchodilation from single dose may last up to 12h. Should not be used with a

spacer device. Never exhale into DISKUS; Never attempt to take apart; Activate and use in a level, horizontal position; Never wash mouthpiece or any part of the DISKUS; KEEP IT DRY; Always keep in a dry place; Discard 1 mos after remove from pouch or after every blister has been used. Avoid exposure to chickenpox or measles; if exposed, consult MD STAT.

Drug Theophylline/Aminoplylline 1348 PO/IV Classification Bronchodilator, CNS agt, resp and cerebral stimulant, xanthine Uses Prophylaxis & symptomatic relief of bronchial asthma Adverse Effect Tachycardia, nausea, dizziness in elderly: safety precautions Nursing PO w/ glass H2O. Give pc meals to minimize GI s/s. Timing of dose is critical.

Adhere to proper dosage intervals. Pts. with severe respiratory problems, liver dysfunction, or pulmonary edema are at greater risk of toxicity because of reduced drug clearance. Children are susceptible to CNS stimulating effects (nervousness, restlessness, insomnia, hyperactive reflexes, twitching, convulsions). Dose reduction may be indicated

Monitor VS, I&O Theophylline level: 10 – 20 ug/ml Theophylline toxicity: A,N,V,dizziness, shakiness, restlessness, abd discomfort, irritability, palpitation, tachycardia, marked HPOT, cardiac arrhythmias, seizures

Patient Education Take meds same time qd Charcoal-broiled foods increase theophylline elimination Limit caffeine intake Smoking lower theophylline concentration Drink 2 L fluids qd Nurse bfr taking drug Avoid OTC meds esp cough

211

suppressants Drug Cromolyn Sodium – Intal 374 Inhalation Classification Anti-Asthmatic; Mast cell stabilizer Anti-Inflammatory Uses Prophylaxis for asthma Contraindications CAD, hx of arrhythmias, Dyspnea; acute asthma; status asthmaticus; Side Effects Sneezing; nasal stinging and burning; irritation of throat and trachea; cough;

transient burning & stinging of eyes; Nausea Monitor Eosinophil Patient Education Clear as much mucus as possible before inhalation. Throat irritation, cough and

hoarseness relief - gargling, drinking water, sucking on a lozenge. Report: urticaria, pruritus, wheezing, edema, redness, anaphylaxis-D/C. Use 15 min before exercise.

Drug Nedocromil Sodium - Tilade 974 Inhaled Classification Anti-Asthmatic; Anti-Inflammatory, Mast cell stabilizer Use Maintenance for mild to moderate asthma. Contraindications Status asthmaticus. Side Effects Bitter taste Administration Reductions in dosage should occur in stages. Monitor Coughing, bronchospasms. Effectiveness if steroid tx is reduced. Patient Education Not for acute bronchospasms. Continue regular therapy even during symptom

free periods. Proper method of drug administration Return demo. Drug N-Acetylcysteine, Mucomyst 11 PO/Inhalation Classification Mucolytic; antidote Uses Viscid mucous; acetaminophen antidote Contraindications Pts. at risk of gastric hemorrhage. Side Effects Nausea, Vomiting b/c of drug odor, bronchospasm-D/C Administration Give when pt. arises, ac, hs. Cough before administration. Patient Education Instruct pt. to STAT report difficulty with clearing the airway or other respiratory

distress. Report nausea, can give anti-emetic. Generic Name Diphenhydramine Hcl/ Benedryl 464 PO/IV/IM Classification Anti-Histamine; H1 Receptor antagonist Uses Allergies, motion sickness, nonproductive cough, night-sedation Side Effects Drowsiness, tachycardia, dry mouth, GI distress. In elderly also dizziness &

HPOT Administration Take w/ food or milk Give 30 mins bfr for motion sickness Monitor BP Patient Education CNS depressant effects with alcohol use. Safety precautions until drug response

known. Inc fluids to expectorate, if not contraindicated. Drug Ondansetron Hcl- Zofran 1026 PO/IV Classification Anti-Emetic Use Prevention nausea and vomiting Side Effects headache, sedation, diarrhea Administration Give Tabs 30 mins < chemotherapy &1-2 h < radiation therapy. Monitor F/E status, Diarrhea, CV status: tachycardia & angina Patient Education Headache may need analgesic. Drugs Atropine Sulfate 110 IV,IM,SC,Nebulizer, Ophthalmic Classification ANS agt, anticholinergic (parasympatholytic), antimuscarinic How Blocks Acetylcholine

212

Uses Preanesthesia, arrhthmias, COPD, Uveitis, cycloplegia, antidote Contraindications Angle-closure glaucoma, tachycardia 2nd to cardiac insufficiency or

thyrotoxicosis Adverse Effect Constipation, drowsiness, dry mouth, postural HPOT aft parenteral, urinary

retention, blurred vision Monitor VS – be alert to chgs in P, R, BP, T I&O Nursing Paradoxic bradycardia after IV usually lasts 1-2 mins; most likely to occur when

IV administered slowly or when small doses are used; Infants, small children are prone to developing “atropine fever” (hyperpyrexia due to suppression of perspiration and heat loss) which increases the risk of heatstroke

Drug Valproic Acid, Depakene 1446 PO/IV Classification Anti-Convulsant; GABA inhibitor Uses Seizures, mania, migraine headache prophylaxis Contraindications bleeding disorders or hepatic dysfunction or disease. Side Effects Sedation, drowsiness; nausea, vomiting, indigestion (transient), prolonged

bleeding time. Administration Should be swallowed whole. Avoid using a carbonated drink to dilute syrup

because it will release drug from delivery vehicle. Free drug painfully irritates oral and pharyngeal membranes. Give w/food.

Monitor Therapeutic serum levels are 50-100 ug/ml. Monitor alertness. Platelets, bleeding time Liver Increased dosage increases frequency of adverse effects. Monitor pt. carefully when dose adjustments are being made and report promptly if side effects persist.

Patient Education Abrupt d/c can lead to loss of seizure control. Warn pt. not to stop or alter dosage regimen without consulting MD. Inform diabetic pt. that drug may cause a false-positive test for urine ketones. Notify MD; a differential diagnostic blood test may be indicated. If spontaneous bleeding or bruising (petechiae, ecchymotic are as, otorrhagia, epistaxis, melena) occurs, notify MD promptly. W/H dose and report to MD: visual disturbances, rash, jaundice, light colored stools, protracted vomiting, diarrhea. Fatal hepatic failure has occurred in pts. receiving this drug. Avoid alcohol and OTC meds unless MD approved. Safety precautions until drug response known. Pts. on multiple drug therapy are at risk from hyperammonemia (lethargy, anorexia, asterixis, increased seizure frequency, vomiting). STAT report symptoms. If they persist with decreased dosage the drug will be d/c. Prior any surgery (including dental) pt. should inform the MD or dentist of taking drug. Advise to carry Medic Alert information about medication and epilepsy diagnosis.

Generic Name Clonazepam/ Klonopin 348 PO Classification CNS agt, Anti-Convulsant, benzodiazepine Uses Seizures, panic disorders Contraindications Hypersensitivity to benzodiazepines, narrow-angle glaucoma Adverse Effects drowsiness, sedation, ataxia Administration Taper off. S/s of w/d: convulsion, tremor, abdominal and muscle cramps,

vomiting, sweating Psychological and physical dependence may occur with long-term, high-dose therapy. Watch that pt. does not cheek tablet. Limit availability of large amounts in addiction-prone-individual.

Monitor I&O, renal, liver, platelets, blood counts, drug efficacy O/D s/s: somnolence, confusion, irritability, sweating, muscle & abd cramps, diminished reflexes, coma.

Patient Education Report loss of seizure control Take as prescribed. NO OTC drugs without consulting MD. No driving until drug response known. Pt should carry Medic Alert ID.

213

Drug Fentanyl Citrate/ Duragesic 581 IM/IV/PO/transdermal Classification CNS agt, Narcotic Agonist, Analgesic Uses Premedication, adjunct for regional anesthesia, general anesthesia, post op pain,

chronic pain Side Effects sedation; nausea Monitor V/S – observe for signs of skeletal and thoracic muscle (depressed respirations)

rigidity and weakness Post-op: watch for respiratory depression and movements of various groups of skeletal muscle in extremities, external eye, and neck. Duration of respiratory depressant effect may be considerably longer than narcotic analgesic affect. Have available – O2, resuscitative and intubation equipment and an opioid antagonist such as naloxone.

Generic Name Erythromycin Ethylsuccinate – EES 534 PO Classification Anti-Infective, antibiotic Uses Moderate to severe infections Side Effects Nausea, abdominal cramps Contraindications Preexisting liver disease Administration PO suspensions stable for 14 days room temp. Ck exp date. Monitor Liver, blood cell counts, ototoxicity Assessment C&S prior to treatment. Cholestatic hepatitis syndrome most likely to occur in

adults who received erythromycin estolate for > 10 d or who have had repeated courses of therapy. Condition clears within 3-5 d after cessation of therapy.

Patient Education Report adverse reactions and s/s of jaundice: Report: tinnitus, vertigo, hearing impairment.

Drug Gentamicin Sulfate – Garamycin 633 IV/IM Classification Anti-Infective; Aminoglycoside antibiotic Uses Moderate to severe infection Side Effects nephrotoxicity; ototoxicity, decreased creatinine clearance; Monitor Baseline weight, V/S, and renal function tests and vestibular and auditory

function before therapy and at regular intervals during. Vestibular and auditory function should be checked again 3-4 weeks after drug is d/c (time that deafness most likely to occur). I&O S/S of ototoxicity: headache, dizziness or vertigo, N and V with motion, ataxia, nystagmus, tinnitus, sensation of fullness in ears, hearing impairment. Peak serum: 4-10 ug/ml Trough serum: 1-2 ug/ml. Blood specimens for peak serum concentrations are drawn 30min-1h after IM and 30 min after IV. For trough levels, blood specimens are drawn just before the next IM or IV dose. Collect blood in nonheparinized tubes Signs of bacterial overgrowth: diarrhea, anogenital itching, vaginal discharge, stomatitis, glossitis.

Nursing C&S initially (prior 1st dose) and periodically during therapy. If improvement does not occur in 3-5 d, susceptibility tests should be repeated and therapy reevaluated. Report oliguria, unusual appearance of urine, change in I&O ratio or pattern, and edema.

Drug Hydroxyurea – Hydrea 695 PO Classification Anti-Neoplastic; Antimetabolite Uses Chronic myelocytic leukemia; sickle cell anemia Contraindications Children; Myelosuppression Side Effects leukopenia Administration If pt. has difficulty swallowing capsule, open, mix with water, and give

immediately. Monitor Kidney, liver, bone marrow function, WBC, Hgb, WBC, platelets I&O D/C :

WBC < 2500, Platelets < 100,000 Patient Education Pt’s w/ marked renal dysfunction may develop visual & auditory hallucinations

and hematologic toxicity. Report: fever, chills, sore throat, N, V, D, loss of

214

appetite and unusual bruising or bleeding. Use barrier contraceptive during therapy. Drug is teratogenic

Drug Insulin Regular, Humulin R 720 SC/IV Classification Hormone Uses Diabetes Mellitus Contraindications Hypersensitivity to animal protein Side Effects Hypoglycemia: profuse sweating, nausea, tremulousness, palpitation Administration Insulins should not be mixed unless prescribed. Regular drawn 1st. SC

Administration - always use an insulin syringe. Regular insulin is administered 30 min before a meal. Avoid cold insulin-lipodystrophy, reduced absorption and local reaction.

Monitor Acetone with sugar in the urine=onset of ketoacidosis. STAT call MD. Monitor for hypoglycemia at peak 2 – 3 hrs. Onset 30 min – 1 h Peak 2-3 h Duration 5 – 7 h BP, I&O, BG, ketones qh w/IV. Severe hypoglycemia give: glucagon, epinephrine or IV glucose. If conscious give: oral CHO

Patient Education Active sports-inject into abdomen i/o of muscle that will be taxed. Hypoglycemia can result from excess insulin, insufficient food intake, vomiting, diarrhea, unaccustomed exercise, infection, illness, nervous or emotional tension, or overindulgence in alcohol Severe hypoglycemia is EMERGENCY. Respond STAT-4 oz of any fruit juice followed by meal of longer-acting carbohydrate or protein. Failure to recover within 30 mins = EMERGENCY TX Carry fast-acting CHO to prevent hypoglycemia. Loss of control (hyperglycemia or hypoglycemia) occurs at menses. Check BG regularly during and adjust insulin dosage accordingly Teach S/S of diabetic ketoacidosis Illness-take insulin, go to bed and drink liberally (QH) non-caloric liquids. Consult MD for insulin regulation if unable to eat Avoid OTC meds unless MD okay

Drug Insulin Isophane – NPH, Humulin N SC/IM 724 Classification Hormone Uses Diabetes Mellitus Contraindications Hypersensitivity to animal protein Side Effects Lymphedema around injection site, Hypoglycemia Administration 30 mins before 1st meal of day; second smaller dose may be prescribed 30 mins

before supper or hs. Should not be mixed with lente forms. Insulins should not be mixed unless prescribed. Regular drawn first. Intermediate Acting Onset 1-2 h Peak 8 – 12 h Duration 18 – 24 h

Monitor Hypoglycemia: fatigue, weakness, sweating, tremor, nervousness Patient Education Insulin given before breakfast, hypoglycemia likely between midafternoon and

dinnertime - peaking. Eat a snack midafternoon. Carry sugar/candy to treat a reaction. Bedtime snack will prevent reaction during night. Teach S/S of hypoglycemia and hyperglycemia

Generic Name Albuterol Trade Name Ventolin Classification Bronchodilator Uses Bronchospasm - acute and chronic asthma Contraindications Oral syrup in child < 2 yrs Adverse Effects CNS: tremor Administration Store away from direct sunlight Assessment

Fine tremors in fingers - may interfere with precision handwork. Inform MD of unusual symptoms Children 2-6 y/o prone to CNS stimulation (hyperactivity, excitement, nervousness, insomnia), tachycardia, GI symptoms. STAT report to MD

215

Drug-induced insomnia-call MD re: take last dose hours before sleep Patient Education

Correct use of drug and inhaler. Periodically check technique. Pt. also receiving beclomethasone (Vanceril) inhalation treatment, admin albuterol 20-30 mins prior to allow deeper penetration of beclomethasone into lungs, unless otherwise directed by MD. Do not increase # or frequency of inhalations without MD ok

Improvement should occur in 60-90 mins. Notify MD if drug fails to provide relief-worsening of pulmonary function. Reevaluation of condition and therapy may be indicated. Can cause dizziness or vertigo - take precautions No OTC meds without MD OK. May contain sympathomimetics that intensify drug action.

Generic Name Aminophylline Trade Name Theophylline Classification Bronchodilator Uses Prevent and relieve symptoms of acute bronchial asthma Contraindications Hypersensitivity to xanthine derivatives; Cardiac arrhythmias; Adverse Effect CNS: nervousness CV: cardiac arrhythmias; cardiac arrest

GI: nausea, vomiting, anorexia Administration Oral with glass H2O on empty stomach ½ - 1h ac or 2h pc. Absorption may be

delayed by food. Minimize GI s/s by taking pc or with food.

ER should not be chewed or crushed; if tablet is scored, it can be broken in half, then swallowed; or contents of ER capsules may be mixed with soft, moist food and swallowed without chewing Rectal preps are ordered for pt. who must fast or cannot tolerate orally. Drug absorption is enhanced if rectum is empty. Parenteral (IV) - Rapid infusion may cause cardiac arrest. Monitor rate carefully

Assessment

Toxicity r/t serum levels over 20ug/ml (NL: 10-20 ug/ml); High incidence of toxicity assoc. with rectal suppository 2nd to erratic absorption rate. Parenteral - observe for hypotension, arrhythmias and convulsions until serum level stabilizes within therapeutic range. Monitor V/S and I&O. Improvements in quality & rate of pulse and respirations, diuresis are expected effects. Sudden, sharp unexplained rise in HR is an indicator of toxicity. Pts. with severe respiratory problems, liver dysfunction, or pulmonary edema are at greater risk of toxicity because of reduced drug clearance. Children are susceptible to CNS stimulating effects (nervousness, restlessness, insomnia, hyperactive reflexes, twitching, convulsions). Dose reduction may be indicated

216

Patient Education

Smoking tends to increase elimination (prolongs half-life); dose requirements may be higher and dose intervals shorter than in non-smokers

Report excessive nervousness or insomnia. Dose reduction may be indicated Dizziness is a common side effect, esp. in elderly. Advise pt. to take precautions OTC asthma or cough remedies contain ephedrine with various salts of theophylline. Take only meds approved by MD

Generic Name Atropine Sulfate Trade Name Classification Uses Irritable bowel syndrome Contraindications Hypersensitivity to belladonna alkaloids; synechiae; parotitis; obstructive

uropathy; intestinal atony; parlytic ileus, obstructive disease of GI tract, severe ulcerative colitis, toxic megacolon;

Adverse Effect CV: ventricular fibrillation Administration Smaller doses indicated for elderly Assessment Monitor V/S - Pulse is an indicator of response. *Alert to change in quality, rate,

and rhythm of Pulse & Respirations and changes in BP & Temperature

Paradoxic bradycardia after IV usually lasts 1-2 mins; most likely to occur when IV administered slowly or when small doses are used; Postural hypotension when pt. ambulates too soon after parenteral admin. May contribute to urinary retention. Palpate lower abdomen for distension. Monitor I&O. Have pt. void before giving med. Constipation – check for abdominal distension and ausculate for bowel sounds Infants, small children are prone to developing “atropine fever” (hyperpyrexia

due to suppression of perspiration and heat loss) which increases the risk of

heatstroke

Infants, children with spastic paralysis, brain damage or Down syndrome and

blonde, blue-eyed individuals appear to be highly sensitive to the effects of

atropine and related drugs.

Patient Education Keep dental appts - Saliva is a natural mouthwash, teeth are vulnerable to decay when salivation is suppressed. Dry mouth relief: hydration; small, frequent mouth rinses with tepid water; meticulous mouth and dental hygiene; gum chewing or sucking hard, sour candy (sugarless)

Generic Name Clonazepam Trade Name Klonopin Classification Anti-Convulsant

217

Uses Seizures Contraindications Hypersensitivity Adverse Effects CNS: drowsiness, sedation, ataxia, COMA

RESP: RESPIRATORY DEPRESSION Administration New anti-convulsant substitution-add to regimen as former med is gradually w/d.

Slow tapering over several days is imperative. Abrupt w/d in pt. on high doses or long-term therapy can precipitate status epilepticus. Other s/s: convulsion, tremor, abdominal and muscle cramps, vomiting, sweating

Assessment

Monitor I&O ratio and renal function; Excess accumulation of metabolites 2nd to impaired excretion leads to toxicity. If multiple anti-convulsants are being given, watch pt. carefully for signs of overdose or drug interaction. eg: increased depressant adverse effects. Liver function tests, platelet counts, blood counts, and clinical evaluation of drug efficacy should be part of follow up care Psychological and physical dependence may occur with long-term, high-dose therapy. Watch that pt. does not cheek tablet. Limit availability of large amounts in addiction-prone-individual. Overdose: somnolence, confusion, irritability, sweating, muscle and abdominal cramps, diminished reflexes, coma.

Patient Education

Anti-convulsant activity is often lost after 3 mos of therapy; dosage adjustment may reestablish. Pt. should be aware of necessity to report loss of seizure control STAT. Take as prescribed-do not alter regimen or stop without MD okay. NO OTC drugs without consulting MD. Drowsiness occurs in approximately 50% of pts. Pt should carry Medic Alert ID.

Generic Name Cromolyn Sodium Trade Name Intal Classification Anti-Asthmatic; Anti-Inflammatory Uses Bronchial asthma; Prevent exercise related bronchospasm; Contraindications

Dyspnea; acute asthma; status asthmaticus; pt. unable coordinate actions/follow instructions; children under 6 y/o; {capsule not recommended for children}

Side Effects ENT: Sneezing; nasal stinging and burning; irrigation of throat and trachea; cough; EYE: transient burning, stinging GI: Nausea HYPER: Angioedema; Bronchospasm; Anaphylaxis

Administration Therapeutic effect is dependent on proper inhalation.

Advise pt. to clear as much mucus as possible before inhalation. Exhale as much as possible before placing mouthpiece between lips, tilt head back and inhale rapidly & deeply with steady, even breaths. Remove inhaler from mouth, hold breath for a few seconds, exhale into the air. Repeat until entire dose is taken. Don’t exhale into inhaler- moisture will interfere with operation. Capsule is for inhalation only - ineffective if swallowed.

218

Assessment

Hx advisable prior to tx - Capsules contain lactose vehicle-pt. with lactose deficiency may react. Exacerbation of symptoms including breathlessness and cough may occur in pts. receiving drug during cortiocosteriod w/d. Drug does not eliminate need for therapy with bronchodilators, expectorants, antibiotics, or corticosteroids - amount and frequency of these meds may be reduced. Eosinophil count is an indicator of allergy and should be monitored. Therapeutic effects may be noted in few days to 2 weeks.

Patient Education

Throat irritation, cough and hoarseness relief - gargling, drinking water, sucking on a lozenge. Specific instructions re: what to do in an acute asthmatic attack. Drug is of no value with acute asthma. Report unusual s/s: urticaria, pruritus, wheezing, edema, redness, anaphylaxis. Hypersensitivity can be severe and life-threatening. D/c medication if an allergic reaction occurs. Treatment 15 min before protracted exercise blunts effects of vigorous exercise as well as cold air.

Generic Name Diphenhydramine Hcl Trade Name Benedryl Classification Anti-Histamine; Uses

Temporary relief various allergic conditions; Treatment and prevent motion sickness, vertigo; Anaphylaxis as adjunct to epinephrine;

Contraindications

Hypersensitivity; Lower RT symptoms (incl. acute asthma); children under age 12 yrs;

Side Effects CNS: drowsiness; CV: tachycardia; cv collapse; GI: dry mouth; HYPER: anaphylactic shock

Administration GI side effects (Oral) lessened by admin with food +/or milk. Motion sickness: 1st dose 30 mins prior exposure to motion; for duration of exposure: give before meals and on retiring. IM deep into muscle mass; avoid SQ 2nd to irritating effects; hypersensitivity reactions likely with parenteral administration. Elixir or syrup formulations are used for cough relief.

Assessment

Close monitor pts. with high BP problems receiving IM Drowsiness is most prominent 1st days therapy and disappears with continued therapy. Elderly pts. likely to manifest dizziness, sedation, and hypotension. Side rails and ambulation caution.

219

Patient Education

Possible additive CNS depressant effects with concurrent use of alcohol and other CNS depressants. Caution with activities requiring alertness and coordination until reaction known. Atropine-like drying effect (thickens bronchial secretions) may make expectoration difficult. Increase fluid intake if not contraindicated.

Generic Name Erythromycin Ethylsuccinate Trade Name EES Classification Anti-Infective; Uses Pneumonia; Prophylaxis of rheumatic fever; Contraindications Hypersensitivity; Preexisting liver disease Side Effects GI: Nausea, abdominal cramps; Administration PO without regard to meals in pts. over 2 y/o;

Chewable tabs – chew – not swallow whole; Assessment

C&S prior initiate therapy. Cholestatic hepatitis syndrome most likely to occur in adults who received erythromycin estolate for > 10 d or who have had repeated courses of therapy. Condition clears within 3-5 d after cessation of therapy. Hepatic function tests and blood cell counts should be conducted periodically if therapy is prolonged 10 d or more.

Patient Education

Advise report STAT onset adverse reactions. Alert s/s associated with jaundice: Ototoxicity most likely to occur in pt. receiving high dose or have impaired renal function. STAT report: tinnitus, vertigo, hearing impairment.

Generic Name Fentanyl Citrate Trade Name Duragesic Classification Narcotic Analgesic; Uses

Short-acting analgesic - operative and peri-operative periods; Narcotic analgesic supplement in general and regional anesthesia;

Contraindications Children under 2 y/o.; Side Effects CNS: sedation; CV: circulatory depression; cardiac arrest

GI: nausea; RESP: laryngospasm, resp. depression or arrest Administration Parenteral

Assessment

V/S – observe for signs of skeletal and thoracic muscle (depressed respirations) rigidity and weakness; Post-op: watch for respiratory depression and movements of various groups of skeletal muscle in extremities, external eye, and neck. These may present pt. management problems - STAT report. Duration of respiratory depressant effect may be considerably longer than narcotic analgesic affect. Have STAT available – O2, resuscitative and intubation equipment and an opioid antagonist such as naloxone.

Patient Education N/A Generic Name Gentamicin Sulfate Trade Name Garamycin Classification Anti-Infective; Uses

Serious infections: GI, respiratory and urinary tracts, CNS, bone, skin and soft tissue (including burns);

220

Contraindications Hypersensitivity/toxic reaction with any aminoglycoside anti-biotic. Side Effects RENAL: nephrotoxicity; decreased creatinine clearance; Administration Parenteral IV

Pediatric: amount of infusion fluid may be proportionately smaller depending on

pts. needs but should be sufficient to be infused over the same time period as

adults.

Commercial piggyback preparations contain no preservatives and must be used promptly. Discard any unused portion Medication for IV or IM administration is clear and colorless or slightly yellow. Medication for intrathecal use is clear and colorless. Discard discolored or particulate matter solutions. Ophthalmic Immediately apply pressure to inner canthus for 1 min after instillation of drops. Instruct pt. to keep eyes closed for 1-2 mins to assure contact. Caution pt. that vision will be blurred for a few minutes. Topical Wash area with mild soap and water, rinse, and dry thoroughly, unless otherwise prescribed. Gently apply small amount of medication to lesions. Cover with sterile gauze if desired. Topical applications, particularly cream preparations, should not be made to large denuded body surfaces because systemic absorption and toxicity are possible.

Assessment

Parenteral C&S initially (prior 1st dose) and periodically during therapy. If improvement does not occur in 3-5d, susceptibility tests should be repeated and therapy reevaluated. Baseline weight, V/S, and renal function tests and vestibular and auditory function should be determined before therapy and at regular intervals during. Vestibular and auditory function should be checked again 3-4 weeks after drug is d/c (time that deafness most likely to occur). Creatinine and serum drug concentrations - determined at frequent intervals, esp. pts. with impaired renal function, infants (renal immaturity), the elderly, and pts. receiving high doses or therapy beyond 10 d, pts. with fever or extensive burns, edema, obesity. I&O-well hydrated during therapy to prevent chemical irritation of renal tubules. Report oliguria, unusual appearance of urine, change in I&O ratio or pattern, and edema (prolongs elimination time) Ototoxic effect is greatest on vestibular branch of 8th cranial (acoustic) nerve. Symptoms: headache, dizziness or vertigo, N and V with motion, ataxia, nystagmus). Damage to the auditory branch (tinnitus, roaring noises, sensation of fullness in ears, hearing impairment) may also occur. Prompt report to prevent permanent damage. Dosages are adjusted to maintain peak serum concentrations of 4-10 ug/ml and

221

trough concentrations of 1-2 ug/ml. Peak concentrations above 12 ug/ml and trough concentrations above 2 ug/ml are associated with toxicity. Blood specimens for peak serum concentrations are drawn 30min-1h after IM admin and 30 min after completion of 30-60 min IV infusion. For trough levels, blood specimens are drawn just before the next IM or IV dose. Collect blood in nonheparinized tubes. Alert for signs of bacterial overgrowth (opportunistic infections) with resistant or nonsusceptible organisms (diarrhea, anogenital itching, vaginal discharge, stomatitis, glossitis)

Patient Education

Pts. using topical applications: 1 – avoid excessive exposure to sunlight-danger of photosensitivity; 2 – w/d and notify MD if condition fails to improve within 1 wk, worsens, or signs of irritation or sensitivity occur; 3 – apply as directed and only for time prescribed (overuse can result in superinfections)

Generic Name Hydroxyurea Trade Name Hydrea Classification Anti-Neoplastic; Uses Chronic myelocytic leukemia Contraindications Children; Myelosuppression; Side Effects HEMAT: bone marrow suppression (leukopenia); Administration If pt. has difficulty swallowing capsule, open, mix with water, and give

immediately. Assessment

Kidney, liver, bone marrow function-determine prior and periodic. Hgb, WBC, and platelet - monitor at least once weekly. If WBC drops to 2500/mm or platelets to 100,000/mm – therapy will be interrupted. Monitor I&O. Advise patients with high serum uric acid levels to drink at least 10-12 glasses of fluid QD to prevent uric acid nephropathy. Pts. with marked renal dysfunction may rapidly develop visual and auditory hallucinations and hematologic toxicity.

Patient Education

Toxicity incidence is 66% with doses of 40mg/kg body weight. Report fever, chills, sore throat, N, V, D, loss of appetite and unusual bruising or bleeding. Use barrier contraceptive during therapy. Drug is teratogenic.

Generic Name Insulin Regular Trade Name Humulin R Classification Insulin Uses IDDM; With intermediate or long-acting to provide control BG Contraindications Hypersensitivity to animal protein Side Effects Hyper: anaphylaxis (rare); Hypoglycemia: profuse sweating, nausea,

tremulousness, palpitation, coma Administration Insulins should not be mixed unless prescribed. Regular drawn 1st.

SC Administration - always use an insulin syringe. Regular insulin is administered 30 min before a meal. Avoid cold insulin-lipodystrophy, reduced absorption and local reaction.

222

Assessment

Frequency of BG monitoring determined by regimen and health status of the pt. Urine for ketones in new, unstable, and juvenile diabetes; pt. has lost weight, exercises vigorously or illness and BG elevated. Acetone without sugar in the urine = insufficient carbohydrates Acetone with sugar in the urine=onset of ketoacidosis. STAT call MD. Monitor for hypoglycemia at peak. Onset of hypoglycemia [blood sugar: 50-40 mg/dl] may be rapid and sudden. Severe hypoglycemia-glucagon, epinephrine or IV glucose. As soon as pt. is fully conscious, oral carbohydrate (dilute corn syrup, OJ with sugar, Gatorade, Pedialyte) should be given to prevent secondary hypoglycemia.

Patient Education

Active sports-inject into abdomen i/o of muscle that will be taxed. Hypoglycemia can result from excess insulin, insufficient food intake, vomiting, diarrhea, unaccustomed exercise, infection, illness, nervous or emotional tension, or overindulgence in alcohol Severe hypoglycemia is EMERGENCY. Respond STAT-4 oz of any fruit juice followed by meal of longer-acting carbohydrate or protein. Failure to recover within 30 mins = EMERGENCY TX Carry fast acting carbohydrate [lump sugar, life savers or other candy] at all times to prevent hypoglycemia Loss of control (hyperglycemia or hypoglycemia) occurs at menses. Check BG regularly during and adjust insulin dosage accordingly Inform of S/S of diabetic ketoacidosis Illness-take insulin, go to bed and drink liberally (QH) non-caloric liquids. Consult MD for insulin regulation if unable to eat Avoid OTC meds unless MD okay

Generic Name Insulin Isophane – NPH Trade Name Humulin N Classification Insulin Uses Hyperglycemia Contraindications Hypersensitivity to animal protein Side Effects Hyper: anaphylaxis (rare); Hypoglycemia: profuse sweating, nausea,

tremulousness, palpitation, coma Administration 30 mins before 1st meal of day; second smaller dose may be prescribed 30 mins

before supper or hs. Rotate between palms and invert end to end - Do not shake. May be mixed with insulin injection without altering either solution. Should not be mixed with lente forms. Insulins should not be mixed unless prescribed. Regular drawn first.

Assessment Hypoglycemia: fatigue, weakness, sweating, tremor, nervousness

223

Patient Education

Insulin given before breakfast, hypoglycemia likely between midafternoon and dinnertime - peaking. Eat a snack midafternoon and to carry sugar/candy to treat a reaction. Bedtime snack will prevent reaction during night. Teach S/S of: Hypoglycemia – acute fatigue, restlessness, malaise, marked irritability and weakness, cold sweats, excessive hunger, headache, dizziness, confusion, slurred speech, loss of consciousness and death. Hyperglycemia –flushed, dry skin, low BP and elevated pulse, tachypnea, Kussmaul’s respirations, polyuria, polydipsia, polyphagia, lethargy and drowsiness.

Generic Name Montelukast Trade Name Singulair Classification Bronchodilator; Uses Prophylaxis and chronic treatment of asthma. Contraindications

Hypersensitivity; severe asthma attacks; bronchoconstriction due to asthma or NSAID’s; status asthmaticus

Side Effects CNS: headache Administration Given in the evening for maximum effectiveness.

Chewable tablets for children should not be swallowed whole.

Assessment

Therapeutic effectiveness indicated by improved pulmonary function and better-controlled asthma symptoms. Monitor effectiveness when used with phenobarbital.

Patient Education

Should not be used for reversal of an acute asthmatic attack. Inform MD if short-acting inhaled bronchodilators are needed more often than usual while on medication. Chewable tablets contain phenylalanine and should be used with caution by phenylketonurics.

Generic Name N-Acetylcysteine Trade Name Mucomyst Classification Mucolytic; Uses

Adjuvant in pts. with abnormal, viscid, or inspissated mucous secretions in acute and chronic bronchopulmonary diseases; pulmonary complications of cystic fibrosis and surgery, tracheostomy, and atelectasis.

Contraindications Hypersensitivity; Pts. at risk of gastric hemorrhage. Side Effects GI: Vomiting Administration Treatment administered when pt. arises, before meals, and prior to retiring at

night. For maximum effect, instruct pt. to clear airway, if possible, coughing productively prior to aerosol administration.

Assessment

Have suction apparatus available. Increased volume of respiratory tract fluid may be liberated; suction or endotracheal aspiration may be necessary to establish and maintain an open airway. Elderly and debiltated pts. require close monitoring to prevent aspiration of excessive secretions. Bronchospasm is most likely to occur in pts. with asthma, and it may happen unpredictably. If it occurs, drug should be d/c STAT.

224

Unpleasant rotten egg odor and excess volume of liquefied bronchial secretions may cause nausea and vomiting, esp. when face mask is used. Odor becomes less noticeable with continued inhalation.

Patient Education

Instruct pt. to STAT report difficulty with clearing the airway or other respiratory distress. Advise pt. to report nausea, as an anti-emetic may be indicated.

Generic Name Nedocromil Sodium Trade Name Tilade Classification Anti-Asthmatic; Anti-Inflammatory; Uses Mild to moderate asthma. Contraindications

Hypersensitivity; Acute bronchospasm, particularly status asthmaticus.

Side Effects GENERAL: abnormal bitter taste Administration Correct administration essential for maximum drug efficacy.

Reductions in dosage should occur in stages.

Assessment

Assess medication induced coughing and bronchospasms. Indicate need to d/c drug and should be reported. Monitor pts. for whom systemic or inhaled steroid therapy has been reduced, as medication may not fully substitute for the decrease in dose of steroid.

Patient Education

Because medication is not a bronchodilator, instruct not to use drug to treat acute bronchospasms. Stress need to continue regular therapy even during symptom free periods. Instruct on proper method of drug administration. Review pt. instruction leaflet.

Generic Name Ondansetron Hcl Trade Name Zofran Classification Anti-Emetic; Uses Prevention and Post-Op nausea and vomiting; Contraindications Hypersensitivity Side Effects CNS: headache, sedation; GI: diarrhea Administration Administer tablets 30 mins prior to chemotherapy and 1-2 h prior to radiation

therapy. Assessment

Monitor fluid and electrolyte status. Diarrhea, which may cause imbalance, is a potential adverse effect of the drug. Monitor cardiovascular status, especially in pts. with history of CAD. Rare cases of tachycardia and angina have been reported.

Patient Education

Inform that headache requiring an analgesic for relief is a common adverse effect.

Generic Name Valproic Acid Trade Name Depakene Classification Anti-Convulsant; Uses Absence (petit mal) and mixed seizures; Contraindications bleeding disorders or hepatic dysfunction or disease. Side Effects CNS: drowsiness; GI: nausea, vomiting, indigestion (transient), hepatic failure;

HEMAT: prolonged bleeding time; bone marrow depression; OD: deep coma, death

225

Administration Should be swallowed whole. Avoid using a carbonated drink dilutent for the syrup because it will release drug from delivery vehicle. Free drug painfully irritates oral and pharyngeal membranes.

Serious GI side effects can lead to d/c therapy. To reduce gastric irritation, administer drug with food. Enteric-Coated tablet or syrup formulation is usually well tolerated. Abrupt d/c can lead to loss of seizure control. Warn pt. not to stop or alter dosage regimen without consulting MD.

Assessment

Therapeutic serum levels are 50-100 ug/ml.

Monitor alertness in pt. on multiple drug therapy for seizure control. Plasma levels of the adjunctive anti-convulsants should be evaluated periodically as indicators for possible neurologic toxicity. Increased dosage increases frequency of adverse effects. Monitor pt. carefully when dose adjustments are being made and report promptly if side effects persist. Platelet counts & bleeding time recommended prior to treatment and periodic. Liver function tests should be performed initially and at least q2mo, especially during the first 6mo of therapy.

Patient Education

Inform diabetic pt. that drug may cause a false-positive test for urine ketones. Notify MD; a differential diagnostic blood test may be indicated. If spontaneous bleeding or bruising (petechiae, ecchymotic areas, otorrhagia, epistaxis, melena) occurs, notify MD promptly. Withhold dose and report to MD: visual disturbances, rash, jaundice, light colored stools, protracted vomiting, diarrhea. Fatal hepatic failure has occurred in pts. receiving this drug. Avoid alcohol and self-medication with other depressants during therapy. All OTC drugs should be approved by the physician. Unsafe: combination drugs containing aspirin, sedatives and medications for hay fever or other allergies. Mental alertness and physical coordination reaction warning. Pts. on multiple drug therapy are at risk from hyperammonemia (lethargy, anorexia, asterixis, increased seizure frequency, vomiting). STAT report symptoms. If they persist with decreased dosage the drug will be d/c. Prior any surgery (including dental) pt. should inform the MD or dentist of taking drug. Advise to carry Medic Alert information about medication and epilepsy diagnosis.

Generic Name Fluticasone Propionate and Salmeterol Xinafoate Trade Name Advair Diskus Classification Corticosteriod/Beta-adrenergic Receptor Agonist Uses Asthma - long-term Contraindications Hepatic dysfunction; Status asthmaticus; Hypersensitivity; Side Effects METABOLIC: aggravate DM and ketoacidosis; hypokalemia;

HYPER: urticaria, angioedema, rash, bronchospasm

226

Administration Not used during deteriorating or potentially life-threatening episodes of asthma. Do not use to treat acute symptoms. Watch for Increasing Use - marker of deteriorating Asthma. Do Not Exceed Recommended Dosage. Drug can produce paradoxical bronchospasm, which may be life threatening. Symptoms of laryngeal spasm, irritation, or swelling, such as stridor and choking, have been reported. Use with caution in patients with cardiovascular disorders.

Assessment Impaired liver function may lead to accumulation of drug in plasma. Close monitor pts with hepatic disease

Patient Education Bronchodilation from single dose may last up to 12h. Other inhaled drugs and asthma meds should be used only as directed. Should not be used with a spacer device. If pregnant or nursing, contact MD about the use. Effective and safe use: Never exhale into DISKUS; Never attempt to take apart; Activate and use in a level, horizontal position; Never wash mouthpiece or any part of the DISKUS; KEEP IT DRY; Always keep in a dry place; Discard 1 mos after remove from pouch or after every blister has been used. Avoid exposure to chickenpox or measles; if exposed, consult MD STAT.

Common Cardiac Medications

ACE inhibitors: Accupril (Quinipril) Aceon (Perindopril) Altace (Ramipril) Capoten (Captopril) Lotensin (Benzepril)

Mavik (Trandolopril) Monopril (Fosinopril) Prinvil, Zestril (Lisinopril) Univasc (Moexipril) Vasotec (Enalapril)

ACE inhibitors block enzymes that lead to the production of substances that can increase the workload of the heart. Blocking these substances helps to lower the blood pressure and reduce the amount of work the heart does. ARB (Angiotensin Receptor Blockers) Atacand (Candesartan) Avapro (Irbestartan) Cozaar (Losartan)

Diovan (Valsartan) Micardis (Telmisartan) Teveten (Eprosartan)

ARBs blocks a substance that constricts the blood vessels. This helps to lower the blood pressure and reduce the amount of work the heart does. They are often used as alternatives to or in conjunction with ACE inhibitors.

227

Nitrates: Ismo, Monoket, Imdur (Isosorbide mononitrate) Isordil, Dinitrate SR (Isosorbide dinitrate) Nitro-Dur, Transderm (Nitroglycerin Tranderm patches, and sublingual tabs) Nitrates act as vasodilators and arteriodilators which reduces venous return (preload) and decreases cardiac output (afterload). They lower systolic pressures and peripheral resistance and enhance oxygen supply by dilation of the coronary arteries. Direct Vasodilators: Apressoline (Hydralazine hydrochloride) Loniten (Minoxidil)

Nipride (Sodium nitroprusside) Hyperstat (Diazoxide)

Vasodilators dilate the arteries of the body, reducing the amount of work on the heart. Diuretics: Loop Diuretics Bumex (Bumetanide) Demadex (Torsemide)

Lasix (Furosemide)

Thiazide and Thiazide-like HCTZ, HydroDIURIL (Hydrochorothiazide) Lozol (Indapamide)

Zaroxolyn (Metolazone)

Potassium Sparing Diuretics Aldactone (Spironolactone) Dyazide (Triamterene)

Moduretic (Amiloride) Maxzide (Triamterene/Hydrochlorothiazide)

Diuretics work by “tricking” the kidneys into releasing salt and water by decreasing their reabsorption. Glycosides: Digoxin (Lanoxin) Glycosides stimulate the force of cardiac contraction, and alter the electric impulse generation and conduction in the heart to control rate and rhythm. Beta-Blockers Coreg (Carvedilol) Toprol, Lopressor (metoprolol) Zebeta (Bisoprolol) Inderal (Propranolol)

Tenormin (Atenolol) Corgard (Nadolol) Levatol (Penbutolol)

Beta-Blockers reduce blood pressure by blocking sympathetic effects on the heart, thus decreasing the heart rate and cardiac output. They also decrease blood pressure by blocking adrenergic nerve mediated release of rennin, thereby decreasing peripheral vascular resistance. Sexual Side Effects.

228

Alpha-Blockers Minipress (Prazosin) Hytrin (Terazosin)

Cardura (Doxazosin Mesylate)

Alpha-blockers act by blocking the sympathetic vasoconstrictor impulses at receptors in vascular smooth muscle, which decreases peripheral vascular resistance and reduces blood pressure.

Alpha-Beta Blockers Normodyne, Trandate (Labetalol)

These combine the effects of Alpha and Beta blockers Calcium Channel Blockers Norvasc (Amlodipine) Cardizem, Tiazac, Cartia (Diltiazem) Procardia (Nifedipine)

Calan, Isoptin, Verelan (Verapamil hydrochloride) DynaCirc (Isadipine)

Calcium channel blockers prevent the influx of calcium ions into vascular muscle, decreasing peripheral vascular resistance and blood pressure. Antiarrhythmic Drugs Cordarone (Amiodarone)

Prolongs the duration of the action potential of atrial and ventricular muscles and prolongs the repolarization phase. This slows the heart rate.